...

12 KENDRIYA VIDYALAYA SANGATHAN DEHRADUN REGION

by user

on
Category: Documents
412

views

Report

Comments

Transcript

12 KENDRIYA VIDYALAYA SANGATHAN DEHRADUN REGION
केंिीय िवद्यालय-संगठन
KENDRIYA VIDYALAYA SANGATHAN
दे हरादन
ू -संभाग
DEHRADUN REGION
अध्ययन-साममी
STUDY MATERIAL
कक्षा: 12
CLASS: 12
िहं दी(केंििक)
HINDI (CORE)
2012-13
1 केंिीय िवद्यालय-संगठन
दे हरादन
ू -संभाग
अध्ययन-साममी
कक्षा-12, िहं दी(केंििक)
2012-13
मुख्य संरक्षक: ौी अिवनाश दीिक्षत,
आयुक्त, केंिीय िवद्यालय-संगठन, नई िदल्ली
संरक्षक
: ौी नरे न्ि िसंह राणा,
उपायुक्त, केंिीय िवद्यालय-संगठन,दे हरादन
ू -संभाग
सलाहकार
िनदेर् शक
: ौी सरदार िसंह चौहान,सहायक आयुक्त, केंिीय िवद्यालय-संगठन,
दे हरादन
ू -संभाग
: ौी दे वी ूसाद ममगाईं,ूाचायर्, केंिीय िवद्यालय, बमांक-२
भा.स.िव. हाथीबडकला, दे हरादन
ू
सह िनदेर् शक
:ौीमती भारती दे वी राणा,
ूाचायर्, केंिीय िवद्यालय, आइ.एम ्.ए.,दे हरादन
ू
2 केंिीय िवद्यालय-संगठन, दे हरादन
ू -संभाग
अध्ययन-साममी: कक्षा-12, िहं दी(केंििक)
2012-13
संरक्षक
ौी नरे न्ि िसंह राणा, उपायुक्त, केंिीय िवद्यालय-संगठन, दे हरादन
ू -संभाग
सलाहकार
ौी सरदार िसंह चौहान, सहायक आयुक्त, केंिीय िवद्यालय-संगठन, दे हरादन
ू -संभाग
िनदेर् शक
ौी दे वी ूसाद ममगाईं,
ूाचायर्, केंिीय िवद्यालय, बमांक-२ भा.स.िव., हाथीबडकला,
दे हरादन
ू
सह िनदेर् शक
ौीमती भारती दे वी राणा, ूाचायर्, केंिीय िवद्यालय, आइ.एम ्.ए.,दे हरादन
ू
समन्वयक
ौी अशोक कुमार वांणेर्य, उप ूाचायर्, केंिीय िवद्यालय, आइ.एम ्.ए.,दे हरादन
ू
साममी-िवन्यास
डॉ. नीलम सरीन, ःनातकोत्तर िशिक्षका,केंिीय िवद्यालय, आयुध-िनमार्णी, रायपुर, दे हरादन
ू
डॉ. िवजय राम पाण्डे य, ःनातकोत्तर िशक्षक,केंिीय िवद्यालय, अपरकैंप, दे हरादन
ू
डॉ. सुरेन्ि कुमार शमार्,ःनातकोत्तर िशक्षक,केंिीय िवद्यालय, बमांक-२, भा.स.िव.,
हाथीबडकला, दे हरादन
ू
ौीमती रजनी िसंह,ःनातकोत्तर िशिक्षका, केंिीयिवद्यालय, आइ.एम ्.ए.,दे हरादन
ू
ौीमती कमला िनखुपार्,ःनातकोत्तर िशिक्षका, केंिीय िवद्यालय, एफ.आर.आई.,दे हरादन
ू
3 िहन्दी (केिन्िक)- पाठ्यबम कोड सं. 302 कक्षा XII (बारहवीं ) ¾
¾
अपिठत बोध (गद्यांश और काव्यांश-बोध) 15+5=20 रचनात्मक-लेखन एवं जनसंचार माध्यम, अिभव्यिक्त और माध्यम (िूंट माध्यम, सम्पादकीय, िरपोटर् ,
आलेख, फीचर-लेखन) 5+5+5+5+5= 25 ¾
पाठ्यपुःतक – आरोह भाग-२ (काव्यांश‐20 गद्यांश‐20) 40 पूरक-पुःतक िवतान भाग-२ 15 कुल अंक
100
(क) अपिठत बोध 20 ूश्न 1-काव्यांश बोध पर आधािरत पाँच लघूत्तरात्मक ूश्न 1*5= 5 ूश्न 2-गद्यांश बोध पर आधािरत बोध, ूयोग, रचनान्तरण, शीषर्क आिद पर लघूत्तरात्मक ूश्न 15 (ख) रचनात्मक-लेखन एवं जनसंचार माध्यम ूश्न 3- िनबंध (िकसी एक िवषय पर) ूश्न 4- कायार्लय पऽ (िवकल्प सिहत) 25 5 5 ूश्न 5-(अ) िूंट माध्यम, सम्पादकीय, िरपोटर् , आलेख आिद पर पाँच अित लघूत्तरात्मक ूश्न 1*5=5 (आ) आलेख (िकसी एक िवषय पर) 5 ूश्न 6- फीचर लेखन (जीवन-सन्दभोर्ं से जुडी घटनाओं और िःथितयों पर फीचर लेखन िवकल्प सिहत) 5 (ग) आरोह भाग-२ (काव्य भाग और गद्य भाग) (20+20)=40 ूश्न 7- दो काव्यांशों में से िकसी एक पर अथर् महण के ४ ूश्न 8 ूश्न 8- काव्यांश के सौंदयर्-बोध पर दो काव्यांशों में िवकल्प िदया जाएगा तथा िकसी एक काव्यांश के तीनों ूश्नों के उत्तर दे ने होंगे | 6 ूश्न 9- किवताओं की िवषयवःतु से संबंिधत तीन में से दो लघूत्तरात्मक ूश्न (3+3)= 6 ूश्न 10- दो में से िकसी एक गद्यांश पर आधािरत अथर्-महण के चार ूश्न(2+2+2+2) =8 ूश्न 11- पाठों की िवषयवःतु पर आधािरत पाँच में चार बोधात्मक ूश्न (3+3+3+3)=12 पूरक पुःतक िवतान भाग -२ 15 ूश्न 12-पाठों की िवषयवःतु पर आधािरत तीन में से दो बोधात्मक ूश्न (3+3)=6 ूश्न 13-िवचार/संदेश पर आधािरत तीन में से दो लघूत्तरात्मक
ूश्न(2+2)=4 ूश्न 14-िवषयवःतु पर आधािरत दो में से एक िनबंधात्मकूश्न 5 िनधार्िरत पुःतकें (i)
आरोह भाग-२ (एन.सी.ई.आर.टी. द्वारा ूकािशत) (ii) िवतान भाग-२ (एन.सी.ई.आर.टी. द्वारा ूकािशत) (ii)
अिभव्यिक्त और माध्यम (एन.सी.ई.आर.टी. द्वारा ूकािशत) 4 अध्ययन-साममी
िहं दी (केंििक) २०१२-१३
अपिठत:िनधार्िरत अंक: २० (गद्य के िलए १५ तथा पद्य के िलए ५ अंक िनधार्िरत हैं)
अपिठत अंश को हल करने के िलए आवँयक िनदेर् श :
अपिठत अंश में २० अंकों के ूश्न पूछे जाएँगे, जो गद्य और पद्य दो रूपों
में होंगे| ये ूश्न
एक या दो अंकों के होते हैं | उत्तर दे ते समय िनम्न बातों को ध्यान में रख कर उत्तर
दीिजए –
१. िदए गए गद्यांश अथवा पद्यांश कोपूछे गए ूश्नों के साथ ध्यान पूवक
र्
दो बार पिढ़ए |
२. ूश्नों के उत्तर दे ने के िलए सबसे पहले सरलतम ूश्न का उत्तर दीिजए और िमलने पर
उसको रे खांिकत
कर ूश्न संख्या िलख दीिजए, िफर सरलतम से सरलतर को बम से
छाँट कर रे खांिकत कर ूश्न संख्या िलखते जाएँ |
३. उत्तर की भाषा आपकी अपनी भाषा होनी चािहए |
४. गद्यांश में व्याकरण से तथा काव्यांश में सौंदयर्-बोध से संबंिधत
ूश्नों को भी पूछा जाता
है , इसिलए व्याकरण और काव्यांग की सामान्य जानकारी को अद्यतन रखें |
५. उत्तर को अिधक िवःतार न दे कर संक्षेप में िलखें|
६. पूछे गए अंश के कथ्य में िजस तथ्य को बार-बार उठाया गया है , उसी के आधार
पर
शीषर्क िलखें | शीषर्क एक या दो शब्दों का होना चािहए |
१. अपिठत गद्यांश का नमूना- िनधार्िरत अंक: १५
मैं िजस समाज की कल्पना करता हँू , उसमें गृहःथ संन्यासी और संन्यासी गृहःथ होंगे अथार्त
संन्यास और गृहःथ के बीच वह दरी
ू नहीं रहे गी जो परं परा से चलती आ रही है| संन्यासी उत्तम
कोिट का मनुंय होता है , क्योंिक उसमें संचय की वृित्त नहीं होती, लोभ और ःवाथर् नहीं
होता |
यही गुण गृहःथ में भी होना चािहए | संन्यासी भी वही ौेष्ठ है जो समाज के िलए कुछ काम
करे| ज्ञान और कमर् को िभन्न करोगे तो समाज में िवषमता उत्पन्न होगी ही |मुख में किवता
और करघे पर हाथ, यह आदशर् मुझे पसंद था |इसी की िशक्षा मैं दसरों
को भी दे ता हँू और
ू
तुमने सुना है या नहीं की नानक ने एक अमीर लडके के हाथ से पानी पीना अःवीकार कर िदया
था | लोगों ने कहा Ð गुरु जी यह लड़का तो अत्यंत संॅांत कुल का है , इसके हाथ का पानी पीने
में क्या दोष है ?
के
नानक बोले- तलहत्थी में मेहनत के िनशाननहीं हैं | िजसके हाथ में मेहनत
ठे ले पड़े नहीं होते उसके हाथ का पानी पीने में मैं दोष मानता हँू |
नानक ठीक थे | ौेष्ठ
समाज वह है , िजसके सदःय जी खोलकर मेहनत करते हैं और तब भी जरूरत से ज्यादा धन
पर अिधकार जमाने की उनकी इच्छा नहीं होती |
ूश्नों का नमूना(क)Ôगृहःथ संन्यासी और संन्यासी गृहःथ होंगेÕ से लेखक का क्या आशय है ?
(ख) संन्यासी को उत्तम कोिट का मनुंय कहा गया है , क्यों ? १
1 २
(ग) ौेष्ठ समाज के क्या लक्षण बताए गए हैं ? १
(घ) नानक ने अमीर लड़के के हाथ से पानी पीना क्यों अःवीकार िकया ? २
(ङ) Ôमुख में किवता और करघे पर हाथÕ- यह उिक्त िकसके िलए ूयोग की गई है और क्यों
? २
(च) ौेष्ठ संन्यासी के क्या गुण बताए गए हैं ?१
(छ) समाज में िवषमता से
आप क्या समझते हैं और यह
कब उत्पन्न होती है ? २
(ज) संन्यासी शब्द का संिध-िवच्छे द कीिजए | १
(झ) िवषमता शब्द का िवलोम िलख कर उसमें ूयुक्त ूत्यय अलग कीिजए | २
(ञ) गद्यांश का उपयुक्त शीषर्क दीिजए | १
उत्तर –
(क)गृहःथ जन संन्यािसयों की भाँित धन-संमह और मोह से मुक्त रहें तथा संन्यासी जन
गृहःथों की भाँित सामािजक कमोर्ं में सहयोग करें , िनठल्ले न रहें |
(ख)
संन्यासी लोभ, ःवाथर् और संचय से अलग रहता है |
(ग) ौेष्ठ
समाज के सदःय भरपूर पिरौम करते हैं तथा आवँयकता से अिधक धन पर
अपना अिधकार नहीं जमाते |
(घ) अमीर लड़के
के हाथों में मेहनतकश के हाथों की तरह मेहनत करने के िनशान
नहीं थे
और नानक मेहनत करना अिनवायर् मानते थे |
(ङ) मुख में किवता और करघे में हाथÕ कबीर के िलए कहा गया है | क्योंिक उसके घर में
जुलाहे का कायर् होता था और किवता करना उनका ःवभाव था |
(च) ौेष्ठ संन्यासी समाज के िलए भी कायर् करता है |
(छ)समाज में जब ज्ञान और कमर् को िभन्न
में
मानकर आचरण िकए जाते हैं तब उस समाज
िवषमता मान ली जाती है |ज्ञान और कमर् को अलग करने पर ही समाज में
िवषमता फैलती है |
(ज)सम ् + न्यासी
(झ)
िवषमता Ð समता, ÔताÕ ूत्यय
(ञ) संन्यास-गृहःथ
२. अपिठत काव्यांश का नमूना- िनधार्िरत अंक: ५
तुम भारत, हम भारतीय हैं , तुम माता, हम बेटे,
िकसकी िहम्मत है िक तुम्हें दष्टता
ु -दृिष्ट से दे खे |
ओ माता, तुम एक अरब से अिधक भुजाओं वाली,
सबकी रक्षा में तुम
सक्षम, हो अदम्य बलशाली |
2 भाषा, वेश, ूदे श िभन्न हैं , िफर भी भाई-भाई,
भारत की साझी संःकृ ित में पलते भारतवासी |
सुिदनों में
हम एक साथ हँ सते, गाते, सोते हैं ,
दिदर्
ु न में भी साथ-साथ जागते, पौरुष धोते हैं |
तुम हो शःय-ँयामला, खेतों में तुम लहराती हो,
ूकृ ित ूाणमयी, साम-गानमयी, तुम न िकसे भाती हो |
तुम न अगर होती तो धरती वसुधा क्यों कहलाती ?
गंगा कहाँ बहा करती, गीता क्यों गाई जाती ?
ूश्न नमूना:
(क)
साझी संःकृ ित का क्या भाव है ? १
(ख)
भारत को अदम्य
(ग)
सुख-दःख
के िदनों में भारतीयों का परःपर सहयोग कैसा होता है ? १
ु
(घ)
बलशाली
क्यों कहा गया है ? १
साम-गानमयी का क्या तात्पयर् है ?
(ङ)
१
Ôओ माता, तुम एक अरब से अिधक भुजाओं वालीÕ में
कौन-सा अलंकार
है ? १
उत्तर –
(क)भाषा, वेश, ूदे श िभन्न होते हए
एक हैं |
ु
ु भी सभी के सुख-दःख
(ख)
भारत की एक अरब से अिधक जनता अपनी मजबूत भुजाओं से सबकी सुरक्षा करने
में समथर् है |
(ग) भारतीयों
का व्यवहार आपसी सहयोग और अपनेपन से भरा है सब संग-संग हँ सते-गाते
हैं और संग-संग किठनाइयों से जूझते हैं |
(घ) सुमधुर संगीत से युक्त |
(ङ) रूपक |
३. िनबंध-लेखन - िनधार्िरत अंक: ५
िनबंध-लेखन करते समय छाऽोंको िनम्न बातें ध्यान में रखनी चािहए –
• िदए गए िवषय की एक रूपरे खा बना लें |
• रूपरे खा-लेखन के समय पूवार्पर संबध
ं के िनयम का िनवार्ह िकया जाए | पूवार्पर
संबंध के िनवार्ह का अथर् है िक ऊपर की बात उसके ठीक नीचे की बात से जुड़ी
होनी चािहए, िजससे िवषय का बम बना रहे |
• पुनरावृित्त दोष न आए |
• भाषा सरल, सहज और बोधगम्य हो |
• िनबंध का ूारम्भ िकसी कहावत, उिक्त, सूिक्त आिद से िकया जाए |
3 • िवषय को ूामािणक बनाने के उद्दे ँय से िहन्दी, संःकृ त, अंमेजी,उदर् ू की
सूिक्तयाँएवं उद्धरण भी बीच-बीच में दे ते रहना चािहए |
• भूिमका/ूःतावना में िवषय का सामान्य पिरचय तथा उपसंहार में िवषय का
िनंकषर् होना चािहए |
िनबंध हे तु नमूना रूपरे खा :
िवज्ञान ; वरदान या अिभशाप
१. भूिमका/ ूःतावना
२. िवज्ञान का अथर्
३. िवज्ञान वरदान है –
•
िशक्षा के क्षेऽ में
•
िचिकत्सा के क्षेऽ में
•
मनोरं जन के क्षेऽ में
•
कृ िष के क्षेऽ में
•
यातायात के क्षेऽ में
४. िवज्ञान अिभशाप
है –
•
िशक्षा के क्षेऽ में
•
िचिकत्सा के क्षेऽ में
•
मनोरं जन के क्षेऽ में
•
कृ िष के क्षेऽ में
•
यातायात के क्षेऽ में
५. िवज्ञान के ूित हमारे उत्तरदाियत्व
६. उपसंहार
िवशेष: उक्त रूपरे खा को आवँयकता के अनुरूप िविभन्न क्षेऽों को जोड़कर बढ़ाया जा
सकता है |
अभ्यास हे तु िनबंध१. महानगरीय जीवन: अिभशाप या वरदान
२. आधुिनक िशक्षा-पद्धित: गुण व दोष
३. िवज्ञान व कला
४. बदलते जीवन मूल्य
५. नई सदी: नया समाज
६. कामकाजी मिहलाओं की समःयाएँ/ देश की ूगित में मिहलाओं का योगदान
4 ७.राष्टर्-िनमार्ण में युवा पीढ़ी का योगदान
८. इं टरनेट की दिनयाँ
ु
९. पराधीन सपनेहुँ सुख नाहीं
१०. लोकतंऽ में मीिडया की भूिमका
१ १. ूगित के पथ पर भारत
१२. जन आंदोलन और सरकार
१३. ॅष्टाचार: समःया और समाधान
१४. महँ गाई की मार
१५. खेल-कूद में उतरता भारत/ ऑलंिपक २०१२
४. पऽ-लेखन-िनधार्िरत अंक: ५
िवचारों, भावों, संदेशों एवं सूचनाओं के संूेषण के िलए पऽ सहज, सरल तथा पारं पिरक
माध्यम है । पऽ अनेक ूकार के हो सकते हैं , पर ूाय: परीक्षाओं में िशकायती-पऽ, आवेदनपऽ तथा संपादक के नाम पऽ पूछे जाते हैं । इन पऽों को िलखते समय िनम्न बातों का
ध्यान रखा जाना चािहए:
पऽ-लेखन के अंग:१. पता और िदनांक- पऽ के ऊपर बाईं ओर ूेषक का पता व िदनांक िलखा जाता है
(छाऽ पते के िलए परीक्षा-भवन ही िलखें)
२. संबोधन और पता– िजसको पऽ िलखा जारहा है उसको यथानुरूप संबोिधत िकया
जाता है , औपचािरक पऽों में पद-नाम और कायार्लयी पता रहता है |
३. िवषय Ð केवल औपचािरक पऽों में ूयोग करें (पऽ के कथ्य का संिक्षप्त रूप, िजसे पढ़
कर पऽ की साममी का संकेत िमल जाता है )
४. पऽ की साममी – यह पऽ का मूल िवषय है , इसे संक्षेप में सारगिभर्त और िवषय के
ःपष्टीकरण के साथ िलखा जाए |
५. पऽ की समािप्त – इसमें धन्यवाद, आभार सिहत अथवा साभार जैसे शब्द िलख कर
लेखक अपने हःताक्षर और नाम िलखता है |
ध्यान दें , छाऽ पऽ में कहीं अपना अिभज्ञान (नाम-पता) न दें | औपचािरक पऽों में
िवषयानुरूप ही अपनी बात कहें | िद्व-अथर्क और बोिझल शब्दावली से बचें |
६. भाषा शुद्ध, सरल, ःपष्ट, िवषयानुरूप तथा ूभावकारी होनी चािहए।
पऽ का नमूना :
अःपताल के ूबंधन पर संतोष व्यक्त करते हए
ु िचिकत्सा-अधीक्षक को पऽ िलिखए |
परीक्षा-भवन,
5 िदनांक: ----मानाथर्
िचिकत्सा-अधीक्षक,
कोरोनेशन अःपताल,
दे हरादन
ू |
िवषय : अःपताल के ूबंधन पर संतोष व्यक्त करने के संदभर् में मान्यवर,
इस पऽ के माध्यम से मैं आपके िचिकत्सालय के सुूबंधन से ूभािवत हो कर आपको
धन्यवाद दे रहा हँू | गत सप्ताह मेरे िपता जी हृदय-आघात से पीिड़त
होकर आपके यहाँ दािखल
हए
ु थे | आपके िचिकत्सकों और सहयोगी ःटाफ ने िजस तत्परता, कतर्व्यिनष्ठा और ईमानदारी
से उनकी दे खभाल तथा िचिकत्सा की उससे हम सभी पिरवारी जन संतुष्ट हैं | हमारा िवश्वास
बढ़ा है | आपके िचिकत्सालय का अनुशासन ूशंसनीय है |
आशा है जब हम पुनपर्रीक्षण हे तु आएँगे, तब भी वैसी ही सुव्यवःथा िमलेगी |
साभार !
भवदीय
क ख ग
अभ्यासाथर् ूश्न:१. िकसी दै िनक समाचार-पऽ के संपादक के नाम पऽ िलिखए िजसमें वृक्षों की कटाई को
रोकने के िलए सरकार का ध्यान आकिषर्त िकया गया हो।
२. िहं सा-ूधान िफ़ल्मों को दे ख कर बालवगर् पर पड़ने वाले दंूभाव
का वणर्न करते
ु
हए
ु िकसी दै िनक पऽ के संपादक के नाम पऽ िलिखए।
३. अिनयिमत डाक-िवतरण की िशकायत करते हए
ु पोःटमाःटर को पऽ िलिखए।
४. िलिपक पद हे तु िवद्यालय के ूाचायर् को आवेदन-पऽ िलिखए।
५. अपने क्षेऽ में िबजली-संकट से उत्पन्न किठनाइयों का वणर्न करते हए
ु अिधशासी
अिभयन्ता िवद्युत-बोडर् को पऽ िलिखए।
७. दै िनक पऽ के संपादक को पऽ िलिखए, िजसमें िहं दी भाषा की िद्व-रूपता को
समाप्त करने के सुझाव िदए गए हों |
५. (क) अिभव्यिक्तऔरमाध्यम: (एक-एक अंक के ५ ूश्न पूछे जाएँगे तथा उत्तर संक्षेप में िदए
जाएँगे )
उत्तम अंक ूाप्त करने के िलए ध्यान दे ने योग्य बातें१. अिभव्यिक्त और माध्यम
से संबंिधत ूश्न िवशेष रूप से तथ्यपरक होते हैं अत: उत्तर
6 िलखते
समय सही तथ्यों को ध्यान में रखें।
२. उत्तर िबंदवार
िलखें, मुख्य िबंद ु को सबसे पहले िलख दें ।
ु
३. शुद्ध वतर्नी का ध्यान रखें ।
४. लेख साफ़-सुथरा एवम पठनीय हो ।
५. उत्तर में अनावँयक बातें न िलखें ।
६. िनबंधात्मक
ूश्नों में बमबद्धता तथा िवषय के पूवार्पर संबंध
का ध्यान रखें, तथ्यों तथा
िवचारों की पुनरावृित्त न करें ।
जनसंचारमाध्यम
१. संचार िकसे कहते हैं ?
ÔसंचारÕ शब्द चर ् धातु के साथ सम ् उपसगर् जोड़ने से बना है - इसका अथर् है चलना या
एक ःथान से दसरे
ःथान तक पहँु चना |संचार संदेशों का आदान-ूदान है |
ू
सूचनाओं, िवचारों और भावनाओं का िलिखत, मौिखक या दृँय-ौव्य माध्यमों के जिरये
२.
सफ़लता पूवक
र् आदान-ूदान करना या एक जगह से दसरी
जगह पहँु चाना संचार है ।
ू
संचार अनुभवों की साझेदारी है - िकसने कहा है ?
ूिसद्ध संचार शास्तर्ी िवल्बर ौेम
ने |
३. संचार माध्यम से आप क्या समझते हैं ?
संचार-ूिबया को संपन्न करने में सहयोगी तरीके तथा उपकरण संचार के माध्यम
कहलाते हैं ।
४. संचार के मूल तत्त्व िलिखए |
•
संचारक या ॐोत
•
एन्कोिडं ग (कूटीकरण )
•
संदेश ( िजसे संचारक ूाप्तकतार् तक पहँु चाना चाहता है)
•
माध्यम (संदेश को ूाप्तकतार् तक पहँु चाने वाला माध्यम होता है जैसे- ध्विन-तरं गें,
वायु-तरं गें, टे लीफोन, समाचारपऽ, रे िडयो, टी वी आिद)
•
ूाप्तकत्तार् (डीकोिडं ग कर संदेश को ूाप्त करने वाला)
•
फीडबैक (संचार ूिबया में ूाप्तकत्तार् की ूितिबया)
•
शोर (संचार ूिबया में आने वाली बाधा)
५. संचारकेूमुखूकारोंकाउल्लेखकीिजए ?
•
सांकेितकसंचार
•
मौिखकसंचार
•
अमौिखक संचार
•
अंत:वैयिक्तकसंचार
•
अंतरवैयिक्तकसंचार
•
समूहसंचार
7 •
जनसंचार
६. जनसंचारसेआपक्यासमझतेहैं ?
ूत्यक्ष संवाद के बजाय िकसी तकनीकी या यांिऽक माध्यम के द्वारा समाज के
एकिवशाल वगर्से संवाद कायम करना जनसंचार कहलाता है ।
७. जनसंचार के ूमुख माध्यमों का उल्लेख कीिजए |
अखबार, रे िडयो, टीवी, इं टरनेट, िसनेमा आिद.
८. जनसंचार की ूमुखिवशेषताएँ िलिखए |
•
इसमें फ़ीडबैक तुरंत ूाप्त नहीं होता।
•
इसके संदेशों की ूकृ ित सावर्जिनक होती है ।
•
संचारक और ूाप्तकत्तार् के बीच कोई सीधा संबंध नहीं होता।
•
जनसंचार के िलए एक औपचािरक संगठन की आवँयकता होती है ।
•
इसमेंढेर सारे द्वारपाल काम करते हैं ।
९. जनसंचार के ूमुख कायर् कौन-कौनसेहैं ?
•
सूचना दे ना
•
िशिक्षत करना
•
मनोरं जन करना
•
िनगरानी करना
•
एजेंडा तय करना
•
िवचार-िवमशर् के िलए
मंच उपलब्ध कराना
१०. लाइव से क्या अिभूाय है ?
िकसी घटना का घटना-ःथल से सीधा ूसारण लाइव कहलाता है |
१ १. भारत का पहला समाचार वाचक िकसे माना जाता है ?
दे विषर् नारद
१२. जन संचार का सबसे पहला महत्त्वपूणर् तथा सवार्िधक िवःतृत माध्यम कौन सा था ?
समाचार-पऽ और पिऽका
१३. िूंट मीिडया के ूमुख तीन पहलू कौन-कौन से
•
समाचारों को संकिलत करना
•
संपादन करना
•
मुिण तथा ूसारण
हैं ?
१४. समाचारों को संकिलत करने का कायर् कौन करता है ?
संवाददाता
१५. भारत में पऽकािरता की शुरुआत कब और िकससे हई
ु ?
भारत में पऽकािरता की शुरुआत सन १७८० में जेम्स आगःट िहकी के बंगाल गजट से
हई
ु जो कलकत्ता से िनकला था |
8 १६. िहं दी का पहला साप्तािहक पऽ िकसे माना जाता है ?
िहं दी का पहला साप्तािहक पऽ Ôउदं त मातर्ंडÕ को माना जाता है जो कलकत्ता से पंिडत जुगल
िकशोर शुक्ल के संपादन में िनकला था |
१७. आजादी से पूवर् कौन-कौन ूमुख पऽकार हए
ु ?
महात्मा गांधी , लोकमान्य ितलक, मदन मोहन मालवीय, गणेश शंकर िवद्याथीर् ,
माखनलाल चतुवद
ेर् ी, महावीर ूसाद िद्ववेदी , ूताप नारायण िमौ, बाल मुकुंद गुप्त आिद
हए
ु |
१८. आजादी से पूवर् के ूमुख समाचार-पऽों और पिऽकाओं के नाम िलिखए |
केसरी, िहन्दःतान
, सरःवती, हं स, कमर्वीर, आज, ूताप, ूदीप, िवशाल भारत आिद |
ु
१९. आजादी के बाद की ूमुख पऽ-पिऽकाओं तथा पऽकारों के नाम िलखए |
ूमुख पऽ ---- नव भारत टाइम्स, जनसत्ता, नई दिनया
, िहन्दःतान
, अमर उजाला,
ु
ु
दै िनक भाःकर, दै िनक जागरण आिद |
ु , आउट
ूमुख पिऽकाएँ Ð धमर्यग
ु , साप्तािहक िहन्दःतान
, िदनमान , रिववार , इं िडया टडे
ु
लुक आिद |
ूमुख पऽकार- अज्ञेय, रघुवीर सहाय, धमर्वीर भारती, मनोहरँयाम जोशी, राजेन्ि माथुर,
ूभाष जोशी आिद ।
अन्य महत्त्वपूणर् ूश्न:
१. जनसंचार और समूह संचार का अंतर ःपष्ट कीिजए ?
२. कूटवाचन से आप क्या समझते हैं ?
३. कूटीकरण िकसे कहते हैं ?
४. संचारक की भूिमका पर ूकाश डािलए ।
५. फीडबैक से आप क्या समझते हैं ?
६. शोर से क्या तात्पयर् है ?
७. औपचािरक संगठन से आप क्या समझते हैं ?
८. सनसनीखेज समाचारों से सम्बंिधत पऽकािरता को क्या कहते हैं ?
९. कोई घटना समाचार कैसे बनती है ?
१०. संपादकीय पृष्ठ से आप क्या समझते हैं ?
१ १. मीिडया की भाषा में द्वारपाल िकसे कहते हैं ?
पऽकािरता के िविवध आयाम
१. पऽकािरताक्याहै ?
ऐसी सूचनाओं का संकलन एवं संपादन कर आम पाठकों तक पहँु चाना, िजनमें अिधक से
अिधक लोगों की रुिच हो तथा जो अिधक से अिधक लोगों को ूभािवत करती हों,
पऽकािरताकहलाता है ।(दे श-िवदे श में घटने वाली घटनाओं की सूचनाओं को संकिलत एवं
संपािदत कर समाचार के रूप में पाठकों तक पहँु चाने की िबया/िवधा को पऽकािरता कहते
9 हैं)
२. पऽकारीय लेखन तथा सािहित्यक सृजनात्मक लेखन में क्या अंतर है ?
पऽकारीय
लेखन का ूमुख उद्दे ँय सूचना ूदान करना होता है , इसमें तथ्यों की ूधानता
होती है , जबिक सािहित्यक सृजनात्मक लेखन भाव, कल्पना एवं सौंदयर्-ूधान होता है ।
३. पऽकािरता के ूमुख आयाम कौन-कौन से हैं ?
संपादकीय, फ़ोटो पऽकािरता, काटर्ू न कोना , रे खांकन और काटोर्माफ़ |
४. समाचारिकसेकहतेहैं ?
समाचार िकसी भी ऐसी ताजा घटना, िवचार या समःया की िरपोटर् है ,िजसमें अिधक से
अिधक लोगों की रुिच हो और िजसका अिधक से अिधक लोगों पर ूभाव पड़ता हो ।
५. समाचारके तत्त्वों को िलिखए |
पऽकािरता की दृिष्ट से िकसी भी घटना, समःया व िवचार को समाचार का रूप धारण
करने के िलए उसमें िनम्न तत्त्वों में से अिधकांश या सभी का होना आवँयक होता है नवीनता, िनकटता, ूभाव, जनरुिच, संघषर्, महत्त्वपूणर् लोग, उपयोगी जानकािरयाँ,
अनोखापन आिद ।
६. डे डलाइनसेआपक्यासमझतेहैं ?
समाचार माध्यमों के िलए समाचारों को कवर करने के िलए िनधार्िरत समय-सीमा
कोडे डलाइनकहते हैं ।
७. संपादन से क्या अिभूाय है ?
ूकाशन के िलए ूाप्त समाचार-साममी से उसकी अशुिद्धयों को दरू करके पठनीय
तथा
ूकाशन योग्य बनाना संपादन कहलाता है ।
८. संपादकीयक्याहै ?
संपादक द्वारा िकसी ूमुख घटना या समःया पर िलखे गए िवचारात्मक लेख को, िजसे
संबंिधत समाचारपऽ की राय भी कहा जाता है , संपादकीय कहते हैं ।संपादकीय िकसी एक
व्यिक्त का िवचार या राय न होकर समम पऽ-समूह की राय होता है , इसिलए संपादकीय
में संपादक अथवा लेखक का नाम नहीं िलखा जाता ।
९. पऽकािरता के ूमुखूकारिलिखए |
•
खोजी पऽकािरता
•
िवशेषीकृ त पऽकािरता
•
वॉचडॉग पऽकािरता
•
एडवोकेसी पऽकािरता-
•
पीतपऽकािरता
•
पेज ाी पऽकािरता
१०. खोजी पऽकािरता क्याहै ?
िजसमेंआम तौर पर सावर्जिनक महत्त्व के मामलों,जैसे-ॅष्टाचार, अिनयिमतताओं
10 और गड़बिड़यों की गहराई से छानबीन कर सामने लाने की कोिशश की जाती है ।
िःटं ग ऑपरे शन खोजी पऽकािरता का ही एक नया रूप है ।
१ १. वॉचडॉगपऽकािरता से आप क्या समझते हैं ?
लोकतंऽ में पऽकािरता और समाचार मीिडया का मुख्य उत्तरदाियत्व सरकार के
कामकाज पर िनगाह रखना है और कोई गड़बड़ी होने पर उसका परदाफ़ाश करना
होता है , परं परागत रूप से इसे वॉचडॉग पऽकािरता कहते हैं ।
१२. एडवोकेसी पऽकािरता िकसेकहतेहैं ?
इसे पक्षधर पऽकािरता भी कहते हैं । िकसी खास मुद्दे या िवचारधारा के पक्ष में
जनमत बनाने केिलए लगातार अिभयान चलाने वाली पऽकािरता को एडवोकेसी
पऽकािरता कहते हैं ।
१३. पीतपऽकािरतासेआपक्यासमझतेहैं ?
पाठकों को लुभाने के िलए झूठी अफ़वाहों, आरोपों-ूत्यारोपों, ूेमसंबंधों आिद से
संबंिधत सनसनीखेज
समाचारों से संबंिधत पऽकािरता को पीतपऽकािरता कहते
हैं ।
१४. पेज ाी पऽकािरता िकसेकहतेहैं ?
ऐसी पऽकािरता िजसमें फ़ैशन, अमीरों की पािटर् यों , महिफ़लों और जानेमाने लोगों
के िनजी जीवन
के बारे में बताया जाता है ।
१५. पऽकािरता के िवकास में कौन-सा मूल भाव सिबय रहता है ?
िजज्ञासा का
१६. िवशेषीकृ त पऽकािरता क्या है ?
िकसी िवशेष क्षेऽ की िवशेष जानकारी दे ते हए
ु उसका िवश्लेषण करना िवशेषीकृ त
पऽकािरता है |
१७. वैकिल्पक पऽकािरता िकसे कहते हैं ?
मुख्य धारा के मीिडया के िवपरीत जो मीिडया ःथािपत व्यवःथा के िवकल्प को
सामने लाकर उसके
अनुकूल सोच को अिभव्यक्त करता है उसे वैकिल्पक
पऽकािरता कहा जाता है ।आम तौर पर इस तरह के मीिडया को सरकार और
बड़ीपूँजी का समथर्न ूाप्त नहीं होता और न ही उसे बड़ी कंपिनयों के िवज्ञापन
िमलते हैं ।
१८. िवशेषीकृ त पऽकािरता के ूमुख क्षेऽों का उल्लेख कीिजए |
•
संसदीय पऽकािरता
•
न्यायालय पऽकािरता
•
आिथर्क पऽकािरता
•
खेल पऽकािरता
•
िवज्ञान और िवकास पऽकािरता
11 •
अपराध पऽकािरता
•
फैशन और िफल्म पऽकािरता
अन्य महत्त्वपूणर् ूश्न :
१. पऽकािरता के िवकास में कौन-सा मूल भाव सिबय रहता है ?
२. कोई
घटना समाचार कैसे बनती है ?
३. सूचनाओं का संकलन, संपादन कर पाठकों तक पहँु चाने की िबया को क्या कहते हैं ?
४. सम्पादकीय में सम्पादक का नाम क्यों नहीं िलखा जाता ?
५. िनम्न के बारे में िलिखए –
(क) डे ड लाइन
(ख)फ्लैश/ॄेिकंग न्यूज
(ग) गाइड लाइन
(घ) लीड
12 िविभन्न माध्यमों के िलए लेखन
िूंट माध्यम (मुिित माध्यम)१. िूंट मीिडया से क्या आशय है ?
छपाई वाले संचार माध्यम को िूंट मीिडया कहते हैं | इसे मुिण-माध्यम भी कहा
जाता है | समाचार-पऽ ,पिऽकाएँ, पुःतकें आिद इसके ूमुख रूप हैं |
२. जनसंचार के आधुिनक माध्यमों में सबसे पुराना माध्यम कौन-सा है ?
जनसंचार के आधुिनक माध्यमों में सबसे पुराना माध्यम िूंट माध्यम है |
३. आधुिनक छापाखाने का आिवंकार िकसने िकया ?
आधुिनक छापाखाने का आिवंकार जमर्नी के गुटेनबगर् ने िकया।
४.
भारत में पहला छापाखाना कब और कहाँ पर खुला था ?
भारत में पहला छापाखाना सन १५५६ में गोवा में खुला, इसे ईसाई िमशनिरयों ने
धमर्-ूचार की पुःतकें छापने के िलए खोला था |
५. जनसंचार के मुिित माध्यम कौन-कौन से हैं ?
मुिित माध्यमों के अन्तगर्त अखबार, पिऽकाएँ, पुःतकें आिद आती हैं ।
६. मुिित माध्यम की िवशेषताएँ िलिखए |
• छपे हए
ु शब्दों में ःथाियत्व होता है , इन्हें सुिवधानुसार िकसी भी ूकार से पढा◌़
जा सकता है ।
• यह
माध्यम िलिखत भाषा का िवःतार है ।
• यह िचंतन, िवचार- िवश्लेषण का माध्यम है ।
७. मुिित माध्यम की सीमाएँ (दोष) िलिखए |
• िनरक्षरों के िलए मुिित माध्यम िकसी काम के नहीं होते।
• ये तुरंत घटी घटनाओं को संचािलत नहीं कर सकते।
• इसमें ःपेस तथा शब्द सीमा का ध्यान रखना पड़ता है ।
• इसमें एक बार समाचार छप जाने के बाद अशुिद्ध-सुधार नहीं िकया जा सकता।
८. मुिित माध्यमों के लेखन के िलए िलखते समय िकन-िकन बातों का
जाना चािहए |
• भाषागत शुद्धता का ध्यान रखा जाना चािहए।
• ूचिलत भाषा का ूयोग िकया जाए।
• समय, शब्द व ःथान की सीमा का ध्यान रखा जाना चािहए।
• लेखन में तारतम्यता एवं सहज ूवाह होना चािहए।
13 ध्यान रखा
रे िडयो (आकाशवाणी)
१. इलैक्शािनक माध्यम से क्या तात्पयर् है ?
िजस जन संचार में इलैक्शािनक उपकरणों का सहारा िलया जाता है इलैक्शािनक माध्यम
कहते हैं । रे िडयो, दरदशर्
न , इं टरनेट ूमुख इलैक्शािनक माध्यम हैं ।
ू
२. आल इं िडया रे िडयो की िविधवत ःथापना कब हई
ु ?
सन १९३६ में
३. एफ़.एम. रे िडयो की शुरुआत कब से हई
ु ?
एफ़.एम. (ि६क्वेंसी माड्युलेशन) रे िडयो की शुरूआत सन १९९३ से हई
ु ।
४. रे िडयो िकस ूकार का माध्यम है ?
रे िडयो एक इलैक्शोिनक ौव्य माध्यम है । इसमें शब्द एवं आवाज का महत्त्व
होता है ।
यह एक एक रे खीय माध्यम है ।
५. रे िडयो समाचार िकस शैली पर आधािरत होते हैं ?
रे िडयो समाचार की संरचना उल्टािपरािमड शैली पर आधािरत होती है ।
६. उल्टा िपरािमड शैली क्या है ? यह िकतने भागों में बँटी होती है ?
िजसमें तथ्यों को महत्त्व के बम से
ूःतुत िकया जाता है , सवर्ूथम सबसे ज्यादा
महत्त्वपूणर् तथ्य को तथा उसके उपरांत महत्त्व की दृिष्ट से घटते
जाता है उसे उल्टा िपरािमड शैली कहते हैं ।
बम में तथ्यों
को रखा
उल्टािपरािमड शैली में समाचार को तीन
भागों में बाँटा जाता है -इं शो, बाँडी और समापन।
७. रे िडयो समाचार-लेखन के िलए िकन-िकन बुिनयादी बातों पर ध्यान िदया जाना चािहए?
•
समाचार वाचन के िलए तैयार की गई कापी साफ़-सुथरी ओ टाइप्ड कॉपी
•
कॉपी को िशपल ःपेस में टाइप िकया जाना चािहए।
•
पयार्प्त हािशया छोडा◌़ जाना चािहए।
•
अंकों को िलखने में सावधानी रखनी चािहए।
•
संिक्षप्ताक्षरों के ूयोग से बचा जाना चािहए।
हो।
टे लीिवजन(दरदशर्
न) :
ू
१. दरदशर्
न जन संचार का िकस ूकार का माध्यम है ?
ू
दरदशर्
न जनसंचार का सबसे लोकिूय
ू
दृँयों का भी
व सशक्त माध्यम है । इसमें ध्विनयों के साथ-साथ
समावेश होता है । इसके िलए
समाचार
िलखते समय इस बात का ध्यान
रखा जाता है िक शब्द व पदेर् पर िदखने वाले दृँय में समानता हो।
२. भारत में टे लीिवजन का आरं भ
और िवकास िकस ूकार हआ
?
ु
भारत में टे लीिवजन का ूारं भ १५ िसतंबर १९५९ को हआ
। यूनेःको की एक शैिक्षक
ु
पिरयोजना के अन्तगर्त िदल्ली के आसपास के एक गाँव में दो टी.वी. सैट लगाए गए,
14 िजन्हें २०० लोगों ने दे खा । १९६५ के बाद िविधवत टीवी
दरदशर्
न नामक िनकाय की ःथापना हई।
ू
ु
सेवा आरं भ हई
ु । १९७६ में
३. टी०वी० खबरों के िविभन्न चरणों को िलिखए ।
दरदशर्
न मे कोई भी सूचना िनम्न चरणों या सोपानों को पार कर दशर्कों तक पहँु चती
ू
है ।
(१) झलैश या ॄेिकंग न्यूज (समाचार को कम-से-कम शब्दों में दशर्कों तक तत्काल
पहँु चाना)
(२)साई एंकर (एंकर द्वारा शब्दों
में खबर के िवषय में बताया जाता है)
(३) फ़ोन इन (एंकर िरपोटर् र से फ़ोन पर
बात कर दशर्कों तक सूचनाएँ पहँु चाता है )
(४) एंकर-िवजुअल(समाचार के साथ-साथ संबंिधत
दृँयों को िदखाया जाना)
(५) एंकर-बाइट(एंकर का ूत्यक्षदशीर् या संबंिधत व्यिक्त के कथन या बातचीत
द्वारा ूामािणक खबर ूःतुत करना)
(६) लाइव(घटनाःथल से खबर का सीधा ूसारण)
(७) एंकर-पैकेज (इसमें एंकर द्वारा ूःतुत सूचनाएँ; संबिं धत घटना के दृँय, बाइट,
मािफ़क्स आिद द्वारा व्यविःथत ढं ग से िदखाई जाती हैं)
इं टरनेट
१. इं टर नेट क्या है ? इसके गुण-दोषों पर ूकाश डािलए ।
इं टरनेट िवश्वव्यापी अंतजार्ल है , यह जनसंचार
का सबसे नवीन व लोकिूय
माध्यम है । इसमें जनसंचार के सभी माध्यमों के गुण समािहत हैं । यह जहाँ
सूचना, मनोरं जन, ज्ञान और व्यिक्तगत एवं सावर्जिनक संवादों के आदान-ूदान के
िलए
ौेष्ठ माध्यम है , वहीं अश्लीलता, दंू
ु चारव गंदगी फ़ैलाने का भी जिरया है ।
२. इं टरनेट पऽकािरताक्या है ?
इं टरनेट(िवश्व्व्यापी अंतजार्ल) पर समाचारों का ूकाशन या आदान-ूदान इं टरनेट
पऽकािरता कहलाता है । इं टरनेट पऽकािरता दो रूपों में होती है । ूथम- समाचार
संूेषण के िलए नेट का ूयोग करना । दस
ू रा- िरपोटर् र अपने समाचार को ई-मेल
द्वारा
अन्यऽ
भेजने
व
समाचार
को
संकिलत
करने
सत्यता,िवश्वसनीयता िसद्ध करने के िलए करता है ।
३. इं टरनेट पऽकािरता को और िकन-िकन नामों से जाना जाता है ?
ऑनलाइन पऽकािरता, साइबरपऽकािरता,वेब पऽकािरता आिद नामों से ।
४.
िवश्व-ःतर पर इं टरनेट पऽकािरता का िवकास िकन-िकन चरणों में हआ
?
ु
िवश्व-ःतर पर इं टरनेट पऽकािरता का िवकास िनम्निलिखत चरणों में हआ
ु • ूथम चरण------- १९८२ से १९९२
• िद्वतीय चरण------- १९९३ से २००१
15 तथा
उसकी
• तृतीय चरण------- २००२ से अब तक
५. भारत में इं टरनेट पऽकािरता का ूारम्भ कब से हआ
?
ु
पहला चरण १९९३ से तथा दसरा
चरण २००३ से शुरू माना जाता है । भारत में
ू
सच्चे
अथोर्ं
में
वेब
पऽकािरता
करने
वाली
साइटें
’रीिडफ़
डॉट
कॉम’,
इं िडयाइं फ़ोलाइन’व’सीफ़ी’हैं । रीिडफ़ को भारत की पहली साइट कहा जाता है ।
६. वेब साइट पर िवशुद्ध पऽकािरता शुरू करने का ौेय
िकसको जाता है ?
’तहलका डॉट्कॉम’
७. भारत में सच्चे अथोर्ं में वेब पऽकािरता करने वाली साइटों के नाम िलिखए |
’रीिडफ़ डॉट कॉम’,इं िडयाइं फ़ोलाइन’व’सीफ़ी’
८. भारत में कौन-कौन से समाचार-पऽ इं टरनेट पर उपलब्ध हैं ?
टाइम्स आफ़ इं िडया , िहं दःता
ु न टाइम्स, इं िडयन एक्सूैस , िहं द,ू िशब्यून आिद ।
९. भारत की कौन-सी नेट-साइट भुगतान दे कर दे खी जा सकती है ?
ु Õ
Õइं िडया टडे
१०. भारत की पहली साइट कौन-सी है , जो इं टरनेट पर पऽकािरता कर रही है ?
रीिडफ़
१ १. िसफ़र् नेट पर उपलब्ध अखबार का नाम िलिखए।
ूभा साक्षीÕ नाम का अखबार
िूंट रूप में न होकर िसफ़र् नेट पर उपलब्ध है ।
१२. पऽकािरता के िलहाज से िहं दी की सवर्ौष्ठ
े साइट कौन-सी है ?
पऽकािरता के िलहाज से िहन्दी की सवर्ौष्ठ
े साइट बीबीसी की है , जो इं टरनेट के
मानदं डों के अनुसार चल रही है ।
१३. िहं दी वेब जगत में कौन-कौनसी सािहित्यक पिऽकाएँ चल रही हैं ?
िहं दी वेब जगत में ’अनुभिू त’, अिभव्यिक्त, िहं दी नेःट, सराय आिद सािहित्यक पिऽकाएँ
चल रही हैं ।
१४. िहं दी वेब जगत की सबसे बड़ी समःया क्या है ?
िहन्दी वेब जगत की सबसे बडी◌़ समःया मानक की-बोडर् तथा फ़ोंट
डायनिमक फ़ोंट
के अभाव के कारण िहन्दी की ज्यादातर साइटें खुलती ही नहीं हैं ।
अभ्यासाथर् ूश्न:
१. भारत में पहला छापाखान िकस उद्दे ँय से खोला गया ?
२. गुटेनबगर् को िकस क्षेऽ में योगदान के िलए याद िकया जाता है ?
३. रे िडयो समाचर िकस शैली में िलखे जाते हैं ?
४. रे िडयो तथा टे लीिवजन माध्यमों में मुख्य अंतर क्या है ?
५. एंकर बाईट क्या है ?
16 की है ।
६. समाचार को संकिलत करने वाला व्यिक्त क्या कहलाता है ?
७. नेट साउं ड िकसे कहते हैं ?
८. ॄेिकंग न्यूज से आप क्या समझते हैं ?
पऽकारीय लेखन के िविभन्न रूप और लेखन ूिबया
१. पऽकारीय लेखन क्या है ?
समाचार माध्यमों मे काम करने वाले
सूचनाएँ पहँु चाने के िलए लेखन के
पऽकार अपने पाठकों
तथा ौोताओं तक
िविभन्न रूपों का इःतेमाल करते हैं , इसे ही
पऽकारीय लेखन कहते हैं । पऽकारीय लेखन का संबंध समसामियक िवषयों, िवचारों व
घटनाओं से है । पऽकार को िलखते समय यह ध्यान रखना चािहए वह सामान्य जनता के
िलए िलख रहा है , इसिलए उसकी
भाषा सरल व रोचक होनी चािहए। वाक्य छोटे व
सहज हों। किठन भाषा का ूयोग नहीं िकया जाना चािहए। भाषा को ूभावी बनाने के
िलए
अनावँयक िवशेषणों,जागर्न्स(अूचिलत शब्दावली) और क्लीशे (िपष्टोिक्त, दोहराव)
का ूयोग नहीं होना चिहए।
२. पऽकारीय लेखन के अंतगर्त क्या-क्या आता है ?
पऽकिरता या पऽकारीय लेखन के अन्तगर्त
फ़ीचर, ःतम्भ तथा काटर्ू न आिद आते हैं
सम्पादकीय, समाचार, आलेख, िरपोटर् ,
३. पऽकारीय लेखन का मुख्य उद्दे ँय क्या होता है ?
पऽकारीय लेखन का ूमुख उद्दे ँय है - सूचना दे ना, िशिक्षत करना तथा मनोरं जन आ
करना आिद होता है |
४. पऽकारीय लेखन के ूकार िलखए |
पऽकारीय लेखन के कईूकार हैं यथा- खोजपरक पऽकािरता’, वॉचडॉग पऽकािरता और
एड्वोकैसी पऽकािरता आिद।
५. पऽकारिकतने
ूकार के होते हैं ?
पऽकार तीन ूकार के होते हैं •
६. समाचार
पूणर् कािलक
•
अंशकािलक (िःशं गर)
•
६ीलांसर या ःवतंऽ पऽकार
िकस शैली में िलखे जाते हैं ?
समाचार उलटा िपरािमड शैली में िलखे जाते हैं , यह समाचार लेखन की सबसे उपयोगी
और लोकिूय शैली है । इस शैली का िवकास अमेिरका में गृह युद्ध
इसमें महत्त्वपूणर् घटना
के दौरान हआ।
ु
का वणर्न पहले ूःतुत िकया जाता है , उसके बाद महत्त्व की
दृिष्ट से घटते बम में घटनाओं को ूःतुत कर समाचार का अंत िकया जाता है । समाचार
में इं शो, बॉडी और समापन के बम में घटनाएँ ूःतुत की जाती हैं ।
७. समाचार के छह
ककार कौन-कौन से हैं ?
17 समाचार िलखते समय मुख्य रूप से छह ूश्नों- क्या, कौन, कहाँ, कब, क्यों और कैसे का
उत्तर दे ने की कोिशश की जाती है । इन्हें समाचार के छह ककार कहा जाता है । ूथम चार
ूश्नों के उत्तर इं शो में तथा अन्य दो के उत्तर समापन से पूवर् बॉडी वाले भाग में िदए जाते
हैं ।
८. फ़ीचर क्या है ?
फ़ीचर एक ूकार का सुव्यविःथत, सृजनात्मक और आत्मिनष्ठ लेखन है ।
९. फ़ीचर लेखन का क्या उद्दे ँय होता है ?
फ़ीचर का उद्दे ँय मुख्य रूप से पाठकों को सूचना दे ना, िशिक्षत करना तथा उनका
मनोरं जन करना होता है ।
१०. फ़ीचर और समचार में क्या अंतर है ?
समाचार में िरपोटर् र को अपने िवचारों को डालने की ःवतंऽता नहीं होती, जबिक फ़ीचर
में लेखक को अपनी राय , दृिष्टकोण और भावनाओं को जािहर करने का अवसर होता है
। समाचार उल्टा िपरािमड शैली में
िलखे जाते हैं , जबिक फ़ीचर लेखन की कोई
सुिनिश्चत शैली नहीं होती । फ़ीचर में समाचारों की तरह शब्दों की सीमा नहीं होती।
आमतौर पर फ़ीचर, समाचार िरपोटर् से बड़े होते हैं । पऽ-पिऽकाओं में ूाय: २५० से २०००
शब्दों तक के फ़ीचर छपते हैं ।
१ १. िवशेष िरपोटर् से आप क्या समझते हैं ?
सामान्य समाचारों से अलग वे िवशेष समाचार जो गहरी छान-बीन, िवश्लेषण और
व्याख्या के आधार पर ूकािशत िकए जाते हैं , िवशेष िरपोटर् कहलाते हैं ।
१२. िवशेष िरपोटर्
के िविभन्न ूकारों को ःपष्ट कीिजए ।
खोजी िरपोटर् : इसमें अनुपल्ब्ध तथ्यों को गहरी छान-बीन
कर सावर्जिनक िकया जाता
है ।
(२)इन्डे प्थ िरपोटर्:
सावर्जािनक रूप से ूाप्त तथ्यों की गहरी छान-बीन कर उसके
महत्त्वपूणर् पक्षों को पाठकों के सामने लाया जाता है ।
(३) िवश्लेषणात्मक िरपोटर् : इसमें िकसी घटना या समःया का िववरण सूआमता के साथ
िवःतार से िदया जाता है । िरपोटर् अिधक िवःतृत होने पर कई िदनों तक िकःतों में
ूकािशत की जाती है ।
(४)िववरणात्मक िरपोटर् : इसमें िकसी घटना या समःया को िवःतार एवं बारीकी के
साथ ूःतुत िकया जाता है ।
१३. िवचारपरक लेखन िकसे कहते हैं ?
िजस लेखन में िवचार एवं िचंतन की ूधानता
होती है , उसे
िवचार परक लेखन कहा
जाता है ।समाचार-पऽों में समाचार एवं फ़ीचर के अितिरक्त संपादकीय, लेख, पऽ, िटप्पणी,
विरष्ठपऽकारों व िवशेषज्ञों के ःतंभ छपते हैं । ये सभी िवचारपरक लेखन के अंतगर्त आते
18 अंतगर्त आते हैं ।
19 १४. संपादकीय से क्या अिभूाय है ?
संपादक द्वारा िकसी ूमुख घटना या समःया पर िलखे गए िवचारात्मक
लेख को,
िजसेसब
ं ंिधत समाचारपऽ की राय भी कहा जाता है , संपादकीय कहते हैं ।
संपादकीय
िकसी एक व्यिक्त का िवचार या राय न होकर समम पऽ-समूह की राय होता है , इसिलए
संपादकीय में संपादक अथवा लेखक का नाम नहीं िलखा जाता ।
१५. ःतंभलेखन से क्या तात्पयर् है ?
यह एक ूकार का िवचारात्मक लेखन है । कुछ महत्त्वपूणर् लेखक अपने खास वैचािरक
रुझान एवं लेखन शैली
के िलए जाने जाते हैं । ऐसे लेखकों की लोकिूयता को दे खकर
समाचरपऽ उन्हें अपने पऽ में िनयिमत ःतंभ-लेखन की िजम्मेदारी ूदान करते हैं । इस
ूकार िकसी समाचार-पऽ में िकसी ऐसे लेखक द्वारा िकया गया िविशष्ट एवं िनयिमत
लेखन जो अपनी िविशष्ट शैली एवं वैचािरक रुझान के कारण समाज में ख्याित-ूाप्त हो,
ःतंभ लेखन कहा जाता है ।
१६. संपादक के नाम पऽ से आप क्या समझते हैं ?
समाचार पऽों में संपादकीय पृष्ठ पर तथा पिऽकाओं की शुरुआत में संपादक के नाम
आए पऽ ूकािशत िकए जाते हैं । यह ूत्येक समाचारपऽ का िनयिमत ःतंभ होता है ।
इसके माध्यम से समाचार-पऽ अपने पाठकों को जनसमःयाओं तथा मुद्दों पर अपने
िवचार एवमराय व्यक्त करने का अवसर ूदान करता है ।
१७. साक्षात्कार/इंटरव्यू से क्या अिभूाय है ?
िकसी पऽकार के द्वारा अपने समाचारपऽ में ूकािशत करने के िलए, िकसी व्यिक्त िवशेष
से उसके िवषय में अथवा िकसी िवषय या मुद्दे पर िकया गया ूश्नोत्तरात्मक संवाद
साक्षात्कार कहलाता है ।
अन्य महत्त्वपूणर् ूश्न:
१. सामान्य लेखन तथा पऽकारीय लेखन में क्या अंतर है ?
२. पऽकारीय लेखन के उद्दे ँय िलिखए।
३. पऽकार िकतने ूकार के होते हैं ?
४. उल्टा िपरािमड शैली का िवकास कब और क्यों हआ
ु ?
५. समाचार के ककारों के नाम िलिखए |
६. बाडी क्या है ?
७. फ़ीचर िकस शैली में िलखा जाता है ?
८. फ़ीचर व समाचार में क्या अंतर है ?
९. िवशेष िरपोटर् से आप क्या समझते हैं ?
१०. िवशेष िरपोटर् के भेद िलिखए।
20 १ १. इन्डे प्थ िरपोटर् िकसे कहते हैं ?
१२. िवचारपरक लेखन क्या है तथा उसके अन्तगर्त िकस ूकार के लेख आते हैं ?
१३. ःवतंऽ पऽकार िकसे कहते है ?
१४. पूणक
र् ािलक पऽकार से क्या अिभूाय है ?
१५. अंशकािलक पऽकार क्या होता है ?
िवशेष लेखन: ःवरूप और ूकार
१. िवशेष लेखन िकसे कहते हैं ?
िवशेष लेखनिकसी खास िवषय पर
िजसमें
सामान्य लेखन से हट कर िकया गया लेखन है;
राजनीितक, आिथर्क, अपराध, खेल, िफ़ल्म,कृ िष, कानून, िवज्ञान और अन्य
िकसी भी महत्त्वपूणर् िवषय से संबंिधत िवःतृत सूचनाएँ ूदान की जाती हैं ।
२. डे ःकक्या है ?
समाचारपऽ, पिऽकाओं, टीवी और रे िडयो चैनलों में अलग-अलग िवषयों पर िवशेष लेखन
के िलए िनधार्िरत ःथल को डे ःक कहते हैं और उस िवशेष डे ःक पर काम करने वाले
पऽकारों का भी अलग समूह होता है । यथा-व्यापार तथा
कारोबार के िलए अलग तथा
खेल की खबरों के िलए अलग डे ःक िनधार्िरत होता है ।
३. बीटसे क्या तात्पयर् है ?
िविभन्न िवषयों से जुड़े समाचारों के िलए संवाददाताओं के बीच काम का िवभाजन आम
तौर पर उनकी िदलचःपी और ज्ञान को ध्यान में रख कर िकया जाता है । मीिडया की
भाषा में इसे बीट कहते हैं ।
४. बीट िरपोिटर्ं ग तथा िवशेषीकृ त िरपोिटर्ं ग में क्या अन्तर है ?
बीट िरपोिटर्ं ग के िलए
संवाददाता में उस क्षेऽ के बारे में
होना पयार्प्त है , साथ ही उसे
जानकारी व िदलचःपी का
आम तौर पर अपनी बीट से जुड़ीसामान्य खबरें ही िलखनी
होती हैं । िकन्तु िवशेषीकृ त िरपोिटर्ं ग
में सामान्य समाचारों से आगे बढ़कर संबंिधत
िवशेष क्षेऽ या िवषय से जुड़ी घटनाओं, समःयाओं और मुद्दों
का बारीकी से िवश्लेषण कर
ूःतुतीकरण िकया जाता है । बीट कवर करने वाले िरपोटर् र को संवाददाता तथा
िवशेषीकृ त िरपोिटर्ं ग करने वाले िरपोटर् र को िवशेष संवाददाता कहा जाता है ।
५. िवशेष लेखन की भाषा-शैली पर ूकाश डािलए |
िवशेष लेखन की भाषा-शैली सामान्य लेखन से अलग होती है । इसमें संवाददाता को
संबंिधत िवषय की तकनीकी शब्दावली का ज्ञान होना आवँयक होता है , साथ ही यह भी
आवँयक होता है िक वह पाठकों को उस शब्दावली से पिरिचत कराए िजससे पाठक
िरपोटर् को समझ सकें। िवशेष लेखन की कोई िनिश्चत शैली नहीं होती ।
६. िवशेष लेखन के क्षेऽ कौन-कौन से हो सकते हैं ?
िवशेष लेखन के अनेक क्षेऽ होते हैं , यथा- अथर्-व्यापार, खेल, िवज्ञान-ूौद्योिगकी, कृ िष,
िवदे श, रक्षा, पयार्वरण िशक्षा, ःवाःथ्य, िफ़ल्म-मनोरं जन, अपराध, कानून व सामािजक
21 मुद्दे आिद ।
अभ्यासाथर् महत्त्वपूणर् ूश्न :
१. िकसी खास िवषय पर िकए गए लेखन को क्या कहते हैं ?
२. िवशेष लेखन के क्षेऽ िलिखए।
४.पऽकारीय भाषा में लेखन
के िलए िनधार्िरत ःथल को क्या कहते है ?
५. बीट से आप क्या समझते हैं ?
६. बीट िरपोिटर्ं ग क्या है ?
७. बीट िरपोिटर्ं ग तथा िवशेषीकृ त िरपोिटर्ं ग में क्या अंतर है ?
८. िवशेष संवाददाता िकसे कहते हैं ?
बोडर् परीक्षा में पूछे गए ूश्न
एवं अन्य महत्त्वपूणर् पृष्टव्य ूश्नों का कोश :
१. िूंट माध्यम िकसे कहते हैं ?
२. जनसंचार के ूचिलत माध्यमों में सबसे पुराना माध्यम क्या है ?
३. िकन्हीं दो मुिित माध्यमों के नाम िलिखए |
४. छापाखाने के आिवंकार का ौेय िकसको जाता है ?
५. िहं दी का पहला समाचार-पऽ कब, कहाँ से िकसके द्वारा ूकािशत िकया गया ?
६. िहं दी में ूकािशत होने वाले दो दै िनक समाचार-पऽों तथा पिऽकाओं के नाम िलिखए |
७. रे िडयो की अपेक्षा टीवी समाचारों की लोकिूयता के दो कारण िलिखए |
८. पऽकारीय लेखन तथा सािहित्यक सृजनात्मक लेखन का अंतर बताइए |
९. पऽकािरता का मूलतत्व क्या है ?
१०. ःतंभलेखन से क्या तात्पयर् है ?
१ १. पीत पऽकािरता िकसे कहते हैं ?
१२. खोजी पऽकािरता का आशय ःपष्ट कीिजए |
१३. समाचार शब्द को पिरभािषत कीिजए |
१४. उल्टा िपरािमड शैली क्या है ?
१५. समाचार लेखन में छह ककारों का क्या महत्त्व है ?
१६. मुिित माध्यमों
की िकन्हीं दो िवशेषताओं का उल्लेख कीिजए |
१७. डे ड लाइन क्या है ?
१८. रे िडयो नाटक से आप क्या समझते हैं ?
१९. रे िडयो समाचार की भाषा की दो िवशेषताएँ िलिखए
२०. एंकर बाईट िकसे कहते हैं ?
22 २१. टे लीिवजन समाचारों में एंकर बाईट क्यों जरूरी है ?
२२. मुिित माध्यम को ःथायी माध्यम क्यों कहा जाता है ?
२३. िकन्हीं दो समाचार चैनलों के नाम िलिखए |
२४. इं टरनेट पऽकािरता के लोकिूय होने के क्या कारण हैं ?
२५. भारत के िकन्हीं चार
समाचार-पऽों के नाम िलिखए जो इं टरनेट पर उपलब्ध हैं ?
२६. पऽकािरता की भाषा में बीट िकसे कहते हैं ?
२७. िवशेष िरपोटर् के दो ूकारों का उल्लेख कीिजए |
२८. िवशेष लेखन के िकन्हीं दो ूकारों का नामोल्लेख कीिजए |
२९. िवशेष िरपोटर् के
लेखन में िकन बातों पर अिधक बल िदया जाता है ?
३०. बीट िरपोटर् र िकसे कहते हैं ?
३१. िरपोटर् लेखन की भाषा की दो िवशेषताएँ िलिखए |
३२. संपादकीय के साथ संपादन-लेखक का नाम क्यों नहीं िदया जाता ?
३३. संपादकीय लेखन क्या होता है ?
अथवा
संपादकीय से क्या तात्पयर् है ?
३४. संपादक के दो ूमुख उत्तरदाियत्वों का उल्लेख कीिजए |
३५. ऑप-एड पृष्ठ िकसे कहते हैं ?
३६. न्यूजपेग क्या है ?
३७. आिडएंस से आप क्या समझते हैं ?
३८. इलेक्शोिनक मीिडया क्या है ?
३९. काटर्ू न कोना क्या है ?
४०. भारत में िनयिमत अपडे ट साइटों के नाम बताइए।
४१. कम्प्यूटर के लोकिूय होने का ूमुख कारण बताइए।
४२. िवज्ञापन िकसे कहते हैं ?
४३. फीडबैक से क्या अिभूाय है ?
४४. जनसंचार से आप क्या समझते हैं ?
४५. समाचार और फीचर में क्या अंतर है ?
23 (ख)आलेख/िरपोटर् – िनधार्िरत अंक:५
आलेख
आलेख-लेखन हे तु महत्त्वपूणर् बातें:
१. िकसी िवषय पर सवार्ंगपूणर् जानकारी जो तथ्यात्मक, िवश्लेषणात्मक अथवा िवचारात्मक हो
आलेख कहलाती है |
२. आलेख का आकार संिक्षप्त होता है |
३. इसमें िवचारों और तथ्यों की ःपष्टता रहती है , ये िवचार बमबद्ध रूप में होने चािहए|
४. िवचार या तथ्य की पुनरावृित्त न हो |
५. आलेख की शैली िववेचन ,िवश्लेषण अथवा िवचार-ूधान हो सकती है |
६. ज्वलंत मुद्दों, समःयाओं , अवसरों, चिरऽ पर आलेख िलखे जा सकते हैं |
७. आलेख गंभीर अध्ययन पर आधािरत ूामािणक रचना होती है |
नमूना आलेख :
शेर का घर िजमकाबेर्ट नेशनल पाकर् –
जंगली जीवों की िविभन्न ूजाितयों को सरं क्षण दे ने तथा उनकी संख्या को बढाने के उद्दे ँय से
िहमालय की तराई से लगे उत्तराखंड के पौड़ी और नैनीताल िजले में भारतीय महाद्वीप के पहले
राष्टर्ीय अभयारण्य की ःथापना
ूिसद्ध अंगरे जी
लेखक िजम काबेर्ट के नाम पर की गई | िजम
काबेर्ट नॅशनल पाकर् नैनीताल से एक सौ पन्िह िकलोमीटर और िदल्ली से २९० िकलोमीटर दरू
है । यह अभयारण्य पाँच सौ
इक्कीस िकलोमीटर क्षेऽ में फैला है | नवम्बर से जून के बीच यहाँ
घूमने-िफरने का सवोर्त्तम समय है |
यह अभयारण्य चार सौ से ग्यारह सौ मीटर की ऊँचाई पर है | िढकाला इस पाकर् का
ूमुख मैदानी ःथल है और कांडा
सबसे ऊँचा ःथान है| जंगल, जानवर, पहाड़ और हरी-भरी
वािदयों के वरदान से िजमकाबेर्ट पाकर् दिनया
के अनूठे पाकोर्ं में है | रायल बंगाल टाइगर और
ु
एिशयाई हाथी पसंदीदा घर है | यह एिशया का सबसे पहला संरिक्षत जंगल है | राम गंगा नदी
इसकी जीवन-धारा है | यहाँ एक सौ दस तरह के पेड़-पौधे, पचास तरह के ःतनधारी जीव,
पच्चीस ूजाितयों के सरीसृप और छह
सौ तरह के रं ग-िवरं गे पक्षी हैं | िहमालयन तेंदआ
ु ,
िहरन, भालू, जंगली कुत्ते, भेिड़ये, बंदर, लंगरू , जंगली भैंसे जैसे जानवरों से यह जंगल आबाद है
| हर वषर् लाखों पयर्टक यहाँ आते हैं | शाल वृक्षों से िघरे
मैदान इसके ूाकृ ितक सौंदयर् में चार चाँद लगा दे ते हैं |
िनम्निलिखत िवषयों पर आलेख िलिखए•
बढ़ती आबादी : दे श की बरबादी
•
सांूदाियकसद्भावना
24 लंबे-लंबे वन-पथ और हरे -भरे घास के
•
कजर् में डू बा िकसान
•
आतंकवाद की समःया
•
डॉक्टर हड़ताल पर, मरीज परे शान
•
वतर्मान परीक्षा-ूणाली
•
बजट और बचत
•
शिक्त, संयम और साहस
•
िरश्वत का रोग
•
सपना सच हो अपना
िरपोटर्/ूितवेदन
िरपोटर्/ूितवेदन का सामान्य अथर्:
सूचनाओं के तथ्यपरक
आदान-ूदान को िरपोटर् या
िरपोिटर्ं ग कहते हैं | ूितवेदन इसका िहं दी रूपांतरण है | िरपोटर् िकसी संःथा, आयोजन या
कायर्बम की तथ्यात्मक जानकारी है | बड़ी-बड़ी कंपिनयां अपने अंशधारकों को वािषर्क/
अद्धर् वािषर्क ूगित िरपोटर् भेजा करती हैं|
िरपोटर् के गुण:
• तथ्यों की जानकारी ःपष्ट, सटीक, ूामािणक हो |
• संःथा/ िवभाग के नाम का उल्लेख हो |
• अध्यक्ष आिद पदािधकािरयों के नाम |
• गितिविधयाँ चलानेवालों के नाम |
• कायर्बम का उद्दे ँय |
• आयोजन-ःथल, िदनांक, िदन तथा समय |
• उपिःथत लोगों की जानकारी |
• िदए गए भाषणों के ूमुख अंश |
• िलये गए िनणर्यों की जानकारी |
• भाषा आलंकािरक या सािहित्यक न हो कर सूचनात्मक होनी चािहए |
• सूचनाएँ अन्यपुरुष शैली में दी जाती हैं | मैं या हम का ूयोग नहीं होता |
• संिक्षप्तता और बिमकता िरपोटर् के गुण हैं |
• नई बात नए अनुच्छे द से िलखें |
• ूितवेदक या िरपोटर् र के हःताक्षर |
िनम्निलिखत िवषयों पर िरपोटर् तैयार कीिजए-
१. पूजा-ःथलों पर दशर्नािथर्यों की अिनयंिऽत भीड़
२. दे श की महँ गी होती व्यावसाियक िशक्षा
३. मतदान केन्ि का दृँय
४. आए िदन होती सड़क दघर्
ु टनाएँ
५. आकिःमक बाढ़ से हई
ु जनधन की क्षित
25 ६.फीचर लेखन- िनधार्िरत अंक:५
समकालीन घटना तथा िकसी भी क्षेऽ िवशेष की िविशष्ट जानकारी
के सिचऽ तथा मोहक
िववरण को फीचर कहते हैं |फीचर मनोरं जक ढं ग से तथ्यों को ूःतुत करने की कला है |
वःतुत: फीचर मनोरं जन की उं गली थाम कर जानकारी परोसता है| इस ूकार मानवीय रूिच के
िवषयों के साथ सीिमत समाचार जब चटपटा लेख बन जाता है तो वह फीचर कहा जाता है |
अथार्त- ज्ञान + मनोरं जन = फीचर |
फीचर में अतीत, वतर्मान और भिवंय की ूेरणा होती है | फीचर लेखक पाठक को वतर्मान
दशा से जोड़ता है , अतीत में ले जाता है और भिवंय के सपने भी बुनता है | फीचर लेखन
की
शैली िविशष्ट होती है | शैली की यह िभन्नता ही फीचर को समाचार, आलेख या िरपोटर् से अलग
ौेणी में ला कर खडा करती है | फीचर लेखन को अिधक ःपष्ट रूप से समझने के िलए िनम्न
बातों का ध्यान रखें –
१. समाचार साधारण जनभाषा में ूःतुत होता है और फीचर एक िवशेष वगर् व िवचारधारा
पर केंिित रहते हए
ु िविशष्ट शैली में िलखा जाता है |
२. एक समाचार हर एक पऽ में एक ही ःवरुप में रहता है परन्तु एक ही िवषय पर फीचर
अलग-अलग पऽोंमें अलग-अलग ूःतुित िलये होते हैं | फीचर के साथ लेखक का नाम
रहता है |
३. फीचर में अितिरक्त साज-सज्जा, तथ्यों और कल्पना का रोचक िमौण रहता है |
४. घटना के पिरवेश, िविवध ूितिबयाएँ वउनके
हैं |
दरगामी
पिरणाम भी फीचर में रहा करते
ू
५. उद्दे ँय की दृिष्ट से फीचर तथ्यों की खोज के साथ मागर्दशर्न और मनोरं जन की दिनया
ु
भी ूःतुत करता है |
६. फीचर
फोटो-ूःतुित से अिधक ूभावशाली बन जाता है |
नमूना फीचर :
िपयक्कड़ तोता :
संगत का असर आता है , िफर चाहे वह आदमी हो या तोता | िॄटे न में एक तोते को
अपने मािलक की संगत में शराब की ऐसी लत लगी िक उसने घर वालों और पड़ोिसयों
का जीना बेहाल कर िदया | जब तोते को सुधारने के सारे हथकंडे फेल हो गए तो
मजबूरन मािलक को ही शराब छोड़नी पड़ी | माकर् बेटोिकयो ने अृीकी ूजाित का तोता
मिलर्न पाला| माकर् यदा-कदा शराब पी लेते | िगलास में बची शराब मिलर्न चट कर जाता
| धीरे -धीरे मिलर्न की तलब बढ़ने लगी| वह वक्त-बेवक्त शराब माँगने लगा |------------------------------
26 िनम्निलिखत िवषयों पर फ़ीचरिलिखए:
•
चुनावी वायदे
•
महँ गाई के बोझतले मजदरू
•
वाहनों की बढ़ती संख्या
•
विरष्ठ नागिरकों के ूित हमारा नजिरया
•
िकसान का एक िदन
•
बांित के ःवप्न-िष्टा अब्दलकलाम
ु
•
•
िबकेट का नया संःकरण ट्वेंटी-ट्वेंटी
बेहतर संसाधन बन सकती है जनसंख्या
27 अध्ययन-साममी
कक्षा द्वादश- िहं दी (केंििक)
पद्य खंड
इस खंड से तीन ूकार के ूश्न पूछे जाएँगे –
ूश्न :-७ िदए गए काव्यांश में से अथर् महण संबंधी चार
ूश्न- २*४ =८ अंक
ूश्न ८ :- िदए गए काव्यांश में से सौंदयर्-बोध संबंधी तीन ूश्न -२*३=६ अंक
ूश्न ९ :-िवषय-वःतु पर आधािरत तीन ूश्नों में से दो ूश्नों के उत्तर -३*२=६ अंक
अिधक अंक-ूािप्त हे तु ध्यान दे ने योग्य बातें :•
किव, किवता का नाम ,एक दो वाक्यों में सुगिठत ूसंग या पंिक्तयों का सार अिनवायर्
रूप से याद होना चािहए .
•
वतर्नी एवं वाक्य गठन की अशुिद्धयाँ
० ( जीरो) % रखने के िलए उत्तरों का िलिखत
अभ्यास अिनवायर् है .
•
तीन अंक के ूश्न के उत्तर में अिनवायर्त: तीन िवचार-िबंद,ु शीषर्क बना कर रे खांिकत
करते हए
ु , बमबद्ध रूप से उत्तर िलखा जाना चािहए . अन्य ूश्नों के उत्तरों में
भी इसी
ूकार अंकों के अनुसार िवचार-िबंदओं
की संख्या होनी चािहए .
ु
•
समय पर पूणर् ूश्न-पऽ हल करने के िलए उत्तरों के मुख्य िबंद ु बम से याद होने चािहए .
•
उत्तरों के िवचार-िबन्दओं
ु एवं वाक्यों का िनिश्चत बम होना चािहए जो पहले से उत्तर याद
िकए िबना संभव नहीं होता .
•
काव्य-सोंदयर् संबंधी ूश्नों में अलंकार ,छं द आिद िवशेषताएँ किवता में से उदाहरण अंश
िलख कर ःपष्ट की जानी चािहए .
28 •
भाव-साम्य के रूप में अथर् से िमलती-जुलती अन्य किवयों की पंिक्तयाँ िलखना उपयुक्त है
.
•
किव एवं किवता की पृंठ्भूिम िशक्षक की सहायता से अिनवायर् रूप से समझ लें |
उपयुक्त ःथान पर इसे उत्तर में संकेितत भी करें .
29 1 किवता :–आत्म पिरचय
हिरवंश राय बच्चन
Ôआत्मपिरचयÕ- Ôिनशा िनमंऽणÕ गीत-संमह
का एक गीत
सार :१ ःवयं को जानना दिनया
को जानने से अिधक किठन भी है और आवँयक भी .
ु
२ व्यिक्त के िलए समाज से िनरपेक्ष एवं उदासीन रहना न तो संभव है न ही उिचत है .दिनया
ु
अपने व्यंग्य बाणों ,शासन Ðूशासन से चाहे िकतना कष्ट दे ,पर दिनया
से कट कर
ु
व्यिक्त
अपनी पहचान नहीं बना सकता .पिरवेश ही व्यिक्त को बनाता है , ढालता है .
३ इस किवता में किव ने समाज एवं पिरवेश से ूेम एवं संघषर् का संबंध िनभाते हए
ु जीवन में
सामंजःय ःथािपत करने की बात की है .
४ छायावादोत्तर
गीित काव्य में ूीित-कलह का यह िवरोधाभास िदखाई दे ता है . व्यिक्त और
समाज का संबंध इसी ूकार ूेम और संघषर् का है िजसमें किव आलोचना की परवाह न करते
हए
ु संतुलन ःथािपत करते हए
ु चलता है .
५ Ôनादान वहीं है हाय ,जहाँ पर दानाÕ पंिक्त के माध्यम से किव सत्य की खोज के िलए ,अहं कार
को त्याग कर नई सोच अपनाने पर जोर दे रहा है .
काव्य-खंड पर आधािरत दो ूकार के ूश्न पूछे जाएँगे Ð अथर्महण-संबंधी एवं सौंदयर्-बोध-संबंधी
अथर्महण-संबंधी ूश्न
1‐ मैं जग-जीवन का भार िलए िफरता हँू ,
िफर भी जीवन में प्यार िलए िफरता हँू ,
ू
कर िदया िकसी ने झंकृत िजनको छकर
,
मैं साँसों के दो तार िलए िफरता हँू .
30 ूश्न १:-किव अपने हृदय में क्या - क्या िलए िफरता है ?
उत्तर:- किव अपने सांसािरक अनुभवों के सुख - दख
ु हृदय में िलए िफरता है ।
ूश्न २:- किव का जग से कैसा िरँता है ?
उत्तर:- किव का जगजीवन से खट्टामीठा िरँता है ।
ूश्न३:- पिरवेश का व्यिक्त से क्या संबंध है ?मैं साँसों के दो तार िलए िफरता हँू . के माध्यम से
किव क्या कहना चाहता है ?
उत्तर:- संसार में रह कर संसार से िनरपेक्षता संभव नहींहै क्योंिक पिरवेश में रहकर ही व्यिक्त की
पहचान बनती है ।उसकी अिःमता सुरिक्षत रहती है ।
ूश्न४:- िवरोधों के बीच किव का जीवन िकस ूकार व्यतीत होता है ?
उत्तर:-दिनया
के साथ संघषर्पूणर् संबंध के चलते किव का जीवन-िवरोधों के बीच सामंजःय करते
ु
हए
ु व्यतीत होता है ।
सौंदयर्-बोध संबंधी ूश्न
“ मैं ःनेह-सुरा का पान िकया करता हँू ,
मैं कभी न जग का ध्यान िकया करता हँू ,
जग पूछ रहा उनको जो जग की गाते,
मैं अपने मन का गान िकया करता हँू ।”
ूश्न १:- किवता की इन पंिक्तयों से अलंकार छाँट कर िलिखए|
उत्तर :-ःनेह- सुरा– रूपक अलंकार
ूश्न २:- किवता में ूयुक्त मुहावरे िलिखए :उत्तर :-Ôजग पूछ रहाÕ‚ Ôजग की गातेÕ‚ Ôमन का गानÕ आिद मुहावरों का ूयोग
ूश्न३:- किवता में ूयुक्त शैली का नाम िलखें |
31 उत्तर :- गीित-शैली
32 किवता
आत्म-पिरचय
िवषय-वःतु पर आधािरत ूश्नोत्तर
ूश्न१:-किव कौन-कौन-सी िःथितयों में मःत रहता है और क्यों?
उत्तर:- किव सांसािरक सुख-दख
ु की दोनों पिरिःथितयों में मग्न रहता है ।उसके पास ूेम की
सांत्वना दाियनी अमूल्य िनिध है ।
ूश्न२:-किव भव-सागर से तरने के िलए क्या उपाय अपना रहा है ?
उत्तर:- संसार के कष्टों को सहते हए
ु हमें यह ध्यान रखना चािहए िक कष्टों को सहना पड़े गा।
इसके िलए मनुंय को हँ स कर कष्ट सहना चािहए।
ूश्न३:-Õअपने मन का गानÕ का क्या आशय है ?
उत्तर:- संसार उन लोगों को आदर दे ता है जो उसकी बनाई लीक पर चलते हैं परं तु किव केवल
वही कायर् करता है जो उसके मन, बुिद्ध और िववेक
को अच्छा लगता है ।
ूश्न४:- Õनादान वहीं हैं हाय जहाँ पर दानाÕ का क्या आशय है ?
उत्तर:- जहाँ कहीं मनुंय को िवद्वत्ता का अहं कार है वाःतव में वही नादानी का सबसे बड़ा लक्षण
है ।
ूश्न५:- Õरोदन में रागÕ कैसे संभव है ?
उत्तर:- किव की रचनाओं में व्यक्त पीड़ा वाःतव में उसके हृदय में मानव माऽ के ूित व्याप्त ूेम
का ही सूचक है ।
ूश्न६:- मैं फूट पड़ा तुम कहते छं द बनाना” का अथर् ःपष्ट कीिजए।
उत्तर:- किव की ौेष्ठ रचनाएँ वाःतव में उसके मन की पीड़ा की ही अिभव्यिक्त है िजनकी
सराहना संसार ौेष्ठ सािहत्य कहकर िकया करता है ।
33 किवता
िदन जल्र्दी जल्दी ढलता है ।
ूःतुत किवता में
किव बच्चन कहते हैं िक समय बीतते जाने का एहसास हमें लआय-ूािप्त के
िलए ूयास करने के िलए ूेिरत करता है ।
मागर् पर चलने वाला राही यह सोचकर अपनी मंिजल की ओर कदम बढ़ाता है िक कहीं राःतें में
ही रात न हो जाए।
पिक्षयों को भी िदन बीतने के साथ यह एहसास होता है िक उनके बच्चे कुछ पाने की आशा में
घोंसलों से झांक रहे होंगे। यह सोचकर उनके पंखो में गित आ जाती है िक वे जल्दी से अपने
बच्चों से िमल सकें।
किवता में आशावादी ःवर है ।गंतव्य का ःमरण पिथक के कदमों में ःफूितर् भर दे ता है ।आशा की
िकरण जीवन की जड़ता को समाप्त कर दे ती है । वैयिक्तक अनुभिू त का किव होने पर भी बच्चन
जी की रचनाएँ िकसी सकारात्मक सोच तक ले जाने का ूयास हैं ।
अथर्महण-संबंधी ूश्न
बच्चे ूत्याशा में होंगे
नीड़ों से झांक रहे होंगे।
यह ध्यान परों में िचिड़या के
भरता िकतनी चंचलता है ।”
ूश्न १ :-पथ और रात से क्या तात्पयर् है ?
उत्तर :-जीवन रूपी पथ में मृत्यु रूपी रात से सचेत रहने के िलए कहा गया है ।
ूश्न२ :-पिथक के पैरों की गित िकस ूकार बढ़ जाती है ?
उत्तर :-मंिजल के पास होने का अहसास‚ व्यिक्त के मन में ःफूितर् भर दे ता है ।
ूश्न३ :-िचिड़या की चंचलता का क्या कारण है ?
उत्तर :-िचिड़या के बच्चे उसकी ूतीक्षा करते होंगे यह िवचार िचिड़या के पंखों में गित भर कर
उसे चंचल बना दे ता है ।
ूश्न४ :-ूयासों में तेजी लाने के िलए मनुंय को क्या करना चािहए?
34 उत्तर :- ूयासों में तेजी लाने के िलए मनुंय को जीवन में एक िनिश्चत मधुर लआय ःथािपत
करना चािहए।
2 पतंग
आलोक धन्वा
पतंग किवता में किव आलोक धन्वा बच्चों की बाल सुलभ इच्छाओं और उमंगों तथा ूकृ ित के
साथ उनके रागात्मक संबंधों का अत्यंत सुन्दर िचऽण िकया है ।भादों मास गुजर जाने के बाद
शरद ऋतु का आगमन होता है ।चारों ओर ूकाश फैल जाता है ।सवेरे के सूयर् का ूकाश लाल
चमकीला हो जाता है ।शरद ऋतु के आगमन से उत्साह एवं उमंग का माहौल बन जाता है ।
शरद ऋतु का यह चमकीला इशारा बच्चों को पतंग उड़ाने के िलए बुलाता है , और पतंग उड़ाने
के िलए मंद मंद वायु चलाकर आकाश को इस योग्य बनाता है िक दिनया
की सबसे हलके
ु
रं गीन कागज और बांस की सबसे पतली कमानी से बनी पतंगें आकाश की ऊँचाइयों में उड़
सके‘।बच्चों के पाँवों की कोमलता से आकिषर्त हो कर मानो धरती उनके पास आती है अन्यथा
उनके पाँव
धरती पर पड़ते ही नहीं| ऐसा लगता है मानो वे हवा में उड़ते जा रहे हैं ।पतंग उड़ाते
समय बच्चे रोमांिचत होते हैं |एक संगीतमय ताल पर उनके शरीर हवा में लहराते हैं ।वे िकसी भी
खतरे से िबलकुल बेखबर होते हैं ।बाल मनोिवज्ञान. बाल िबया– कलापों एवं बाल सुलभ इच्छाओं
का सुद
ं र िबंबों के माध्यम से अंकन िकया गया है ।
सौंदयर्-बोध संबंधी ूश्न
Ôजन्म से ही लाते हैं अपने साथ कपासÕÔिदशाओं को मृदंग की बजाते हए
ु Õ
Ôऔर भी िनडर हो कर सुनहले सूरज के सामने आते हैंÕ।
Ôछतों को और भी नरम बनाते हए
ु Õ।
Ôजब वे पेंग भरते हए
ु चले आते हैं
डाल की तरह लचीले वेग से अक्सर।Ô
35 ूश्न १:- Ôजन्म से ही लाते हैं अपने साथ कपासÕइस पंिक्त की भाषा संबंधी िवशेषता िलिखए |
उत्तर :- इस पंिक्त की भाषा संबंधी िवशेषता िनम्निलिखत हैं :नए ूतीकों का ूयोग :-कपास-कोमलता
ूश्न२:- इस पंिक्त में ूयुक्त लाक्षिणक अथर् को ःपष्ट कीिजए|
उत्तर :- लाक्षिणकता -Ôिदशाओं को मृदंग की बजाते हए
ु Õ-संगीतमय वातावरण की सृिष्ट
ूश्न३ :- सुनहला सूरज ूतीक का अथर् िलखें |
उत्तर :- सुनहले सूरज के सामने: Ð िनडर ,उत्साह से भरे होना
3 किवता के बहाने
कुँवर नारायण
कुँवर नारायणकी रचनाओं में संयम‚ पिरंकार एवं साफ सुथरापन है ।यथाथर् का कलात्मक
संवेदनापूणर् िचऽण उनकी रचनाओं की िवशेषता है ।उनकी रचनाएँ जीवन को समझने की िजज्ञासा
है यथाथर्– ूािप्त की घोषणा नहीं।वैयिक्तक एवं सामािजक तनाव व्यंजनापूणर् ढ़ं ग से उनकी
रचनाओं में ःथान में पाता है ।ूःतुत किवता में किवत्व शिक्त का वणर्न है । किवता िचिड़या की
उड़ान की तरह कल्पना की उड़ान है लेिकन िचिड़या के उड़ने की अपनी सीमा है जबिक किव
अपनी कल्पना के पंख पसारकर दे श और काल की सीमाओं से परे उड़ जाता है ।
फूल किवता िलखने की ूेरणा तो बनता है लेिकन किवता तो िबना मुरझाए हर युग में अपनी
खुशबू िबखेरती रहती है ।
किवता बच्चों के खेल के समान है और समय और काल की सीमाओं की परवाह िकए िबना
अपनी कल्पना के पंख पसारकर उड़ने की कला बच्चे भी जानते है ।
•
मानवी िबंबों के माध्यम से काव्य रचना– ूिबया को ूःतुत िकया गया है ।
•
किवता में िचिड़या फूल और बच्चे के ूतीकों के माध्यम से बच्चे की रचनात्मक ऊजार्
की तुलना किवता-रचना से की गई है ।िचिड़या की उड़ान फूल का िवकास अपनी सीमा में
आबद्ध है परन्तु किव की कल्पना शिक्त एवं बालक के ःवप्न व ऊजार् असीम है ।
36 •
सािहत्य का महत्व‚ ूाकृ ितक सौन्दयर् की अपेक्षा मानव के भाव-सौन्दयर् की ौेष्ठता का
ूितपादन िकया गया है ।
किवता की उड़ान है िचिड़या के बहाने
किवता की उड़ान भला िचिड़या क्या जाने
बाहर भीतर
इस घर ,उस घर
किवता के पंख लगा उड़ने के माने
िचिड़या क्या जाने?
ूश्न १:- इन पंिक्तयों की भाषा संबंधी िवशेषताएं िलिखए |
उत्तर :- इन पंिक्तयों की भाषा संबंधी िवशेषताएं िनम्निलिखत हैं :१ नए ूतीक :-िचिड़या‚ ।
२ मुहावरों का सटीक ूयोग :–सब घर एक कर दे ना‚ किव की कल्पना की उवर्र शिक्त
िजसका ूयोग करने में किव जमीन-आसमान एक कर दे ता है |
ूश्न २ किवता की उड़ान Ðका लाक्षिणक अथर् ःपष्ट कीिजए |
उत्तर :-काव्य की सूआम अथर् िनरूपण शिक्त ,किव की कल्पना का िवःतार
ूश्न :- किवता के पंख िकसका ूतीक हैं ?
उत्तर :-किव की कल्पना शिक्त का |
37 3 किवता के बहाने
कुँवर नारायण
िवषय-वःतु पर आधािरत ूश्नोत्तर
ूश्न१:-िचिड़या की उड़ान एवं किवता की उड़ान में क्या समानता है ?
उत्तर :-उपकरणों की समानता :- िचिड़या एक घोंसले का सृजन ितनके एकऽ करके करती है|
किव भी उसी ूकार अनेक भावों एवं िवचारों का संमह करके काव्य रचना करता है ।
क्षमता की समानता :- िचिड़या की उड़ान और किव की कल्पना की उड़ान दोनों दरू तक जाती
हैं|
ूश्न२ :-किवता की उड़ान िचिड़या की समझ से परे क्यों है ?
उत्तर :-किवता की उड़ान िचिड़या की उड़ान से कहीं अिधक सूआम और महत्त्वपूणर् होती है ।
ूश्न३ :-“फूल मुरझा जाते हैं पर किवता नहीं” क्यों ःपष्ट कीिजए।
उत्तर :- किवता कालजयी होती है उसका मूल्य शाश्वत होता है
जबिक फूल बहत
ु जल्दी कुम्हला
जाते हैं ।
ूश्न४ :-“बच्चे की उछल-कूद. सब घर एक कर दे ना’
एवं ‘किव का किवता िलखना’ दोनों मे
क्या समानता एवं िवषमता है ?
उत्तर :-बच्चा खेल-खेल में घर का सारा सामान अःतव्यःत कर दे ता है . सब कुछ टटोलता है
एक ःथान पर एकऽ कर लेता है काव्र्य रचना-ूिबया में किव भी पूरा मानव जीवन खंगाल लेता
है . एक जगह िपरोता है पर िफर भी दोनों के ूयासों में बाल-िबयाओं का आनंद किव नहीं
समझ सकता।
किवता – बात सीधी थी पर
ूःतुत किवता में भाव के अनुरूप भाषा के महत्त्व पर बल िदया गया है ।
38 किव कहते हैं िक एक बार वह सरल सीधे कथ्य की अिभव्यिक्त में भी भाषा के चक्कर में ऐसा
फँस
गया िक उसे कथ्य ही बदलाने लगा। किव कहता है िक िजस ूकार जोर बदला सा लग-
जबरदःती करने से कील की चूड़ी मर जाती है और तब चूड़ीदार कील को चूड़ीिवहीन कील की
तरह ठोंकना पड़ता है उसी ूकार कथ्य के अनुकूल भाषा के अभाव में ूभावहीन भाषा में भाव
को अिभव्यिक्त िकया जाता है ।
अंत में भाव ने एक शरारती बच्चे के समान किव से पूछा िक तूने क्या अभी तक भाषा का
ःवाभािवक ूयोग नहीं सीखा।
•
इस किवता में भाषा की संूेषण-शिक्त का महत्त्व
दशार्या गया है ।
•
कृ िऽमता एवं भाषा की अनावँयक पच्चीकारी से भाषा की पकड़ कमज़ोर हो जाती है ।
शब्द अपनी अथर्वत्ता खो बैठता है ।
•
उनकी किवता में व्यथर् का उलझाव, अखबारी सतहीपन और वैचािरक धुंध के बजाय
संयम ,पिरंकार और साफ़-सुथरापन है
आिखरकार वही हआ
िजसका मुझे डर था
ु
ज़ोर ज़बरदःती से
बात की चूड़ी मर गई
और वह
भाषा में बेकार घूमने लगी !
हार कर मैंने उसे कील की तरह ठोंक िदया !
ऊपर से ठीकठाक
पर अंदर से
न तो उसमें कसाव था
न ताकत !
बात ने ,जो एक शरारती बच्चे की तरह
मुझसे खेल रही थी,
मुझे पसीना पोंछते दे खकर पूछा –
39 क्या तुमने भाषा को
सहिलयत
से बरतना कभी नहीं
ू
सीखा ?
ूश्न१:- इनपंिक्तयों की भाषा संबंधी िवशेषताएं िलिखए |
उत्तर :- इनपंिक्तयों की भाषा संबंधी िवशेषताएं िनम्निलिखतहैं :१ िबंब /मुहावरों का ूयोग
२ नए उपमान
ूश्न २ काव्यांश में आए मुहावरों का अथर् ःपष्ट कीिजए|
उत्तर :•
िबंब /मुहावरों का अथर्
:- बात की चूड़ी मर जाना Ðबात में
कसावट न होना
बात का शरारती बच्चे की तरह खेलना Ðबात का
पकड़
में न आना
पेंच को कील की तरह ठोंक दे ना Ðबात का
ूभावहीन
हो जाना
ूश्न ३ :- काव्यांश में आएउपमानों को ःपष्ट कीिजए|
उत्तर:ƒ
नए उपमान – अमूतर् उपमेय भाषा के िलए मूतर् उपमान कील का ूयोग।
ƒ
बात के िलए शरारती बच्चे का उपमान
40 किवता–
बात सीधी थी पर
िवषय-वःतु पर आधािरत ूश्नोत्तर
ूश्न १ :-
भाषा के चक्कर में बात कैसे फंस जाती है ?
उत्तर :-आडं बरपूणर् भाषा का ूयोग करने से बात का अथर् समझना किठन हो जाता है ।
ूश्न२ :-
भाषा को अथर् की पिरणित तक पहँु चाने के िलए किव क्या क्या ूयास करते हैं ?
उत्तर :- भाषा को अथर् की पिरणित तक पहँु चाने के िलए किव उसे नाना ूकार के अलंकरणों से
सजाता है कई ूकार के भाषा और अलंकार संबंधी ूयोग करता है ।
ूश्न३:- भाषा मे पेंच कसना क्या है ?
उत्तर :-भाषा को चामत्कािरक बनाने के िलए िविभन्न ूयोग करना भाषा मे पेंच कसना है
परं तु इससे भाषा का पेंच ज्यादा कस जाता है अथार्त कथ्य एवं शब्दों में कोई तालमेल नहीं
बैठता, बात समझ में ही नहीं आती।
ूश्न४:- किव िकस चमत्कार के बल पर वाहवाही की उम्मीद करता है ?
उत्तर :-किव शब्दों के चामत्कािरक ूयोग के बल पर वाहवाही की उम्मीद करता है ।
ूश्न५:-बात एवं शरारती बच्चे का िबंब ःपष्ट कीिजए।
उत्तर :-िजस ूकार एक शरारती बच्चा िकसी की पकड़ में नहीं आता उसी ूकार एक उलझा दी
गई बात तमाम कोिशशों के बावजूद समझने के योग्य नहीं रह जाती चाहे उसके िलए िकतने
ूयास िकए जाएं,वह एक शरारती बच्चे की तरह हाथों से िफसल जाती है ।
४: कैमरे में बंद अपािहज
41 रघुवीर सहाय
अथर्-महण-संबंधी ूश्न
हम दरदशर्
न पर बोलेंगे
ू
हम समथर् शिक्तवान
हम एक दबर्
ु ल को लाएँगे
एक बंद कमरे में
उससे पूछेंगे तो आप क्या अपािहज हैं ?
तो आप क्यों अपािहज हैं ?
ूश्न १:- Ôहम दरदशर्
न पर बोलेंगे हम समथर् शिक्तवानÕ-का िनिहत अथर् ःपष्ट कीिजए |
ू
उत्तर :- इन पंिक्तयों में अहं की ध्विनत अिभव्यिक्त है ‚ पऽकािरता का बढ़ता वचर्ःव दशार्या गया
है ।
ूश्न२:- हम एक दबर्
ु ल को लाएँगे– पंिक्त का व्यंग्याथर् ःपष्ट कीिजए |
उत्तर :- पऽकािरता के क्षेऽ में करुणा का खोखला ूदशर्न एक पिरपाटी बन गई है|
ूश्न३:- आप क्या अपािहज हैं ?
तो आप क्यों अपािहज हैं ?पंिक्त द्वारा किव िकस िविशष्ट अथर् की अिभव्यिक्त करने में सफल
हआ
है ?
ु
उत्तर :- पऽकािरता में व्यावसाियकता के चलते संवेदनहीनता बढ़ती जा रही है | यहाँ अपेिक्षत
उत्तर ूाप्त करने का अधैयर् व्यक्त हआ
है ।
ु
सौंदयर्-बोध-महण संबंधी अन्य ूश्न
ूश्न १:- इन पंिक्तयों का लाक्षिणक अथर् ःपष्ट कीिजए |
िफर हम परदे पर िदखलाएँगे
42 फूली हई
ु ऑ ंख की एक बड़ी तःवीर
बहुत बड़ी तःवीर
और उसके होंठों पर एक कसमसाहट भी
उत्तर:- - लाक्षिणक अथर् :- दृँय माध्यम का ूयोग, कलात्मक, काव्यात्मक, सांकेितक ूःतुित
का माऽ िदखावा।
कैमरा बस करो
नहीं हआ
ु
रहने दो
लाक्षिणक अथर् :-
व्यावसाियक उद्दे ँय पूरा न होने की खीझ
परदे पर वक्त की कीमत है । –
लाक्षिणक अथर् :-- सूऽ वाक्य ‚बूर व्यावसाियक उद्दे ँय का उद्घाटन
ूश्न२ :- रघुवीर सहाय की काव्य कला की िवशेषताएँ िलिखए |
उत्तर :- रघुवीर सहाय की काव्य कला की िवशेषताएँिनम्निलिखत हैं •
कहानीपन और नाटकीयता
•
बोलचाल की भाषा के शब्दः– बनाने के वाःते‚ संग रुलाने हैं ।
•
सांकेितकता–
•
िबंब :–फूली हई
ु आँख की एक बड़ी तःवीर
परदे पर वक्त की कीमत है ।
कैमरे में बंद अपािहज
िवषय-वःतु पर आधािरत ूश्नोत्तर
ूश्न१:- दरदशर्
न पर एक अपािहज का साक्षात्कार िकस उद्दे ँय से िदखाया जाता है ?
ू
43 उत्तर :-दरदशर्
न पर एक अपािहज का साक्षात्कार‚ व्यावसाियक उद्दे ँयों को पूरा करने के िलए
ू
िदखाया जाता है ।
ूश्न२:- अंधे को अंधा कहना िकस मानिसकता का पिरचायक है ?
उत्तर :-अंधे को अंधा कहना‚ बूर और संवेदनाशून्य मानिसकता का पिरचायक है ।
ूश्न३ :-किवता में यह मनोवृित िकस ूकार उद्घािटत हई
ु है ?
उत्तर :- दरदशर्
न पर एक अपािहज व्यिक्त को ूदशर्न की वःतु मान कर उसके मन की पीड़ा को
ू
कुरे दा जाता है ‚ साक्षात्कारकत्र्ता को उसके िनजी सुखदख
ु से कुछ लेनादे ना नहीं होता है ।
ूश्न४ :-‘हम समथर् शिक्तवान एवं हम एक दबर्
ु ल को लाएंगे’ में िनिहताथर् ःपष्ट कीिजए।
उत्तर :-साक्षात्कारकतार्ःवयं को पूणर् मान कर‚ एक अपािहज व्यिक्त को दबर्
ु ल समझने का अहं कार
पाले हए
ु है ।
ूश्न५:- अपािहज की शब्दहीन पीड़ा को मीिडयाकमीर् िकस ूकार अिभव्यक्त कराना चाहता है ?
उत्तर :-मीिडयाकमीर् अपािहज की लाल सूजी हई
ं को‚ पीड़ा की सांकेितक अिभव्यिक्त के रूप
ु ऑखों
में ूःतुत करना चाहता है ।
ूश्न६:-क्यामीिडयाकमीर् सफल होता है ‚ यिद नहीं तो क्यों?
उत्तर :-मीिडयाकमीर् सफल नहीं होता क्यों िक ूसारण समय समाप्त हो जाता है और ूसारण
समय के बाद यिद अपािहज व्यिक्त रो भी दे ता तो उससे मीिडयाकमीर् का व्यावसाियक उद्दे ँय
पूरा नहीं हो सकता था उसिलए अब उसे अपािहज व्यिक्त के आंसओ
ु ं में कोई िदलचःपी नहीं थी।
ूश्न७:- नाटकीय किवता की अंितम पिरणित िकस रूप में होती है ?
उत्तर :-बार बार ूयास करने पर भी मीिडयाकमीर्‚ अपािहज व्यिक्त को रोता हआ
नहीं िदखा
ु
पाता।वह खीझ जाता है और िखिसयानी मुःकुराहट के साथ कायर्बम समाप्त कर दे ता है
|Õसामािजक उद्दे ँय से युक्त कायर्बमÕशब्दों में व्यंग्य है क्योंिक मीिडया के छद्म व्यावसाियक
उद्दे ँय की पूितर् नहीं हो पाती |
ूश्न८ :-‘परदे पर वक्त की कीमत है ’ में िनिहत संकेताथर् को ःपष्ट कीिजए।
44 उत्तर :-ूसारण
समय में रोचक साममी परोस पाना ही मीिडया किमर्यों का एकमाऽ उद्दे ँय होता
है ।अन्यथा उनके सामािजक सरोकार माऽ एक िदखावा हैं ।
5 सहषर् ःवीकारा है
गजानन माधव मुिक्तबोध
सार
•
किवता में जीवन के सुख– दख
ु ‚ संघषर्– अवसाद‚ उठा– पटक को समान रूप से ःवीकार
करने की बात कही गई है ।
•
ःनेह की ूगाढ़ता अपनी चरम सीमा पर पहँु च कर िवयोग की कल्पना माऽ से ऽःत हो
उठती है ।
•
ूेमालंबन अथार्त िूयजन
पर यह भावपूणर् िनभर्रता‚ किव के मन में िवःमृित की चाह
उत्पन्न करती है ।वह अपने िूय को पूणत
र् या भूल जाना चाहता है |
•
वःतुतः िवःमृित की चाह भी ःमृित का ही रूप है । यह िवःमृित भी ःमृितयों के धुंधलके
ू
से अछती
नहीं है ।िूय की याद िकसी न िकसी रूप में बनी ही रहती है|
•
परं तु किव दोनों ही पिरिःथितयों को उस परम ् सत्ता की परछाईं मानता है ।इस पिरिःथित
को खुशी Ðखुशी ःवीकार करता है |दःख
ु -सुख ,संघषर् Ðअवसाद,उठा Ðपटक, िमलन-िबछोह
को
समान भाव से ःवीकार करता है|िूय के सामने न
होने
पर भी उसके आस-पास
होने का अहसास बना रहता है|
•
भावना की ःमृित िवचार बनकर िवश्व की गुित्थयां सुलझाने में मदद करती है| ःनेह में
थोड़ी िनःसंगता भी जरूरी है |अित िकसी चीज की अच्छी नहीं |ÕवहÕ यहाँ कोई भी हो
सकता है िदवंगत माँ िूय या अन्य |कबीर के राम की तरह ,वड्सर्वथर् की मातृमना
ूकृ ित की तरह यह ूेम सवर्व्यापी होना चाहता है |
मुःकाता चाँद ज्यों धरती पर रात भर
मुझ पर त्यों तुम्हारा ही िखलता वह चेहरा है !
•
छायावाद के ूवतर्क ूसाद की लेखनी से यह ःवर इस ूकार ध्विनत हआ
है –
ु
दख
ु की िपछली रजनी बीच िवकसता सुख का नवल ूभात।
एक परदा यह झीना नील िछपाए है िजसमें सुख गात।
45 यह किवता Ôनई किवताÕ में व्यक्त रागात्मकता को आध्याित्मकता के ःतर पर ूःतुत
करती है ।
अथर्महण-संबंधी ूश्न
िज़ंदगी में जो कुछ भी है
सहषर् ःवीकारा है ;
इसिलए िक जो कुछ भी मेरा है
वह तुम्हें प्यारा है|
गरबीली गरीबी यह, ये गंभीर अनुभव सबयह वैभव िवचार सब
दृढ़ता यह,भीतर की सिरता यह अिभनव सब
मौिलक है , मौिलक है
इसिलए िक पल-पल में
जो कुछ भी जामत है अपलक है संवेदन तुम्हारा है !
ूश्न १:- किव और किवता का नाम िलिखए|
उत्तर:-किव- गजानन माधव मुिक्तबोध
किवताÐसहषर् ःवीकारा है
ूश्न२:- गरबीली गरीबी,भीतर की सिरता आिद ूयोगों का अथर् ःपष्ट कीिजए |
उत्तर :-गरबीली गरीबी– िनधर्नता का ःवािभमानी रूप ।किव के िवचारों की मौिलकता ,अनुभवों
की गहराई ,दृढ़ता ,हृदय का ूेम उसके गवर् करने का कारण है |
ूश्न३ :- किव अपने िूय को िकस बात का ौेय दे रहा है ?
उत्तर:- िनजी जीवन के ूेम का संबंल किव को िवश्व व्यापी ूेम से जुड़ने की ूेरणा दे ता है |अत:
किव इसका ौेय अपने िूय को दे ता है |
46 सौंदयर्-बोध-महण संबंधी ूश्न
जाने क्या िरँता है , जाने क्या नाता है
िजतना भी उं डे लता हँू ,भर Ðभर िफर आता है
िदल में क्या झरना है ?
मीठे पानी का सोता है
भीतर वह ,ऊपर तुम
मुःकाता चाँद ज्यों धरती पर रात- भर
मुझ पर त्यों तुम्हारा ही िखलता वह चेहरा है |
ूश्न१:- किवता की भाषा संबंधी दो िवशेषताएँ िलिखए |
उत्तर:- १-सटीक ूतीकों,
२- नये उपमानों का ूयोग
ूश्न२ :- िदल में क्या झरना है ?
मीठे पानी का सोता है ? - -के लाक्षिणक अथर् को ःपष्ट कीिजए |
उत्तर :- िदल में क्या झरना है ?-हृदय के अथाह ूेम का पिरचायक
मीठे पानी का सोता है ? -अिवरल, कभी समाप्त होने वाला ूेम
ूश्न३:- किवता में ूयुक्त िबंब का उदाहरण िलिखए |
दृँय िबंब–
मुःकाता चाँद
ज्यों धरती पर रात भर। मुझ पर तुम्हारा ही िखलता वह चेहरा।
47 5 सहषर् ःवीकारा है
गजानन माधव मुिक्तबोध
िवषय-वःतु पर आधािरत ूश्नोत्तर
ूश्न१:-किव ने िकसे सहषर् ःवीकारा है ?
उत्तर:•
किवता में जीवन के सुख– दख
ु ‚ संघषर्– अवसाद‚ उठा– पटक को समान रूप से ःवीकार
करने की बात कही गई है ।
•
ु कर भी उसकी ःमृितयों को व्यापक ःतर पर ले जाकर िवश्व चेतना में
िूय से िबछड़
िमला दे ने की बात कही गई है |
ूश्न२:-किव को अपने अनुभव िविशष्ट एवं मौिलक क्यों लगते हैं ?
उत्तर:-किव को अपनी ःवािभमानयुक्त गरीबी, जीवन के गम्भीर अनुभव िवचारों का
वैभव,
व्यिक्तत्व की दृढ़ता, मन की भावनाओं की नदी, यह सब नए रूप में मौिलक लगते हैं क्यों िक
उसके जीवन में जो कुछ भी घटता है वह
और
जामत है , िवश्व
उपयोगी है अत: उसकी उपलिब्ध है
है । उसके जीवन का ूत्येक अभाव ऊजार्
वह उसकी िूया की ूेरणा से ही संभव हआ
ु
बनकर जीवन में नई िदशा ही दे ता रहा है |
ूश्न३:- “िदल का झरना” का सांकेितक अथर् ःपष्ट कीिजए।
उत्तर:-िजस ूकार झरने में चारों ओर की पहािड़यों से पानी इकट्टठा हो जाता है उसे एक कभी
खत्म न होने वाले ॐोत के रूप में ूयोग िकया जा सकता है उसी ूकार किव के िदल में िःथत
ूेम उमड़ता है , कभी समाप्त नहीं होता। जीवन का िसंचन करता है| व्यिक्तगत ःवाथर् से दरू पूरे
समाज के िलए जीवनदायी हो जाता है |
ूश्न४:- ‘िजतना भी उँ ड़े लता हँू भर-भर िफर आता है
उत्तर:-हृदय
का िवरोधाभास ःपष्ट कीिजए।
में िःथत ूेम की िवशेषता यह है िक िजतना अिधक व्यक्त िकया जाए उतना ही
बढ़ता जाता है ।
ूश्न५:- वह रमणीय उजाला क्या है िजसे किव सहन नहीं कर पाता ?
48 उत्तर:-किव ने िूयतमा की आभा से,ूेम के सुखद भावों से सदै व िघरे रहने की िःथित को उजाले
के रूप में िचिऽत िकया है ।इन ःमृितयों से िघरे रहना आनंददायी होते हए
ु भी किव के िलए
असहनीय हो गया है क्योंिक इस आनंद से वंिचत हो जाने का भय भी उसे सदै व सताता रहता
है ।
6. उषा
शमशेर बहादरु िसंह
सार
उषा किवता में सूयोर्दय के समय आकाश मंडल में रं गों के
जाद ू का सुन्दर वणर्न िकया गया
है । सूयोर्दय के पूवर् ूातःकालीन आकाश नीले शंख की तरह बहत
ु नीला होता है । भोरकालीन
नभ की तुलना काली िसल से की गयी है िजसे अभी-अभी केसर पीसकर धो िदया गया है । कभी
किव को वह राख से लीपे चौके के समान लगता है , जो अभी गीला पड़ा है । नीले गगन में सूयर्
की पहली िकरण ऐसी िदखाई दे ती है मानो कोई सुद
ं री नीले जल में नहा रही हो और उसका
गोरा शरीर जल की लहरों के साथ िझलिमला रहा हों।
ूात:कालीन, पिरवतर्नशील सौंदयर्
का दृँय िबंब ,ूाकृ ितक पिरवतर्नों को मानवीय िबयाकलापों
के माध्यम से व्यक्त िकया गया है । यथाथर् जीवन से चुने गए उपमानों जैस:े - राख से लीपा
चौका ,काली िसल,नीला शंख, ःलेट,लाल खिड़या चाक आिद का ूयोग
िवभावरी जाग री से तुलना की जा सकती है ।
किवता– उषा
अथर्-महण-संबंधी ूश्न
ूात नभ था बहत
ु नीला शंख जैसे
भोर का नभ
राख से लीपा चौका
(अभी गीला पड़ा है )
बहत
ु काली िसल ज़रा से लाल केसर
से िक जैसे धुल गई हो
ःलेट पर या लाल खिड़या चाक
मल दी हो िकसी ने
49 । ूसाद की कृ ित –बीती
नील जल में या िकसी की
गौर िझलिमल दे ह
जैसे िहल रही हो |
और .....
ू
जाद ू टटता
है इस उषा का अब
सूयोर्दय हो रहा है|
ूश्न१ :-उषा किवता में सूयोर्दय के िकस रूप को िचिऽत िकया गया है ?
उत्तर :-किव ने ूातःकालीन, पिरवतर्नशील सौंदयर् का दृँय िबंब मानवीय िबयाकलापों के माध्यम
से व्यक्त िकया है ।
ूश्न२ :-भोर के नभ और
राख से लीपे गए
चौके में क्या समानता है ?
उत्तर :-भोर के नभ और
राख से लीपे गए
चौके में यह समानता है िक दोनों ही गहरे सलेटी
रं ग के हैं ,पिवऽ हैं ।नमी से युक्त हैं ।
ूश्न३ :- ःलेट पर लाल ....पंिक्त का अथर् ःपष्ट कीिजए|
उत्तर :-भोर का नभ लािलमा से युक्त ःयाही िलए हए
ु होता है |अत: लाल खिड़या चाक से मली
गई ःलेट जैसा ूतीत होता है |
ूश्न४:- उषा का जाद ू िकसे कहा गया है ?
उत्तर िविवध रूप रं ग बदलती सुबह व्यिक्त पर जादई
ु ूभाव डालते हए
ु उसे मंऽ मुग्ध कर दे ती है
|
सौंदयर्-बोध-संबंधी िवशेषताएँ
ूश्न१:- ।किवता में ूयुक्त उपमानों को ःपष्ट कीिजए।
•
भोर का नभ राख से लीपा चौका दोनों ही गहरे सलेटी रं ग के हैं ।पिवऽ हैं ।नमी से युक्त
हैं ।
50 •
काली िसल भोर का नभ और
लालकेसर से धुली काली िसल दोनों ही लािलमा से
युक्त हैं ।
•
काली िसलेट जो लाल खिड़या चाक से मल दी गई हो और भोर का नभ दोनों ही
लािलमा से
•
युक्त हैं ।
ूातः काल के ःवच्छ िनमर्ल आकाश में सूयर् ऐसा ूतीत होता है मानो नीलजल में कोई
ःविणर्म दे ह नहा रही हो।
ूश्न२:- किवता की भाषा एवं अिभव्यिक्त संबंधी िवशेषताएं िलिखए।
उत्तर :-१
यथाथर् जीवन से चुने गए उपमान –राख से लीपा चौका।
२ दृँयिबंब
ूश्न ३
उत्तर :-
किवता में आए अलंकारों को छॉंटकर िलिखए।
उपमा अलंकार:- भोर का नभ
राख से लीपा चौका
उत्ूेक्षा अलंकार:“ बहत
ु काली िसल जरा से लाल केसर से िक जैसे धुल गई हो।
नील जल में या िकसी की
गौर िझलिमल दे ह
जैसे िहल रही हो “
िवषय-वःतु पर आधािरत ूश्नोत्तर
ूश्न१:- किवता के िकन उपमानों को दे ख कर कहा जा सकता है िक उषा गाँव की सुबह का
गितशील शब्द िचऽ है ?
उत्तर :-किवता में नीले नभ को राख से िलपे गीले चौके के समान बताया गया है | दसरे
िबंब में
ू
उसकी तुलना काली िसल से की गई है| तीसरे में ःलेट पर लाल खिड़या चाक का उपमान
है|लीपा हआ
आँगन ,काली िसल या ःलेट गाँव के पिरवेश से ही िलए गए हैं |ूात: कालीन
ु
सौंदयर् बमश: िवकिसत होता है |सवर्ूथम राख
से लीपा चौका
जो गीली राख के कारण गहरे
ःलेटी रं ग का अहसास दे ता है और पौ फटने के समय आकाश के गहरे ःलेटी रं ग से मेल खाता
51 है |उसके पश्चात तिनक लािलमा के िमौण से काली िसल का जरा से लाल केसर से धुलना
सटीक उपमान है तथा
सूयर् की लािलमा के रात की काली ःयाही में घुल जाने का सुद
ं र िबंब
ूःतुत करता है | धीरे Ðधीरे लािलमा भी समाप्त हो जाती है और सुबह का नीला आकाश नील
जल का आभास दे ता है व सूयर् की ःविणर्म आभा गौरवणीर् दे ह
के नील जल में नहा कर
िनकलने की उपमा को साथर्क िसद्ध करती है | ूश्न२ :भोर का नभ
राख से लीपा चौका
(अभी गीला पड़ा है )
नयी किवता में कोष्ठक ,िवराम िचह्नों और पंिक्तयों के बीच का ःथान भी किवता को अथर् दे ता है
|उपयुक्त
र् पंिक्तयों में कोष्ठक से किवता में क्या िवशेष अथर् पैदा हआ
है ? समझाइए |
ु
उत्तर :- नई
किवता ूयोग धमीर् है |इसमें भाषा- िशल्प के ःतर पर हर नए ूयोग से अथर् की
अिभव्यिक्त की जाती है|ूाय: कोष्ठक अितिरक्त ज्ञान की सूचना दे ता है|यहाँ अभी गीला पड़ा है के
माध्यम से किव गीलेपन की ताजगी को ःपष्ट कर रहा है |ताजा गीलापन ःलेटी रं ग को अिधक
गहरा बना दे ता है जबिक सूखने के बाद राख हल्के ःलेटी रं ग की हो जाती है|
7. बादल राग
सूयक
र् ांत िऽपाठी िनराला
िनराला की यह किवता अनािमका में छह खंडों में ूकािशत
है ।यहां उसका छठा खंड िलया गया
है|आम आदमी के दःख
से ऽःत किव पिरवतर्न के िलए बांित रुपी बादल का आह्वान करता है
ु
|इस किवता में बादल बांित या िवप्लव का ूतीक है । किव िवप्लव के बादल को संबोिधत करते
हए
ु कहता है िक जन मन की आकांक्षाओं से भरी-तेरी नाव समीर रूपी सागर पर तैर रही है ।
अिःथर सुख पर दःख
की छाया तैरती िदखाई दे ती है । संसार के लोगों के हृदय दग्ध हैं (दःखी
ु
ु )।
उन पर िनदर् यी िवप्लव अथार्त ् बांित की माया फैली हई
ु है । बादलों के गजर्न से पृथ्वी के गभर् में
सोए अंकुर बाहर िनकल आते हैं अथार्त शोिषत वगर् सावधान हो जाता है और आशा भरी दृिष्ट से
बांित की ओर दे खने लगता है । उनकी आशा बांित पर ही िटकी है । बादलों की गजर्ना और
मूसलाधार वषार् में बड़े बड़े -पवर्त वृक्ष घबरा जाते हैं ।उनको उखड़कर िगर जाने का भय होता है
|बाित की हंु कार से पूँजीपित घबरा उठते हैं , वे िदल थाम कर रह जाते हैं । बांित को तो छोटे -
52 छोटे लोग बुलाते हैं । िजस ूकार छोटे छोटे पौधे हाथ िहलाकर-बादलों के आगमन का ःवागत
करते हैं वैसे ही शोिषत वगर् बांित के आगमन का ःवागत करता है ।
छायावादी किव िनराला साम्यवादी ूभाव से भी जुड़े हैं ।मुक्त छं द िहन्दी को उन्हीं की दे न
है ।शोिषत वगर् की समःयाओं को समाप्त करने के िलए बांित रूपी
बादल का आह्वान िकया गया
है ।
अथर्-महण-संबंधी ूश्न
किवता–
बादल राग
ितरती है समीर-सागर पर
अिःथर सुख पर दःख
की छाया –
ु
जग के दग्ध हृदय पर
िनदर् य िवप्लव की प्लािवत मायायह तेरी रण-तरी
भरी आकांक्षाओं से ,
घन भेरी Ðगजर्न से सजग सुप्त अंकुर
उर में पृथ्वी के, आशाओं से नवजीवन की ,ऊंचा कर िसर,
ताक रहे हैं ,ऐ िवप्लव के बादल!
िफर Ðिफर
बार Ðबार गजर्न
वषर्ण है मूसलधार ,
हृदय थाम लेता संसार ,
53 सुन- सुन घोर वळ हंु कार |
अशिन पात से शाियत शत-शत वीर ,
क्षत Ðिवक्षत हत अचल शरीर,
गगन- ःपशीर् ःपद्धार् धीर |
ूश्न१:- किवता में बादल िकस का ूतीक है ?और क्यों?
उत्तर :-बादलराग बांित का ूतीक है । इन दोनों के आगमन के उपरांत िवश्व हरा- भरा. समृद्ध
और ःवःथ हो जाता है ।
ूश्न २ :-सुख को अिःथर क्यों कहा गया है ?
उत्तर :-सुख सदै व बना नहीं रहता अतः उसे अिःथर कहा जाता है ।
ूश्न३ :-िवप्लवी बादल की युद्ध रूपी नौका की क्या- क्या िवशेषताएं हैं ?
उत्तर :-बादलों के अंदर आम आदमी की इच्छाएँ भरी हई
ु हैं ।िजस तरह से युद्र्ध नौका
में युद्ध
की साममी भरी होती है ।युद्ध की तरह बादल के आगमन पर रणभेरी बजती है । सामान्यजन की
आशाओं के अंकुर एक साथ फूट पड़ते हैं ।
ूश्न४ :-बादल के बरसने का गरीब एवं धनी वगर् से क्या संबंध जोड़ा गया है ?
उत्तर:-बादल के बरसने से गरीब वगर् आशा से भर जाता है
आशंका से भयभीत हो उठता है ।
54 एवं धनी वगर् अपने िवनाश की
सौंदयर्-बोध-संबंधी ूश्न
हँ सते हैं छोटे पौधे लघुभारशःय अपार ,
िहल िहल
िखल िखल
हाथ िहलाते
तुझे बुलाते।
िवप्लव रव से छोटे ही हैं शोभा पाते|
ूश्न १:- िनम्न िलिखत ूतीकों को ःपष्ट कीिजए– छोटे पौधे, सुप्त अंकुर
उत्तर :- छोटे पौधे- शोिषत वगर् , सुप्त अंकुर- आशाएं
,
ूश्न२:- Ôहँ सते हैं छोटे पौधेÕ-का ूतीकाथर् ःपष्ट कीिजए |
उत्तर :-ूसन्न िचत्त िनधर्न वगर् जो बांित की संभावना माऽ से िखल उठता है ।
ूश्न३:-Ôछोटे ही हैं शोभा पातेÕ में िनिहत लाक्षिणकता क्या है ?
उत्तर:-बचपन में मनुंय िनिश्चंत होता है । िनधर्न मनुंय उस बच्चे के समान है जो बांित के
समय भी िनभर्य होता है और अंतत: लाभािन्वत होता है ।
किवता–
बादल राग
िवषय-वःतु पर आधािरत ूश्नोत्तर
ूश्न१:- पूंजीपितयों की अट्टािलकाओं को आतंक भवन क्यों कहा गया है ?
55 उत्तर :-बादलों की गजर्ना और मूसलाधार वषार् में बड़े बड़े -पवर्त वृक्ष घबरा जाते हैं ।उनको उखड़कर
िगर जाने का भय होता है |उसी ूकार बाित की हंु कार से पूज
ँ ीपित घबरा उठते हैं , वे िदल थाम
कर रह जाते हैं ।उन्हें अपनी संपित्त एवं सत्ता के िछन जाने का भय होता है | उनकी अट्टािलकाएँ
मजबूती का ॅम उत्पन्न करती हैं पर वाःतव में वे अपने भवनों में आतंिकत होकर रहते हैं|
ूश्न२:- किव ने िकसान का जो शब्द-िचऽ िदया है उसे अपने शब्दों में िलिखए |
उत्तर :- िकसान के जीवन का रस शोषकों ने चूस िलया है ,आशा और उत्साह की संजीवनी
समाप्त हो चुकी है |शरीर से भी वह
दबर्
ु ल एवं खोखला हो चुका है | बांित का िबगुल उसके
हृदय में आशा का संचार करता है |वह िखलिखला कर बादल रूपी बांित का ःवागत करता है |
ूश्न३:- अशिन पात क्या है ?
उत्तर:- बादल की गजर्ना के साथ िबजली िगरने से
बड़े Ðबड़े वृक्ष जल कर राख हो जाते हैं |
उसी ूकार बांित की आंधी आने से शोषक, धनी वगर् की सत्ता समाप्त हो जाती है और वे खत्म
हो जाते हैं |
ूश्न४:- पृथ्वी में सोये अंकुर िकस आशा से ताक रहे हैं ?
उत्तर :- बादल के बरसने से बीज अंकुिरत हो लहलहाने लगते हैं | अत: बादल की गजर्न उनमें
आशाएँ उत्पन्न करती है |वे िसर ऊँचा कर बादल के आने की राह िनहारते हैं |ठीक उसी ूकार
िनधर्न व्यिक्त शोषक के अत्याचार से मुिक्त पाने और अपने जीवन की
खुशहाली की आशा में
बांित रूपी बादल की ूतीक्षा करते हैं |
ूश्न५:- रुद्ध कोष है , क्षुब्द्ध तोष Ðिकसके िलए कहा गया है और क्यों ?
उत्तर :- बांित होने पर पूंजीपित वगर् का धन िछन जाता है ,कोष िरक्त हो जाता है | उसके धन
की आमद समाप्त हो जाती है | उसका संतोष भी अब Ôबीते िदनों की बातÕ हो जाता है |
ूश्न६:- अिःथर सुख पर दःख
की छाया का भाव ःपष्ट कीिजए |
ु
उत्तर :- मानव-जीवन में सुख सदा बना नहीं रहता है ,उस पर दःख
की छाया सदा मंडराती रहती
ु
है|
56 ूश्न७:- बादल िकस का ूतीक है ?
उत्तर :- बादल इस किवता में बांित का ूतीक है |िजस ूकार बादल
ूकृ ित ,िकसान और आम
आदमी के जीवन में आनंद का उपहार ले कर आता है उसी ूकार बांित िनधर्न
जीवन में समानता का अिधकार व संपन्नता ले कर
शोिषत वगर् के
आता है
ूश्न८:- बादल को जीवन का पारावार क्यों कहा गया है ?
उत्तर :-
बांित रूपी बादल का आगमन जीवनदायी, सुखद होता है -पारावार अथार्त सागर| वह
जीवन में खुिशयों का खजाना लेकर आता
|सुख समृिद्ध का कारक बनकर
है |िनधर्न वगर् को समानता का अिधकार दे ता है
अत्याचार की अिग्न
57 से मुक्त करता है |
8
किवतावली
तुलसीदास
सार
ौीरामजी को समिपर्त मन्थ ौीरामचिरतमानस उत्तर भारत मे बड़े भिक्तभाव से पढ़ा जाता है ।
लआमण -मूच्छार् और राम का िवलाप
रावण पुऽ मेघनाद द्वारा शिक्त बाण से मूिछर् त हए
ु लआमण को दे खकर राम व्याकुल हो जाते
हैं ।सुषेण वैद्य ने संजीवनी बूटी लाने के िलए हनुमान को िहमालय पवर्त पर भेजा।आधी रात
व्यतीत होने पर जब हनुमान नहीं आए,तब राम ने अपने छोटे भाई लआमण को उठाकर हृदय से
लगा िलया और साधारण मनुंय की भाँित िवलाप करने लगे।राम बोले तुम मुझे ! हे भाई......
कभी दखी
नहीं दे ख सकते थे।तुम्हारा ःवभाव सदा से ही कोमल
ु
था।तुमने मेरे िलए माता िपता
सदीर्को भी छोड़ िदया और मेरे साथ वन में,गमीर् और िविभन्न ूकार की िवपरीत पिरिःथितयों
को भी सहा|जैसे पंख िबना पक्षी,मिण िबना सपर् और सूड
ँ िबना ौेष्ठ हाथी अत्यंत दीन हो जाते
हैं ,हे भाई!यिद मैं जीिवत रहता हँू तो मेरी दशा भी वैसी ही हो जाएगी।
मैं अपनी पत्नी के िलए अपने िूय भाई को खोकर कौन सा मुह
ँ लेकर अयोध्या जाऊँगा।इस
बदनामी को भले ही सह लेता िक राम कायर है और अपनी पत्नी को खो बैठा। स्तर्ी की हािन
िवशेष क्षित नहीं है ,परन्तु भाई को खोना अपूरणीय क्षित है ।
‘रामचिरतमानस’ के ‘लंका कांड’ से गृही लआमण को शिक्त बाण लगने का ूसंग किव की मािमर्क
ःथलों की पहचान का एक ौेष्ठ नमूना है । भाई के शोक में िवगिलत राम का िवलाप धीरे धीरे ूलाप में बदल जाता है , िजसमें लआमण के ूित राम के अंतर में िछपे ूेम के कई कोण सहसा
अनावृत हो जाते हैं ।यह ूसंग ईश्वर राम में मानव सुलभ गुणों का समन्वय कर दे ता है |
हनुमान का संजीवनी लेकर आ जाना करुण रस में वीर रस का उदय हो जाने के समान है|
िवनय पिऽका एक अन्य महत्त्वपूणर् तुलसीदासकृ त काव्य है ।
58 किवत्त और सवैया
सार
इस शीषर्क के अंतगर्त दो किवत्त और एक सवैया संकिलत हैं । ‘किवतावली’ से अवतिरत इन
किवत्तों में किव तुलसी का िविवध िवषमताओं से मःत किलकालतुलसी का युगीन यथाथर् है ,
िजसमें वे कृ पालु ूभु राम व रामराज्य का ःवप्न रचते हैं । युग और उसमें अपने जीवन का न
िसफर् उन्हें गहरा बोध है , बिल्क उसकी अिभव्यिक्त में भी वे अपने समकालीन किवयों से आगे
हैं । यहाँ पाठ में ूःतुत ‘किवतावली’ के छं द इसके ूमाण ःवरूप हैं । पहले छं द-”िकसवी िकसान
....“ में उन्होंने िदखलाया है िक संसार के अच्छे बुरे समःत-लीला ूपंचों का आधार-‘पेट की
आग’का गहन यथाथर् है ; िजसका समाधान वे
राम की भिक्त में दे खते हैं । दिरिजन की व्यथा
दरू करने के िलए राम रूपी घनँयाम का आह्वान िकया गया है । पेट की आग बुझाने के िलए
राम रूपी वषार् का जल अिनवायर् है ।इसके िलए अनैितक कायर् करने की आवँयकता नहीं है ।Ô इस
ूकार, उनकी राम भिक्त पेट की आग बुझाने वाली यानी जीवन के यथाथर् संकटों का समाधान
करने वाली है ; न िक केवल आध्याित्मक मुिक्त दे ने वाली| गरीबी की पीड़ा रावण के समान
दखदायी
हो गई है ।
ु
तीसरे छं द )”धूत कहौ...“) में भिक्त की गहनता और सघनता में उपजे भक्तहृदय के आत्मिवश्वास
का सजीव िचऽण है , िजससे समाज में व्याप्त जातपाँत और
दा
दरामहों
के ितरःकार का साहस ु
होता है । इस ूकारपैभिक्त की रचनात्मक भूिमका का संकेत यहाँ है , जो आज के
मूलक युग में अिधक ूासंिगक है |
अथर्-महण-संबंधी ूश्न
उहाँ राम लिछमनिहं िनहारी। बोले बचन मनुज अनुसारी॥
अधर् राित गइ किप निहं आयउ। राम उठाइ अनुज उर लायउ॥
सकहु न दिखत
दे िख मोिह काऊ। बंधु सदा तव मृदल
ु
ु सुभाऊ॥
मम िहत लािग तजेहु िपतु माता। सहे हु िबिपन िहम आतप बाता॥
सो अनुराग कहाँ अब भाई। उठहु न सुिन मम बच िबकलाई॥
जौं जनतेउँ बन बंधु िबछोह।ू िपता बचन मनतेउँ निहं ओह॥
ू
59 भेदभाव
सुत िबत नािर भवन पिरवारा। होिहं जािहं जग बारिहं बारा॥
अस िबचािर िजयँ जागहु ताता। िमलइ न जगत सहोदर ॅाता॥
जथा पंख िबनु खग अित दीना। मिन िबनु फिन किरबर कर हीना॥
अस मम िजवन बंधु िबनु तोही। जौं जड़ दै व िजआवै मोही॥
जैहउँ अवध कवन मुहु लाई। नािर हे तु िूय भाइ गँवाई॥
बरु अपजस सहतेउँ जग माहीं। नािर हािन िबसेष छित नाहीं॥
ु कठोर उर मोरा॥
अब अपलोकु सोकु सुत तोरा। सिहिह िनठर
िनज जननी के एक कुमारा। तात तासु तुम्ह ूान अधारा॥
सौंपेिस मोिह तुम्हिह गिह पानी। सब िबिध सुखद परम िहत जानी॥
उतरु काह दै हउँ तेिह जाई। उिठ िकन मोिह िसखावहु भाई॥
ूश्न१:-Ôबोले बचन मनुज अनुसारीÕ- का तात्पयर् क्या है
?
उत्तर :- भाई के शोक में िवगिलत राम का िवलाप धीरे - धीरे ूलाप में बदल जाता है -, िजसमें
लआमण के ूित राम के अंतर में िछपे ूेम के कई कोण सहसा अनावृत हो जाते हैं । यह ूसंग
ईश्वर राम में मानव सुलभ गुणों का समन्वय कर दे ता है| वे मनुंय की भांित िवचिलत हो कर
ऐसे वचन कहते हैं जो मानवीय ूकृ ित को ही शोभा दे ते हैं |
ूश्न२:- राम ने लआमण के िकन गुणों का वणर्न िकया है ?
उत्तर :-राम ने लआमण के इन गुणों का वणर्न िकया है •
लआमण राम से बहत
ु ःनेह करते हैं |
•
उन्होंने भाई के िलए अपने माता Ðिपता का भी त्याग कर िदया |
•
वे वन में वषार् ,िहम, धूप आिद कष्टों को सहन कर रहे हैं |
•
उनका ःवभाव
बहत
को नहीं दे ख सकते |
ु है |वे भाई के दःख
ु
ु मृदल
ूश्न३:- राम के अनुसार कौन सी वःतुओं की हािन बड़ी हािन नहीं है और क्यों ?
60 उत्तर :-राम के अनुसार धन ,पुऽ एवं नारी की हािन बड़ी हािन नहीं है क्योंिक ये
पर पुन: ूाप्त िकये जा सकते हैं पर एक बार सगे
सब खो जाने
भाई के खो जाने पर उसे पुन: ूाप्त नहीं
िकया जा सकता |
ूश्न४:- पंख के िबना पक्षी और सूड
ं
के िबना हाथी की क्या दशा होती है काव्य ूसंग में इनका
उल्लेख क्यों िकया गया है ?
उत्तर :-
राम िवलाप करते हए
ु अपनी भावी िःथित का वणर्न कर रहे हैं
िबना पक्षी और सूड
ं
पंख के
के िबना हाथी पीिड़त हो जाता है ,उनका अिःतत्व नगण्य हो जाता है
वैसा ही असहनीय कष्ट राम को लआमण के न होने से होगा |
61 िक जैसे
सौंदयर्-बोध-संबंधी ूश्न
ूश्न१:- काव्यांश की भाषा सौंदयर् संबंधी दो िवशेषताओं का उल्ल्लेख कीिजए|
उत्तर:- १रस -करुण रस
२
अलंकार -
उत्ूेक्षा अलंकार–
मनु करुणा मंह
बीर रस।
जागा िनिसचर दे िखअ कैसा।मानहँु काल दे ह धिर बैसा।
दृष्टांत अलंकार - जथा पंख िबन खग अित दीना।मिन िबनु फिन किरबर कर हीना।
अस मन िजवन बंधु िबन तोही।जो जड़ दै व िजआवै मोही।
िवरोधाभास अलंकार -बहिबिध
सोचत सोच िबमोचन।
ु
ूश्न२:- काव्यांश की भाषा का नाम िलिखए |
उत्तर :- अवधी भाषा
ूश्न३:- काव्यांश में ूयुक्त छं द कौन Ðसा है ?
उत्तर:- १६,१६ माऽाओं का सम मािऽक चौपाई छं द |
सौंदयर्-बोध-संबंधी ूश्न
िकसबी, िकसान-कुल ,बिनक, िभखारी ,भाट,
चाकर ,चपल नट ,चोर, चार ,चेटकी|
पेटको पढ्त,गुन
गढ़त, चढ़त िगिर,
अटत गहन Ðगन अहन अखेट्की|
ऊंचे Ðनीचे करम ,धरम Ðअधरम किर,
पेट ही को पचत, बचत बेटा Ðबेटकी |
ÔतुलसीÕ बुझाई एक राम घनःयाम ही तें ,
62 आिग बड़वािगतें बड़ी है आिग पेटकी|
ूश्न१:- किवतावली िकस भाषा में िलखी गई है ?
उत्तर :-
ॄज भाषा
ूश्न२:- किवतावली में ूयुक्त छं द एवं रस को ःपष्ट कीिजए |
उत्तर :- इस पद में
31. 31 वणोर्ं का चार चरणों वाला समविणर्क किवत्त
छं द है िजसमें 16
एवं 15 वणोर्ं पर िवराम होता है ।
ूश्न३:- किवत्त में ूयुक्त अलंकारों को छांट कर िलिखए
१. अनुूास अंलकार–
िकसबी, िकसान-कुल ,बिनक, िभखारी ,भाट,
चाकर ,चपल नट ,चोर, चार ,चेटकी|
२. रूपक अलंकार– राम– घनँयाम
३. अितशयोिक्त अलंकार– आिग बड़वािगतें बिड़ है आग पेट की।
लआमण- मूच्छार् और राम का िवलाप
िवषय-वःतु पर आधािरत ूश्नोत्तर
ूश्न १:-Ôतव ूताप उर रािख ूभु में िकसके ूताप का उल्लेख िकया गया है ?Õऔर क्यों ?
उत्तर :-इन पँिक्तयों में भरत के ूताप का उल्लेख िकया गया है । हनुमानजी उनके ूताप का
ःमरण करते हए
ु अयोध्या के ऊपर से उड़ते हए
ु संजीवनी ले कर लंका की ओर चले जा रहे हैं ।
ूश्न२:- राम िवलाप में लआमण की कौन सी िवशेषताएँ उद्घिटत हई
ु हैं ?
उत्तर :-लआमण का ॅातृ ूेम. त्यागमय जीवन इन पँिक्तयों के माध्यम से उदघािटत हआ
है ।
ु
ूश्न३:- बोले वचन मनुज अनुसारी से किव का क्या तात्पयर् है ?
63 उत्तर :-भगवान राम एक साधारण मनुंय की तरह िवलाप कर रहे हैं िकसी अवतारी मनुंय की
तरह नहीं। ॅातृ ूेम का िचऽण िकया गया है ।तुलसीदास की मानवीय भावों पर सशक्त पकड़
है ।दै वीय व्यिक्तत्व का लीला रूप ईश्वर राम को मानवीय भावों से समिन्वत कर दे ता है ।
ूश्न४:- भाई के ूित राम के ूेम की ूगाढ़ता उनके िकन िवचारों से व्यक्त हई
ु है ?
उत्तर :-
जथा पंख िबन खग अित दीना।
मिन िबनु फिन किरबर कर हीना।
अस मम िजवन बंधु िबनु तोही।
जो जड़ दै व िजआवै मोही।
ूश्न५:- Ôबहिविध
सोचत सोचिवमोचनÕ का िवरोधाभास ःपष्ट कीिजए।
ु
उत्तर :-दीनजन को शोक से मुक्त करने वाले भगवान राम ःवयं बहत
ु ूकार से सोच में पड़कर
दखी
हो रहे हैं ।
ु
ूश्न६:- हनुमान का आगमन करुणा में वीर रस का आना िकस ूकार कहा जा सकता है ?
उत्तर :-रुदन करते वानर समाज में हनुमान उत्साह का संचार करने वाले वीर रस के रूप में
आ गए। करुणा की नदी हनुमान द्वारा संजीवनी ले आने पर मंगलमयी हो उठती है ।
किवत्त और सवैया
िवषय-वःतु पर आधािरत ूश्नोत्तर
ूश्न१:- पेट की भूख शांत करने के िलए लोग क्या क्या करते हैं ?
उत्तर :-पेट की आग बुझाने के िलए लोग अनैितक कायर् करते हैं ।
ूश्न२:- तुलसीदास की दृिष्ट में सांसािरक दखों
से िनवृित्त का सवोर्त्तम उपाय क्या है ?
ु
उत्तर :- पेट की आग बुझाने के िलए राम कृ पा रूपी वषार् का जल अिनवायर् है ।इसके िलए
अनैितक कायर् करने की आवँयकता नहीं है ।
ूश्न३:- तुलसी के युग की समःयाओं का िचऽण कीिजए।
64 उत्तर :- तुलसी के युग में ूाकृ ितक और ूशासिनक वैषम्य के चलते उत्पन्न पीडा. दिरिजन के
िलए रावण के समान दखदायी
हो गई है ।
ु
ूश्न४:- तुलसीदास की भिक्त का कौन सा ःवरूप ूःतुत किवत्तों में अिभव्यक्त हआ
है ?
ु
उत्तर :- तुलसीदास की भिक्त का दाःय भाव ःवरूप ूःतुत किवत्तों में अिभव्यक्त हआ
है ।
ु
9
रूबाइयाँ
िफ़राक गोरखपुरी
मूल नाम– रघुपित सहाय िफ़राक
उदर् ू शायरी की िरवायत के िवपरीत िफराक गोरखपुरी के सािहत्य में लोक जीवन एवं ूकृ ित
की झलक िमलती है । सामािजक संवेदना वैयिक्तक अनुभिू त बन कर उनकी रचनाओं में व्यक्त हई
ु
है ।जीवन का कठोर यथाथर् उनकी रचनाओं में ःथान पाता है ।उन्होंने लोक भाषा के ूतीकों का
ूयोग िकया है । लाक्षिणक ूयोग उनकी भाषा की िवशेषता है ।िफ़राक की रुबाईयों में घरे लू िहं दी
का रूप िदखता है |
रुबाई उदर् ू और फ़ारसी का एक छं द या लेखन शैली है िजसमें चार पंिक्तयाँ होती हैं |इसकी पहली
,दसरी
और चौथी पंिक्त में तुक (कािफ़या)िमलाया जाता है तथा तीसरी पंिक्त ःवच्छं द होती है |
ू
वो रूपवती मुखड़े पे इक नमर् दमक
बच्चे के घरोंदे में जलाती है िदए।
रक्षाबंधन की सुबह रस की पुतली
िबजली की तरह चमक रहे हैं लच्छे
भाई के है बाँधती चमकती राखी। - जैसे ूयोग उनकी भाषा की सशक्तता के नमूने के तौर पर
दे खे जा सकते हैं |
सार
रूबाइयाँ
65 रक्षाबंधन एक मीठा बंधन है । रक्षाबंधन के कच्चे धागों पर िबजली के लच्छे हैं । सावन में
रक्षाबंधन आता है । सावन का जो संबंध झीनी घटा से है , घटा का जो संबंध िबजली से है वही
संबंध भाई का बहन से होता है ।
गज़ल
पाठ में िफराक की एक गज़ल भी शािमल है । रूबाइयों की तरह ही िफराक की गजलों में भी
िहं दी समाज और उदर् ू शायरी की परं परा भरपूर है । इसका अद्भत
ु नमूना है यह गज़ल। यह गज़ल
कुछ इस तरह बोलती है िक िजसमें
ददर् भी है , एक शायर की ठसक भी है और साथ ही है
काव्य-िशल्प की वह ऊँचाई जो गज़ल की िवशेषता मानी जाती है ।
अथर्-महण-संबंधी ूश्न
ु
आँगन में िलए चाँद के टकड़े
को खड़ी
हाथों पे झुलाती है उसे गोद-भरी
रह-रह के हवा में जो लोका दे ती है
गूँज उठती है िखलिखलाते बच्चे की हँ सी
ु Õका ूयोग िकसके िलए हआ
ूश्न१:- Ôचाँद के टकड़े
है ? और क्यों ?
ु
ु
उत्तर :-बच्चे को चाँद का टकड़ा
कहा गया है जो माँ
के िलए बहत
ु प्यारा होता है ।
ूश्न२:- गोद-भरी ूयोग की िवशेषता को ःपष्ट कीिजए |
उत्तर :- गोद-भरी शब्द-ूयोग माँ के वात्सल्यपूण,र् आनंिदत उत्साह को ूकट करता है |यह
अत्यंत सुद
ं र दृँय िबंब है | सूनी गोद के िवपरीत गोद का भरना माँ के िलए असीम सौभाग्य
का सूचक है |इसी सौभाग्य का सूआम अहसास माँ को तृिप्त दे रहा है|
ूश्न३:- लोका दे ना िकसे कहते हैं ?
उत्तर :- जब माँ
बच्चे को बाहों में लेकर हवा में उछालती है इसे लोका दे ना कहते हैं|छोटे
बच्चों को यह खेल बहत
ु अच्छा लगता है
ूश्न४:- बच्चा माँ की गोद में कैसी ूितिबया करता है ?
66 उत्तर :- हवा में उछालने ( लोका दे न)े से बच्चा माँ का वात्सल्य पाकर ूसन्न होता है और
िखलिखला कर हँ स पड़ता है ।बच्चे की िकलकािरयाँ माँ के आनंद को दगना
कर दे ती हैं|
ु
67 सौंदयर्-बोध-संबंधी ूश्न
नहला के छलके-छलके िनमर्ल जल से
उलझे हए
ु ं में कंघी करके
ु गेसओ
िकस प्यार से दे खता है बच्चा मुह
ँ को
जब घुटिनयों में लेके है िपन्हाती कपड़े
ूश्न१:- ूःतुत पंिक्तयों के भाव सौंदयर् को ःपष्ट कीिजए |
उत्तर :- माँ ने अपने बच्चे को िनमर्ल जल से नहलाया उसके उलझे बालों में कंघी
की |माँ के
ःपशर् एवं नहाने के आनंद से बच्चा ूसन्न हो कर बड़े ूेम से माँ को िनहारता है| ूितिदन की
एक ःवाभािवक िबया से कैसे माँ-बच्चे का ूेम िवकिसत होता है और ूगाढ़ होता चला जाता है
इस भाव को इस रुबाई में बड़ी सूआमता के साथ ूःतुत िकया गया है |
ूश्न२:- काव्यांश में आए िबंबों को ःपष्ट कीिजए |
उत्तर :-१ नहला के छलके-छलके िनमर्ल जल से - इस ूयोग द्वारा किव ने बालक की िनमर्लता
एवं पिवऽता को जल की िनमर्लता के माध्यम से अंिकत िकया है | छलकना शब्द जल की
ताजा बूंदों का बालक के शरीर पर छलछलाने का सुद
ं र दृँय िबंब ूःतुत करता है |
२ Ôघुटिनयों में लेके है िपन्हाती कपड़ेÕ- इस ूयोग में माँ की बच्चे के ूित सावधानी ,चंचल
बच्चे को चोट पहँु चाए िबना उसे कपड़े पहनाने से
माँ के मातृत्व की कुशलता िबंिबतहोती है |
है | माँ
३ िकस प्यार से दे खता है बच्चा मुह
ँ को - पंिक्त में माँ Ðबच्चे का वात्सल्य िबंिबत हआ
ु
से प्यार Ðदलार
,ःपशर् Ðसुख, नहलाए जाने के आनंद को अनुभव करते हए
ु
ु बच्चा माँ को प्यार
भरी नजरों से दे ख कर उस सुख की अिभव्यिक्त कर रहा है |यह सूआम भाव अत्यंत मनोरम बन
पड़ा है |संपण
ू र् रुबाई में दृँय िबंब है|
ूश्न ३:-काव्यांश के शब्द-ूयोग पर िटप्पणी िलिखए |
उत्तर :-गेसु Ðउदर् ू शब्दों का ूयोग
घुटिनयों ,िपन्हाती Ð दे शज शब्दों के माध्यम से कोमलता की अिभव्यिक्त
68 छलके Ðछलके Ðशब्द की पुनरावृित्त से अभी Ðअभी नहलाए गए बच्चे के गीले शरीर का िबंब
आिद िवलक्षण ूयोग रुबाइयों को िविशष्ट बना दे ते हैं |िहं दी,उदर् ू और लोकभाषा के अनूठे
गठबंधनकी झलक, िजसे गांधीजी िहं दःतानी
के रूप में पल्लिवत करना चाहते थे,दे खने को
ु
िमलती है |
िवषय-वःतु पर आधािरत ूश्नोत्तर
रूबाइयाँ
ूश्न १ :-काव्य में ूयुक्त िबंबों का वणर्न अपने शब्दों में कीिजए?
उत्तर :•
दृँय िबंब :-बच्चे को गोद में लेना , हवा में उछालना ,ःनान कराना ,घुटनों में लेकर
कपड़े पहनाना|
•
ौव्यिबंब :- बच्चे का िखलिखला कर हँ स पड़ना।
•
ःपशर् िबंब :-बच्चे को ःनान कराते हए
ु ःपशर् करना |
ु
ूश्न २:- आँगन में ठनक
रहा है िज़दयाया है ,बालक तो हई चाँद पै ललचाया हैÕ- में बालक की
कौन सी िवशेषता अिभव्यक्त हई
ु है ?
उत्तर :- इन पंिक्तयों में बालक की हठ करने की िवशेषता अिभव्यक्त हई
ु है । बच्चे जब िजद पर
आ जाते हैं तो अपनी इच्छा पूरी करवाने
के िलए नाना ूकार की हरकतें िकया करते हैं|
ु
िज़दयाया शब्द लोक भाषा का िवलक्षण ूयोग है इसमें बच्चे का ठनकना
, तुनकना ,पाँव
पटकना, रोना आिद सभी िबयाएँ शािमल हैं |
ूश्न ३ लच्छे िकसे कहा गया है इनका संबंध िकस त्यौहार से है ?
उत्तर :- राखी के चमकीले तारों को लच्छे कहा गया है । रक्षाबंधन के कच्चे धागों पर िबजली के
लच्छे हैं । सावन में रक्षाबंधन आता है । सावन का जो संबंध झीनी घटा से है , घटा का जो संबंध
िबजली से है वही संबंध भाई का बहन से होता है|सावन में िबजली की चमक की तरह राखी के
चमकीले धागों की सुद
ं रता दे खते ही बनती है|
69 अथर्-महण-संबंधी ूश्न
गज़ल
िफ़राक गोरखपुरी
नौरस गुच
ं े पंखिड़यों की नाज़ुक िगरहें खोले हैं
या उड़ जाने को रं गो- बू गुलशन में पर तौले हैं |
ूश्न१:- ÔनौरसÕ िवशेषण द्वारा किव िकस अथर् की व्यंजना करना चाहता है ?
उत्तर :नौरस अथार्त नया रस ! गुच
ं े अथार्त किलयों में नया Ðनया रस भर आया है |
ूश्न२:- पंखिड़यों की नाज़ुक िगरहें खोलने का क्या अिभूाय है ?
उत्तर :- रस के भर जाने से किलयाँ िवकिसत हो रही हैं |धीरे -धीरे उनकी पंखुिड़याँ अपनी बंद
गाँठें खोल रही हैं | किव के शब्दों में नवरस ही उनकी बंद गाँठें खोल रहा है|
ूश्न३:- Ôरं गो- बू गुलशन में पर तौले हैंÕ Ð का अथर् ःपष्ट कीिजए|
उत्तर :-रं ग और सुगध
ं दो पक्षी हैं जो किलयों में बंद हैं तथा उड़ जाने के िलए अपने पंख
फड़फड़ा रहे हैं |यह िःथित किलयों के फूल बन जाने से पूवर् की है जो फूल बन जाने की ूतीक्षा
में हैं |Õपर तौलनाÕ एक मुहावरा है जो उड़ान की क्षमता आँकने के िलए ूयोग िकया जाता है |
ूश्न४:- इस शेर का भाव-सौंदयर् व्यक्त कीिजए|
उत्तर :-किलयों की नई-नई पंखिु ड़याँ िखलने लगी हैं उनमें से रस मानो
टपकना ही चाहता है |
वय:संिध(िकशोरी)नाियका के ूःफुिटत होते सौंदयर् का ूतीकात्मक िचऽण अत्यंत सुद
ं र बन पड़ा
है |
सौंदयर्-बोध-संबंधी ूश्न
हम हों या िकःमत हो हमारी दोनों को इक ही काम िमला
िकःमत हम को रो लेवे हैं
हम िकःमत को रो ले हैं|
जो मुझको बदनाम करे हैं काश वे इतना सोच सकें
70 मेरा पदार् खोले हैं या अपना पदार् खोले हैं |
ूश्न१:- इन
शेरों की भाषा संबंधी िवशेषताएँ ःपष्ट कीिजए |
उत्तर:- १. मुहावरों का ूयोग Ðिकःमत का रोना Ðिनराशा का ूतीक
२.सरल अिभव्यिक्त ,भाषा में ूवाहमयता है ,िकःमत और परदा शब्दों की पुनरावृित्तयाँ
मोहक हैं|
३. िहं दी का घरे लू रूप
ूश्न२:- Ôमेरा परदा खोले हैं या अपना परदा खोले हैं Ô- की भािषक िवशेषता िलिखए |
उत्तर :-मुहावरे के ूयोग द्वारा व्यंजनात्मक अिभव्यिक्त | परदा
खोलना Ð भेद खोलना ,सच्चाई
बयान करना|
ूश्न३:-Ôहमहों या िकःमत हो हमारी Ô Ðूयोग की िवशेषता बताइए |
उत्तर :- हम और िकःमत दोनों शब्द एक ही व्यिक्त अथार्त िफ़राक के िलए ूयुक्त हैं |हम और
िकःमत में अभेद है यही िवशेषता है |
िफ़राक गोरखपुरी
िवषय-वःतु पर आधािरत ूश्नोत्तर
ूश्न१:- तारे आँखें झपकावें हैं Ðका तात्पयर् ःपष्ट कीिजए |
उत्तर:- रािऽ का सन्नाटा भी कुछ कह रहा है ।इसिलए तारे पलकें झपका रहे हैं । िवयोग की
िःथित में ूकृ ित भी संवाद करती ूतीत होती है |
ूश्न२:- Ôहम हों या िकःमत हो हमारीÕ में िकस भाव की अिभव्यिक्त हई
ु है ?
उत्तर:- जीवन की िवडं बना, िकःमत को रोना-मुहावरे के ूयोग से, सटीक अिभव्यिक्त ूाप्त करती
है ।किव जीवन से संतुष्ट नहीं है | भाग्य
से िशकायत का भाव
इन पंिक्तयों में झलकता है |
ूश्न३:- ूेम के िकस िनयम की अिभव्यिक्त किव ने की है ?
उत्तर :-ईश्वर की ूािप्त
के शब्दों में ,
सवर्ःव लुटा दे ने पर होती है । ूेम के संसार का भी यही िनयम है|किव
िफ़तरत का कायम है तवाज़ुन आलमे- हःनो
Ðइँक में भी
ु
71 उसको उतना ही पाते हैं खुद को िजतना खो ले हैं |
१ भाव Ðसाम्य:- कबीर -Ôसीस उतारे भुई धरे तब िमिलहै करतार।Ô-अथार्त -ःवयं को खो कर ही
ूेम ूािप्त की जा सकती है ।
२ भाव साम्य- कबीर Ðिजन ढँू ढा ितन पाइयाँ ,गहरे पानी पैिठ|
मैं बपुरा बूडन डरा ,रहा िकनारे बैिठ|
ूश्न४:- शराब की महिफल में शराबी को दे र रात क्या बात याद आती है ?
उत्तर :- शराब की महिफल में शराबी
को दे र रात याद आती है िक आसमान में मनुंय के पापों
का लेखा-जोखा होता है । जैसे आधी रात के समय फिरँते लोगों के पापों के अध्याय खोलते हैं
वैसे ही रात के समय शराब पीते हए
ु शायर को महबूबा की याद हो आती है मानो महबूबा
फिरँतों की तरह पाप ःथल के आस पास ही है ।
ूश्न५:- सदके िफ़राक–––इन पंिक्तयों का अथर् ःपष्ट कीिजए |
उत्तर :-सदके िफ़राक–––इन पंिक्तयों में िफ़राक कहते हैं िक उनकी शायरी में मीर की शायरी की
उत्कृ ष्टता ध्विनत हो रही है ।
ूश्न६:- पंखुिड़यों की नाज़ुक िगरह खोलना क्या है ?
उत्तर :-पँखुिड़यों की नाजुक िगरह खोलना उनका धीरे -धीरे िवकिसत होना है ।
वय:संिध(िकशोरी)नाियका के ूःफुिटत होते सौंदयर् की ओर संकेत है |
ूश्न७:-‘यों उड़ जाने को रं गो बू गुलशन में पर तौले है ’ भाव ःपष्ट कीिजए।
उत्तर :-किलयों की सुवास उड़ने के िलए मानो पर तौल रही हो।अथार्त खुशबू का झोंका रह–रह
कर उठता है ।
ूश्न८:- किव द्वारा विणर्त रािऽ के दृँय
का वणर्न अपने शब्दों में कीिजए।
उत्तर :- रािऽ का सन्नाटा भी कुछ कह रहा है ।इसिलए तारे पलकें झपका रहे हैं ।लगता है िक
ूकृ ित का कण-कण कुछ कह
रहा है |
ूश्न९:- किव अपने वैयिक्तक अनुभव िकन पँिक्तयों में व्यक्त कर रहा है ◌?
ै
72 जीवन की िवडं बना,-Ôिकःमत को रोनाÔ मुहावरे के ूयोग से, सटीक अिभव्यिक्त ूाप्त करती है ।
हम हों या िकःमत हो हमारी दोनों को इक ही काम िमला
िकःमत हम को रो लेवे हैं
हम िकःमत को रो ले हैं|
ूश्न१०:- शायर ने दिनया
के िकस दःतूर का वणर्न िकया है ?
ु
उत्तर :-शायर ने दिनया
के इस दःतूर का वणर्न िकया है िक लोग दसरों
को बदनाम करते हैं
ु
ू
परं तु वे नहीं जानते िक इस तरह वे अपनी दष्ट
ु ूकृ ित को ही उद्घािटत करते हैं ।
ूश्न११:- ूेम की िफ़तरत किव ने िकन शब्दों में अिभव्यक्त की है ?
ःवयं को खो कर ही ूेम की ूािप्त की जा सकती है । ईश्वर की ूािप्त सवर्ःव लुटा दे ने पर होती
है । ूेम के संसार का भी यही िनयम है ।
ूश्न१२:- िफ़राक गोरखपुरी िकस भाषा के किव हैं ◌?
ै
उत्तर :- उदर् ू भाषा ।
10
छोटा मेरा खेत
उमाशंकर जोशी
(गुजराती किव)।
उमाशंकर जोशी बीसवीं सदी के गुजराती के मूधन्
र् य किव संःकृ त वाङ्मय के िवद्वान हैं ।उन्होंने
गुजराती किवता को ूकृ ित से जोड़ा।आम आदमी के जीवन की झलक उनकी रचनाओं में िमलती
है ।
छोटा मेरा खेत
सार
खेती के रूपक द्वारा काव्य रचना– ूिबया को ःपष्ट िकया गया हे ।काव्य कृ ित की रचना बीज–
वपन से लेकर पौधे के पुिंपत होने के िविभन्न चरणों से गुजरती है ।अंतर केवल इतना है िक
73 किव कमर् की फसल कालजयी, शाश्वत होती है ।उसका रस-क्षरण अक्षय होता है ।कागज का पन्ना,
िजस पर रचना शब्दबद्ध होती है , किव को एक चौकोर खेत की तरह लगता है । इस खेत में
िकसी अँधड़ (आशय भावनात्मक आँधी से होगा) के ूभाव से िकसी क्षण एक बीज बोया जाता
है । यह बीज-रचना िवचार और अिभव्यिक्त का हो सकता है । यह मूल रूप कल्पना का सहारा
लेकर िवकिसत होता है और ूिबया में ःवयं िवगिलत हो जाता है । उससे शब्दों के अंकुर
िनकलते हैं और अंततः कृ ित एक पूणर् ःवरूप महण करती है , जो कृ िष-कमर् के िलहाज से
पल्लिवत -पुिंपत होने की िःथित है । सािहित्यक कृ ित से जो अलौिकक रस-धारा फूटती है , वह
क्षण में होने वाली रोपाई का ही पिरणाम है पर यह रस-धारा अनंत काल तक चलने वाली कटाई
है |
बगुलों के पंख
सार
बगुलों के पंख किवता एक चाक्षुष िबंब
की किवता है । सौंदयर् का अपेिक्षत ूभाव उत्पन्न करने
के िलए किवयों ने कई युिक्तयाँ अपनाई हैं , िजसमें से सबसे ूचिलत युिक्त है -सौंदयर् के व्यौरों के
िचऽात्मक वणर्न के साथ अपने मन पर पड़ने वाले उसके ूभाव का वणर्न और आत्मगत के
संयोग की यह युिक्त पाठक को उस मूल सौंदयर् के काफी िनकट ले जाती है । जोशी जी की इस
किवता में ऐसा ही है । किव काले बादलों से भरे आकाश में पंिक्त बनाकर उड़ते सफेद बगुलों को
दे खता है । वे कजरारे बादलों में अटका-सा रह जाता है । वह इस माया से अपने को बचाने की
गुहार लगाता हैं । क्या यह सौंदयर् से बाँधने और िवंधने की चरम िःथित को व्यक्त करने का एक
तरीका है ।
ूकृ ित का ःवतंऽ (आलंबन गत ) िचऽण
आधुिनक किवता की िवशेषता है ।िचऽात्मक वणर्न
द्वारा किव ने एक ओर काले बादलों पर उड़ती बगुलों की श्वेत पंिक्त का िचऽ अंिकत िकया है तो
दसरी
ओर इस अूितम दृँय के हृदय पर पड़ने वाले ूभाव को िचिऽत िकया है ।मंऽ मुग्ध किव
ू
इस दृँय के ूभाव से आत्म िवःमृित की िःथित तक पहँु च जाता है ।िवषय एवं िवषयीगत
सौन्दयर् के दोनों रूप किवता में उद्घािटत हए
ु हैं ।
अथर्-महण-संबंधी ूश्न
छोटा मेरा खेत चौकोना
74 कागज़ का एक पन्ना ,कोई अंधड़ कहीं से आया
क्षण का बीज वहाँ बोया गया|
कल्पना के रसायनों को पी
बीज गल गया िन:शेष ;शब्द के अंकुर फूटे ,
पल्लव Ðपुंपों से निमत हआ
िवशेष |
ु
ूश्न १‘छोटा मेरा खेत’ िकसका ूतीक है और क्यों?
उत्तर :- ूश्न२ ‘छोटा मेरा खेत’ काग़ज के उस पन्ने का ूतीक है िजस पर किव अपनी किवता
िलखता है ।
ूश्न २ किव खेत में कौन–सा बीज बोता है ?
उत्तर :- किव खेत में अपनी कल्पना का बीज बोता है ?
ूश्न ३ किव की कल्पना से कौन से पल्लव
अंकुिरत होते हैं ?
उत्तर :- किव की कल्पना से शब्द के पल्लव अंकुिरत होते हैं ?
ूश्न ४ उपयुक्त
र् पद का भाव-सौंदयर् ःपष्ट कीिजए |
उत्तर :- खेती के रूपक द्वारा काव्य-रचना–ूिबया को ःपष्ट िकया गया हे ।काव्य कृ ित की रचना
बीज– वपन से लेकर पौधे के पुिंपत होने के िविभन्न चरणों से गुजरती है ।अंतर केवल इतना है
िक किव कमर् की फसल कालजयी, शाश्वत होती है ।उसका रस-क्षरण अक्षय होता है ।
सौंदयर्-बोध-संबंधी ूश्न
झूमने लगे फल,
रस अलौिकक ,
अमृत धाराएँ फूटतीं
रोपाई क्षण की ,
75 कटाई अनंतता की
लुटते रहने से जरा भी कम नहीं होती |
रस का अक्षय पाऽ सदा का
छोटा मेरा खेत चौकोना |
ूश्न इस किवता की भाषा संबंधी िवशेषताओं पर ूकाश डािलए –
उत्तर ;- १ ूतीकात्मकता
२ लाक्षिणकता २रूपक अलंकार–
रस का अक्षय पाऽ सदा का
छोटा मेरा खेत चौकोना।
ूश्न २ रस अलौिकक, अमृत धाराएँ, रोपाई Ð कटाई-ूतीकों के अथर् ःपष्ट कीिजए |
उत्तर :-
रस अलौिकक Ð काव्य रस िनंपित्त
अमृत धाराएँ- काव्यानंद
रोपाई Ð अनुभिू त को शब्दबद्ध करना
कटाई Ðरसाःवादन
िवषय-वःतु पर आधािरत ूश्नोत्तर
किवता –
छोटा मेरा खेत
ूश्न १ उमाशंकर जोशी ने िकस भाषा में किवताएँ िलखी हैं ?
उत्तर :- गुजराती भाषा
ूश्न२
कृ िष–कमर् एवं किव–कमर् में क्या क्या समानताएँ हैं ?
76 उत्तर :- कृ िष–कमर् एवं किव–कमर् में िनम्निलिखत समानताएँ हैं काव्य कृ ित की रचना बीज– वपन से लेकर पौधे के पुिंपत होने के िविभन्न चरणों से गुजरती
है ।
कृ िष–कमर् एवं किव–कमर् में समानताएँ :•
कागज का पन्ना, िजस पर रचना शब्दबद्ध होती है , किव को एक चौकोर खेतलगता है ।
•
इस खेत में िकसी अँधड़ (आशय भावनात्मक आँधी से होगा) के ूभाव से िकसी क्षण
एक बीज बोया जाता है । यह बीज-रचना िवचार और अिभव्यिक्त का हो सकता है ।
•
यह मूल रूप कल्पना का सहारा लेकर िवकिसत होता है और ूिबया में ःवयं िवगिलत
हो जाता है । इसीूकार बीज भी खाद, पानी, सूयर् की रोशनी ,हवा आिद लेकर िवकिसत
होता है |
•
काव्य Ðरचना से
शब्दों के अंकुर िनकलते हैं और अंततः कृ ित एक पूणर् ःवरूप महण
करती है , जो कृ िष-कमर् के िलहाज से पल्लिवत –पुिंपत और फिलत होने की िःथित है ।
अथर्-महण-संबंधी ूश्न
किवता–बगुलों के पंख
नभ में पाँती- बँधे बगुलों के पंख ,
चुराए िलए जातीं वे मेरी आँखें |
कजरारे बादलों की छाई नभ छाया ,
हौलेÐहौले जाती मुझे बाँध
िनज माया से |
उसे कोई तिनक रोक रक्खो |
वह तो चुराए िलए जाती मेरी आँखें
नभ में पाँती- बँधी बगुलों की पाँखें |
तैरती साँझ की सतेज श्वेत काया|
ूश्न१:- इस किवता में किव ने िकसका िचऽण िकया है ?
उत्तर :- किव ने काले बादलों पर उड़ती बगुलों की श्वेत पंिक्त का िचऽण िकया है|
ूश्न२:- आँखें चुराने का क्या अथर् है ?
77 उत्तर :- आँखें चुराने का आशय है Ðध्यान पूरी तरह खींच लेना ,एकटक दे खना ,मंऽमुग्ध कर दे ना
ूश्न३:कजरारे बादलों की छाई नभ छाया ,
हौले Ðहौले जाती मुझे बाँध
िनज माया से | - आशय ःपष्ट कीिजए |
उत्तर:- काले बादलों के बीच साँझ का सुरमई वातावरण बहत
ं र िदखता है | ऐसा अूितम
ु सुद
सौंदयर् अपने आकषर्ण में किव को बाँध लेता है |
ूश्न ४
उसे कोई तिनक रोक रक्खो |Õ- से किव का क्या अिभूाय है ?
उत्तर :- बगुलों की पंिक्त आकाश में दरू तक उड़ती जा रही है किव की मंऽमुग्ध आँखें उनका
पीछा कर रही हैं |किव उन बगुलों को रोक कर
रखने की गुहार लगा रहा है िक कहीं वे उसकी
आँखें ही अपने साथ न ले जाएँ|
सौंदयर्-बोध-संबंधी ूश्न
ूश्न १ किवता की भाषा संबंधी दो िवशेषताएँ िलिखए|
उत्तर :-१ िचऽात्मक भाषा
२ बोलचाल के शब्दों का ूयोग - हौले Ðहौले, पाँती, कजरारे ,साँझ
ूश्न २ किवता में ूयुक्त अलंकार चुन कर िलिखए |
उत्तर :- अनुूास अलंकार - बँधे बगुलों के पंख ,
मानवीकरण अलंकार - चुराए िलए जातीं वे मेरी आँखें |
ूश्न ३ :-Ô िनज मायाÕ के लाक्षिणक अथर् को ःपष्ट कीिजए |
उत्तर :- ूकृ ित का अूितम सौंदयर् वह माया है जो किव को आत्मिवभोर कर दे ती है ।यह पंिक्त
भी ूशंसात्मक उिक्त है ।
िवषय-वःतु पर आधािरत ूश्नोत्तर
78 ूश्न१:- ‘चुराए िलए जाती वे मेरी ऑख
ं ें’ से किव का क्या तात्पयर् है ?
उत्तर :-िचऽात्मक वणर्न द्वारा किव ने एक ओर काले बादलों पर उड़ती बगुलों की श्वेत पंिक्त का
िचऽ अंिकत िकया है तथा
इस अूितम दृँय के हृदय पर पड़ने वाले ूभाव को िचिऽत िकया
है । किव के अनुसार यह दृँय उनकी आँखें चुराए िलए जा रहा है |मंऽ मुग्ध किव इस दृँय के
ूभाव से आत्म िवःमृित की िःथित तक पहँु च जाता है ।
ूश्न२:-किव िकस माया से बचने की बात कहता है ?
उत्तर :-
माया िवश्व को अपने आकषर्ण में बाँध लेने के िलए ूिसद्ध है | कबीर ने भी Ôमाया
महा ठिगनी हम जानीÕ कहकर माया की शिक्त को ूितपािदत िकया है | काले बादलों में बगुलों
की सुद
ं रता अपना माया जाल फैला कर किव को अपने वश में कर रही है |
79 आरोह भाग-२, गद्य-भाग
•
ूश्न-संख्या १०- गद्य खंड में िदए गए पिठत गद्यांश से अथर्महण संबंधी चार ूश्न
पूछे जाएँगे, िजनके िलए िनधार्िरत अंक (२*४=८) हैं |
•
ूश्न-संख्या ११ - पाठों की िवषयवःतु से संबंिधत पाँच में से चार ूश्नों के उत्तर दे ने
हैं , िजनके िलए िनधार्िरत अंक (३*४=१२) हैं|
परीक्षा में अच्छे अंक ूाप्त करने के िलए ध्यान दे ने योग्य बातें –
1. लेख एवं वतर्नी की शुद्धता तथा वाक्य-गठन पर ध्यान दें |
2. हर पाठ का सार,पृष्ठभूिम,िवषयवःतु तथाकथानक को समझना आवँयक है | अत:िवद्याथीर्
हर पाठ का सारांश भलीभाँित याद कर लें |
3. ूश्नों को ध्यान से पढ़ें तदनुसार अपेिक्षत उत्तर िलखें |
4. उत्तर िलखते समय संबंिधत मुख्य िबंदओं
का शीषर्कबनाते हए
ु
ु उत्तर िलखें, यथा-तीन अंक
के ूश्नों के उत्तर िलखते समय कम से कम तीन मुख्य उत्तर-िबंदओं
का उल्लेख करते हए
ु
ु
उत्तर ःपष्ट करें |
5. अंक-योजना के अनुसार िनधार्िरत शब्द-सीमा के अंतगर्त उत्तर िलखें | परीक्षाथीर् कई बार
एक अंक के ूश्न का उत्तर बहत
ु लंबा, कई बार पूरा पृष्ठ िलख दे ते हैं जो समय और ऊजार्
की बबार्दी है |
6. ध्यान रहे िक अिधक िलखने से अच्छे
अंक नहीं आते बिल्क सरल-सुबोध भाषा में िलखे
गए सटीक उत्तर, सारगिभर्त तथ्य तथा उदाहरण के द्वारा ःपष्ट िकए गए उत्तर ूभावशाली
होते हैं |
पाठ 1 1 - भिक्तन
लेिखका- महादे वी वमार्
पाठ का सारांश- भिक्तन िजसका वाःतिवक नाम लआमी था,लेिखका Ôमहादे वी वमार्Õ की सेिवका है
| बचपन में ही भिक्तन की माँ की मृत्यु हो गयी| सौतेली माँ ने पाँच वषर् की आयु में िववाह
तथा नौ वषर् की आयु में गौना कर भिक्तन को ससुराल भेज िदया| ससुराल में भिक्तन ने तीन
बेिटयों को जन्म िदया, िजस कारण उसे सास और िजठािनयों की उपेक्षा सहनी पड़ती थी| सास
और िजठािनयाँ आराम फरमाती थी और भिक्तन तथा उसकी नन्हीं बेिटयों को घर और खेतों का
सारा काम करना पडता था| भिक्तन का पित उसे बहत
ु चाहता था| अपने पित के ःनेह के बल
80 बल पर भिक्तन ने ससुराल वालों से अलगौझा कर अपना अलग घर बसा िलया और सुख से
रहने लगी, पर भिक्तन का दभार्
ु ग्य, अल्पायु में ही उसके पित की मृत्यु हो गई | ससुराल वाले
भिक्तन की दसरी
शादी कर उसे घर से िनकालकर उसकी संपित्त हड़पने की सािजश करने लगे|
ू
ऐसी पिरिःथित में भिक्तन ने अपने केश मुड
ं ा िलए और संन्यािसन बन गई | भिक्तन
ःवािभमानी, संघषर्शील, कमर्ठ और दृढ संकल्प वाली स्तर्ी है जो िपतृसत्तात्मक मान्यताओं और
छ्ल-कपट से भरे समाज में अपने और अपनी बेिटयों के हक की लड़ाई लड़ती है ।घर गृहःथी
सँभालने के िलए अपनी बड़ी बेटी दामाद को बुला िलया पर
दभार्
ु ग्य ने यहाँ भी भिक्तन का
पीछा नहीं छोड़ा, अचानक उसके दामाद की भी मृत्यु हो गयी| भिक्तन के जेठ-िजठौत ने सािजश
रचकर भिक्तन की िवधवा बेटी का िववाह जबरदःती अपने तीतरबाज साले से कर िदया| पंचायत
द्वारा कराया गया यह संबंध दखदायी
रहा | दोनों माँ-बेटी का मन घर-गृहःथी से उचट गया,
ु
िनधर्नता आ गयी, लगान न चुका पाने के कारण जमींदार ने भिक्तन को िदन भर धूप में खड़ा
रखा| अपमािनत भिक्तन पैसा कमाने के िलए गाँव छोड़कर शहर आ जाती है और महादे वी की
सेिवका बन जाती है| भिक्तन के मन में महादे वी के ूित बहत
ु आदर, समपर्ण और अिभभावक
के समान अिधकार भाव है| वह छाया के समान महादे वी के साथ रहती है| वह रात-रात भर
जागकर िचऽकारी या लेखन जैसे कायर् में व्यःत अपनी मालिकन की सेवा का अवसर ढँू ढ लेती
है| महादे वी, भिक्तन को नहीं बदल पायी पर भिक्तन ने महादे वी को बदल िदया| भिक्तन के हाथ
का मोटा-दे हाती खाना खाते-खाते महादे वी का ःवाद बदल गया, भिक्तन ने महादे वी को दे हात के
िकःसे-कहािनयाँ, िकंवदं ितयाँ कंठःथ करा दी| ःवभाव से महाकंजूस होने पर भी भिक्तन, पाई-पाई
कर जोडी हई
ु १०५ रुपयों की रािश को सहषर् महादे वी को समिपर्त कर दे ती है| जेल के नाम से
थर-थर काँपने वाली भिक्तन अपनी मालिकन के साथ जेल जाने के िलए बड़े लाट साहब तक से
लड़ने को भी तैयार हो जाती है| भिक्तन, महादे वी के जीवन पर छा जाने वाली एक ऐसी सेिवका
है िजसे लेिखका नहीं खोना चाहती।
पाठ आधािरत ूश्नोत्तर
नोट- उत्तर में िनिहत रे खांिकत वाक्य, मुख्य संकेत िबंद ु हैं |
ु
ूश्न 1-भिक्तन का वाःतिवक नाम क्या था, वह अपने नाम को क्यों छपाना
चाहती थी?
उत्तर-भिक्तन का वाःतिवक नाम लआमी था, िहन्दओं
के अनुसारलआमी धन की दे वीहै । चूँिक
ु
भिक्तन गरीब थी| उसके वाःतिवक नाम के अथर् और उसके जीवन के यथाथर् में िवरोधाभास है ,
िनधर्न भिक्तन सबको अपना असली नाम लआमी बताकर उपहास का पाऽ नहीं बनना चाहती थी
ु
इसिलए वह अपना असली नाम छपाती
थी।
ूश्न 2- लेिखका ने लआमी का नाम भिक्तन क्यों रखा?
81 उत्तर-घुटा हआ
िसर, गले में कंठी माला और भक्तों की तरह सादगीपूणर् वेशभूषा दे खकर महादे वी
ु
वमार् ने लआमी का नाम भिक्तन रख िदया | यह नाम उसके व्यिक्तत्व से पूणत
र् : मेल खाता था |
ूश्न 3-भिक्तन के जीवन को िकतने
पिरच्छे दों में िवभािजत िकया गया है ?
उत्तर- भिक्तन के जीवन को चार भागों में बाँटा गया है •
पहला पिरच्छे द-भिक्तन का बचपन, माँ की मृत्यु, िवमाता के द्वारा भिक्तन का बाल-िववाह
करा दे ना ।
•
िद्वतीय पिरच्छे द-भिक्तन का वैवािहक जीवन, सास तथा िजठािनयों का अन्यायपूणर्
व्यवहार, पिरवार से अलगौझा कर लेना ।
•
तृतीय पिरच्छे द- पित की मृत्यु, िवधवा के रूप में संघषर्शील जीवन।
•
चतुथर् पिरच्छे द- महादे वी वमार् की सेिवका के रूप में ।
ूश्न 4- भिक्तन पाठ के आधार पर भारतीय मामीण समाज में लड़के-लड़िकयों में िकये जाने वाले
भेदभाव का उल्लेख कीिजए |
भारतीय मामीण समाज में लड़के-लड़िकयों में भेदभाव िकया जाता है| लड़िकयों को खोटा िसक्का
या पराया धन माना जाता है| भिक्तन ने तीन बेिटयों को जन्म िदया, िजस कारण उसे सास और
िजठािनयों की उपेक्षा सहनी पड़ती थी| सास और िजठािनयाँ आराम फरमाती थी क्योंिक उन्होंने
लड़के पैदा िकए थे और भिक्तन तथा उसकी नन्हीं बेिटयों को घर और खेतों का सारा काम
करना पडता था|
भिक्तन और उसकी बेिटयों को रूखा-सूखा मोटा अनाज खाने को िमलता था
जबिक उसकी िजठािनयाँ और उनके काले-कलूटे बेटे दध
ू -मलाई राब-चावल की दावत उड़ाते थे
ूश्न 5-भिक्तन पाठ के आधार पर पंचायत के न्याय पर िटप्पणी कीिजए |
भिक्तन की बेटी के सन्दभर् में पंचायत द्वारा िकया गया न्याय,
तकर्हीन और अंधे कानून पर
आधािरतहै | भिक्तन के िजठौत ने संपित्त के लालच में षडयंऽ कर भोली बच्ची को धोखे से जाल
में फंसाया| पंचायत ने िनदोर्ष लड़की की कोई बात नहीं सुनी और एक तरफ़ा फैसला दे कर
उसका िववाह जबरदःती िजठौत के िनकम्मे तीतरबाज साले से कर िदया | पंचायत के अंधे
कानून से दष्टों
और िनदोर्ष को दं ड िमला |
ु को लाभ हआ
ु
ूश्न 6-भिक्तन की पाक-कला के बारे में िटप्पणी कीिजए |
ू को बहत
भिक्तन को ठे ठ दे हाती, सादा भोजन पसंद था | रसोई में वह पाक छत
ु महत्त्व दे ती थी
| सुबह-सवेरे
नहा-धोकर चौके की सफाई करके वह द्वार पर कोयले की मोटी रे खा खींच दे ती थी|
82 दे ती थी| िकसी को रसोईघर में ूवेश करने नहीं दे ती थी| उसे अपने बनाए भोजन पर बड़ा
अिभमान था| वह अपने बनाए भोजन का ितरःकार नहीं सह सकती थी |
ूश्न 7- िसद्ध कीिजए िक भिक्तन तकर्-िवतकर् करने में मािहर थी |
भिक्तन तकर्पटु थी | केश मुड
ँ ाने से मना िकए जाने पर वह शास्तर्ों का हवाला दे ते
हए
ु कहती है
ु
Ôतीरथ गए मुड
ँ ाए िसद्धÕ | घर में इधर-उधर रखे गए पैसों को वह चुपचाप उठा कर छपा
लेती है ,
टोके जानेपर वह वह इसे चोरी नही मानती बिल्क वह इसे अपने घर में पड़े पैसों को सँभालकर
रखना कहती है|
पढाई-िलखाई से बचने के िलए भी वह अचूक तकर् दे ती है िक अगर मैं भी
पढ़ने लगूँ तो घर का काम कौन दे खेगा?
ूश्न 8-भिक्तन का दभार्
ु ग्य भी कम हठी नही था, लेिखका ने ऐसा क्यों कहा है ?
उत्तर- भिक्तन का दभार्
ु ग्य उसका पीछा नहीं छोड़ता था1- बचपन में ही माँ की मृत्यु ।
2- िवमाता की उपेक्षा ।
3- भिक्तन(लआमी) का बालिववाह ।
4- िपता का िनधन ।
5- तीन-तीन बेिटयों को जन्म दे ने के कारण सास और िजठािनयों के द्वारा भिक्तन की उपेक्षा
।
6- पित की असमय मृत्यु ।
7- दामाद का िनधन और पंचायत के द्वारा िनकम्मे तीतरबाज युवक से भिक्तन की िवधवा
बेटी का जबरन िववाह ।
8- लगान न चुका पाने पर जमींदार के द्वारा भिक्तन का अपमान।
ूश्न 9-भिक्तन ने महादे वी वमार् के जीवन पर कैसे ूभािवत िकया?
उत्तर- भिक्तन के साथ रहकर महादे वी की जीवन-शैली सरल हो गयी, वे अपनी सुिवधाओं की चाह
को िछपाने लगीं और असुिवधाओं को सहने लगीं। भिक्तन ने उन्हें दे हाती भोजन िखलाकर उनका
ःवाद बदल िदया। भिक्तन माऽ एक सेिवका न होकर महादे वी की अिभभावक और आत्मीय बन
गयी।भिक्तन, महादे वी के जीवन पर छा जाने वाली एक ऐसी सेिवका है िजसे लेिखका नहीं खोना
चाहती।
ूश्न 10- भिक्तन के चिरऽ की िवशेषताओं का उल्लेख कीिजए।
83 उत्तर- महादे वी वमार् की सेिवका भिक्तन के व्यिक्तत्व की िवशेषताएं िनम्नांिकत हैं •
समिपर्त सेिवका
•
ःवािभमानी
•
तकर्शीला
•
पिरौमी
•
संघषर्शील
•
ूश्न 1 1 -भिक्तन के दगु
र् ों का उल्लेख करें ।
ु ण
उत्तर- गुणों के साथ-साथ भिक्तन के व्यिक्तत्व में अनेक दगु
र् भी िनिहत है ु ण
ु
1. वह घर में इधर-उधर पड़े रुपये-पैसे को भंडार घर की मटकी में छपा
दे ती है और अपने
इस कायर् को चोरी नहीं मानती।
2. महादे वी के बोध से बचने के िलए भिक्तन बात को इधर-उधर करके बताने को झूठ नही
मानती। अपनी बात को सही िसद्ध करने के िलए वह तकर्-िवतकर् भी करती है ।
3. वह दसरों
को अपनी इच्छानुसार बदल दे ना चाहती है पर ःवयं िबलकुल नही बदलती।
ू
ूश्न 12 िनम्नांिकत भाषा-ूयोगों का अथर् ःपष्ट कीिजए•
पहली कन्या के दो और संःकरण कर डाले- भिक्तन ने अपनी पहली कन्या के बाद उसके
जैसी दो और कन्याएँ पैदा कर दी अथार्त भिक्तन के एक के बाद एक तीन बेिटयाँ पैदा हो
गयीं |
•
खोटे िसक्कों की टकसाल जैसी पत्नी- आज भी अिशिक्षत मामीण समाज में बेिटयों को
खोटा िसक्का कहा जाता है । भिक्तन ने एक के बाद एक तीन बेिटयाँ पैदा कर दी इसिलए
उसे खोटे िसक्के को ढालने वाली मशीन कहा गया।
ूश्न 13-भिक्तन पाठ में लेिखका ने समाज की िकन समःयाओं का उल्लेख िकया है ?
उत्तर- भिक्तन पाठ के माध्यम से लेिखका ने भारतीय मामीण समाज की अनेक समःयाओं का
उल्लेख िकया है 1. लड़के-लड़िकयों में िकया जाने वाला भेदभाव
2. िवधवाओं की समःया
3. न्याय के नाम पर पंचायतों के द्वारा िस्तर्यों के मानवािधकार को कुचलना
4. अिशक्षा और अंधिवश्वास
गद्यांश-आधािरत अथर्महण संबंिधत ूश्नोत्तर
84 पिरवार और पिरिःथितयों के कारण ःवभाव में जो िवषमताएँ उत्पन्न हो गई हैं , उनके भीतर से
एक ःनेह और सहानुभिू त की आभा फूटती रहती है , इसी से उसके संपकर् में आनेवाले व्यिक्त
उसमें जीवन की सहज मािमर्कता ही पाते हैं । छाऽावास की बािलकाओं में से कोई अपनी चाय
बनवाने के िलए दे हली पर बैठी रहती हैं , कोई बाहर खडी मेरे िलए नाँते को चखकर उसके
ःवाद की िववेचना करती रहती है । मेरे बाहर िनकलते ही सब िचिड़यों के समान उड़ जाती हैं
और भीतर आते ही यथाःथान िवराजमान हो जाती है । इन्हें आने में रूकावट न हो, संभवतः
इसी से भिक्तन अपना दोनों जून का भोजन सवेरे ही बनाकर ऊपर के आले में रख दे ती है और
ू के सनातन िनयम से समझौता कर लेती है ।
खाते समय चौके का एक कोना धोकर पाकÐछत
मेरे पिरिचतों और सािहित्यक बंधओ
ु ं से भी भिक्तन िवशेष पिरिचत है , पर उनके ूित
भिक्तन के सम्मान की माऽा, मेरे ूित उनके सम्मान की माऽा पर िनभर्र है और सद्भाव उनके
ूित मेरे सद्भाव से िनिश्चत होता है । इस संबंध में भिक्तन की सहज बुिद्ध िविःमत कर दे ने वाली
है ।
(क) भिक्तन का ःवभाव पिरवार में रहकर कैसा हो गया है ?
उत्तर-िवषम पिरिःथितजन्य उसके उम, हठी और दरामही
ःवभाव के बावजूद भिक्तन के भीतर
ु
ःनेह और सहानुभिू त की आभा फूटती रहती है । उसके संपकर् में आने वाले व्यिक्त उसमें जीवन
की सहज मािमर्कता ही पाते हैं ।
(ख) भिक्तन के पास छाऽावास की छाऽाएँ क्यों आती हैं ?
उत्तर- भिक्तन के पास कोई छाऽा अपनी चाय बनवाने आती है और दे हली पर बैठी रहती है , कोई
महादे वी जी के िलए बने नाँते को चखकर उसके ःवाद की िववेचना करती रहती है । महादे वी
को दे खते ही सब छाऽाएँ भाग जाती हैं , उनके जाते ही िफर वापस आ जातीं हैं
भिक्तन का
सहज-ःनेह पाकर िचिड़यों की तरह चहचहाने लगती हैं |
(ग) छाऽाओं के आने में रुकावट न डालने के िलए भिक्तन ने क्या उपाय िकया ?
ू
उत्तर- छाऽाओं के आने में रुकावट न डालने के िलए भिक्तन ने अपने पाक-छत
के िनयम से
समझौता कर िलया | भिक्तन अपना दोनों वक्त का खाना बनाकर सुबह ही आले में रख दे ती और
खाते समय चौके का एक कोना धोकर वहाँ बैठकर खा िलया करती थी तािक छाऽाएँ िबना रोकटोक के उसके पास आ सकें।
(घ) सािहत्यकारों के ूित भिक्तन के सम्मान का क्या मापदं ड है ?
उत्तर-भिक्तन महादे वी के सािहित्यक िमऽ के ूित सद्भाव रखती थी िजसके ूित महादे वी ःवयं
सद्भाव रखती थी | वह सभी से पिरिचत है पर उनके ूित सम्मान की माऽा महादे वी जी के
85 सम्मान की
माऽा पर िनभर्र करती है । वह एक अद्भत
ु ढं ग से जान लेती थी िक कौन िकतना
सम्मान करता है । उसी अनुपात में उसका ूाप्य उसे दे ती थी।
12
बाजार-दशर्न
लेखक- जैनेंि कुमार
पाठ का सारांश बाजार-दशर्न पाठ में बाजारवाद और उपभोक्तावाद के साथ-साथ अथर्नीित एवं
दशर्न से संबिं धत ूश्नों को सुलझाने का ूयास िकया गया है । बाजार का जाद ू तभी असर करता
है जब मन खाली हो| बाजार के जाद ू को रोकने का उपाय यह है िक बाजार जाते समय मन
खाली ना हो, मन में लआय भरा हो| बाजार की असली कृ ताथर्ता है जरूरत के वक्त
काम आना|
बाजार को वही मनुंय लाभ दे सकता है जो वाःतव में अपनी आवँयकता के अनुसार खरीदना
चाहता है| जो लोग अपने पैसों के घमंड में अपनी पचेर्िजंग पावर को िदखाने के िलए चीजें
खरीदते हैं वे बाजार को शैतानी व्यंग्य शिक्त दे ते हैं| ऐसे लोग बाजारूपन और कपट बढाते हैं |
पैसे की यह व्यंग्य शिक्त व्यिक्त को अपने
सगे लोगों के ूित भी कृ तघ्न बना सकती है |
साधारण जन का हृदय लालसा, ईंयार् और तृंणा से जलने लगता है | दसरी
ओर ऐसा व्यिक्त
ू
िजसके मन में लेश माऽ भी लोभ और तृंणा नहीं है , संचय की इच्छा नहीं है वह इस व्यंग्यशिक्त से बचा रहता है | भगतजी ऐसे ही आत्मबल के धनी आदशर् माहक और बेचक हैं िजन पर
पैसे की व्यंग्य-शिक्त का कोई असर नहीं होता | अनेक उदाहरणों के द्वारा लेखक ने यह ःपष्ट
िकया है िक एक ओर बाजार, लालची, असंतोषी और खोखले मन वाले व्यिक्तयों को लूटने के
िलए है वहीं दसरी
ओर संतोषी मन वालों के िलए बाजार
ू
की चमक-दमक, उसका आकषर्ण कोई
महत्त्व नहीं रखता।
ूश्न१ - पचेर्िजंग पावर िकसे कहा गया है , बाजार पर इसका क्या ूभाव पड़ता है ?
उत्तर- पचेर्िजंग पावर का अथर् है खरीदने की शिक्त। पचेर्िजंग पावर के घमंड में व्यिक्त िदखावे के
िलए आवँयकता से अिधक खरीदारीकरता है और बाजार को शैतानी व्यंग्य-शिक्त दे ता है । ऐसे
लोग बाजार का बाजारूपन बढ़ाते हैं ।
ूश्न२ -लेखक ने बाजार का जाद ू िकसे कहा है , इसका क्या ूभाव पड़ता है ?
उत्तर- बाजार की चमक-दमक के चुंबकीय आकषर्ण को
बाजार का जाद ू कहा गया है , यह जाद ू
आंखों की राह कायर् करता है । बाजार के इसी आकषर्ण के कारण माहक सजी-धजी चीजों को
आवँयकता न होने पर भी खरीदने को िववश हो जाते हैं ।
ूश्न३ -आशय ःपष्ट करें ।
86 •
मन खाली होना
•
मन भरा होना
•
मन बंद होना
उत्तर-मन खाली होना- मन में कोई िनिश्चत वःतु खरीदने का लआय न होना। िनरुद्दे ँय बाजार
जाना और व्यथर् की चीजों को खरीदकर
लाना।
मन भरा होना- मन लआय से भरा होना। िजसका मन भरा हो वह भलीभाँित
जानता है िक उसे
बाजार से कौन सी वःतु खरीदनी है , अपनी आवँयकता की चीज खरीदकर वह बाजार को
साथर्कता ूदान करता है ।
मन बंद होना-मन में िकसी भी ूकार की इच्छा न होना अथार्त अपने मन को शून्य कर दे ना।
ूश्न४ - ‘जहाँ तृंणा है , बटोर रखने की ःपृहा है , वहाँ उस बल का बीज नहीं है ।’ यहां िकस बल
की चचार् की गयी है ?
उत्तर- लेखक ने संतोषी ःवभाव के व्यिक्त के आत्मबलकी चचार् की है । दसरे
शब्दों में यिद मन
ू
में संतोष हो तो व्यिक्त िदखावे और ईंयार् की भावना से दरू रहता है उसमें संचय करने की
ूवृित्त नहीं होती।
ूश्न५ - अथर्शास्तर्, अनीितशास्तर् कब बन जाता है ?
उत्तर- जब बाजार में कपट और शोषण बढ़ने लगे, खरीददार
िदखावे
अपनी पचेर्िचंग पावर के घमंड में
के िलए खरीददारी करें | मनुंयों में परःपर भाईचारा समाप्त हो जाए| खरीददार और
दकानदार
एक दस
को अपना लाभ
ु
ू रे को ठगने की घात में लगे रहें , एक की हािन में दसरे
ू
िदखाई दे तो बाजार का अथर्शास्तर्, अनीितशास्तर्
बन जाता है । ऐसे बाजार मानवता के िलए
िवडं बना है ।
ूश्न६-भगतजी बाजार और समाज को िकस ूकार साथर्कता ूदान कर रहे हैं ?
उत्तर- भगतजी के मन में सांसािरक आकषर्णों के िलए कोई तृंणा नहीं है । वे संचय, लालच और
िदखावे से दरू रहते हैं । बाजार और व्यापार उनके िलए आवँयकताओं की पूितर् का साधन माऽ
है । भगतजी के मन का संतोष और िनःपृह भाव, उनको ौेष्ठ उपभोक्ता और िवबेता बनाते हैं ।
ूश्न ७ _ भगत जी के व्यिक्तत्व के सशक्त पहलुओं का उल्लेख कीिजए |
उत्तर-िनम्नांिकत िबंद ु उनके व्यिक्तत्व के सशक्त पहलू को उजागर करते हैं ।
•
पंसारी की दकान
से केवल अपनी जरूरत का सामान (जीरा और नमक) खरीदना।
ु
87 •
िनिश्चत समय पर चूरन बेचने के िलए िनकलना।
•
छ्ह आने की कमाई होते ही चूरन बेचना बंद कर दे ना।
•
बचे हए
ु चूरन को बच्चों को मुझत बाँट दे ना।
•
•
•
सभी काजय-जय राम कहकर ःवागत करना।
बाजार की चमक-दमक से आकिषर्त न होना।
समाज को संतोषी जीवन की िशक्षा दे ना।
ूश्न7-बाजार की साथर्कता िकसमें है ?
उत्तर- मनुंय की आवँयकताओं की पूितर् करने में ही बाजार की साथर्कता है । जो माहक अपनी
आवँयकताओं की चीजें खरीदते हैं वे बाजार को साथर्कता ूदान करते हैं । जो िवबेता, माहकों
का शोषण नहीं करते और छल-कपट से माहकों को लुभाने का ूयास नही करते वे भी बाजार
को साथर्क बनाते हैं ।
गद्यांश-आधािरत अथर्महण-संबंिधत ूश्नोत्तर
बाजार मे एक जाद ू है । वह जाद ू आँख की तरह काम करता है । वह रूप का जाद ू है
पर जैसे चुंबक का जाद ू लोहे पर ही चलता है , वैसे ही इस जाद ू की भी मयार्दा है जेब भरी हो,
और मन खाली हो, ऐसी हालत में जाद ू का असर खूब होता है । जेब खाली पर मन भरा न हो
तो भी जाद ू चल जाएगा। मन खाली है तो बाजार की अनेकानेक चीजों का िनमंऽण उस तक
पहँु च जाएगा। कहीं हई
ु उस वक्त जेब भरी, तब तो िफर वह मन िकसकी मानने वाला है । मालूम
होता है यह भी लू,ँ वह भी लू।ँ सभी सामान जरूरी और आराम को बढ़ाने वाला मालूम होता है
पर यह सब जाद ू का असर है । जाद ू की सवारी उतरी िक पता चलता है िक फैंसी-चीजों की
बहतायत
आराम में मदद नहीं दे ती, बिल्क खलल ही डालती है । थोड़ी दे र को ःवािभमान को
ु
जरूर सेंक िमल जाता है पर इससे अिभमान को िगल्टी की खुराक ही िमलती है । जकड़ रे शमी
डोरी की हो तो रे शम के ःपशर् के मुलायम के कारण क्या वह कम
जकड़ दे गी ?
पर उस जाद ू की जकड़ से बचने का एक सीधा उपाय है वह यह िक बाजार जाओ तो
खाली मन न हो । मन खाली हो तब बाजार न जाओ कहते हैं , लू में जाना हो तो पानी पीकर
जाना चािहए पानी भीतर हो, लू का लूपन व्यथर् हो जाता है । मन लआय से भरा हो तो बाजार
फैला का फैला ही रह जाएगा। तब वह घाव िबलकुल नहीं दे सकेगा, बिल्क कुछ आनंद ही दे गा।
तब बाजार तुमसे कृ ताथर् होगा, क्योंिक तुम कुछ न कुछ सच्चा लाभ उसे दोगे। बाजार की
असली कृ ताथर्ता है आवँयकता के समय काम आना।
ूश्न-1 बाजार के जाद ू को लेखक ने कैसे ःपष्ट िकया है ?
88 उत्तर- बाजार के रूप का जाद ू आँखों की राह से काम करता हआ
हमें आकिषर्त करता है । बाजार
ु
का जाद ू ऐसे चलता है जैसे लोहे के ऊपर चुंबक का जाद ू चलता है । चमचमाती रोशनी में सजी
फैंसी चींजें माहक को अपनी ओर आकिषर्त करती हैं| इसी चुम्बकीय शिक्त के कारण व्यिक्त
िफजूल सामान को भी खरीद लेता है |
ूश्न-2 जेब भरी हो और मन खाली तो हमारी क्या दशा होती है ?
उत्तर- जेब भरी हो और मन खाली हो तो हमारे ऊपर बाजार का जाद ू खूब असर करता है ।
मन, खाली है तो बाजार की अनेकानेक चीजों का िनमंऽण मन तक पहँु च जाता है और उस
समय यिद जेब भरी हो तो मन हमारे िनयंऽण में नहीं रहता।
ूश्न-3 फैंसी चीजों की बहतायत
का क्या पिरणाम होता है ?
ु
उत्तर- फैंसी चीजें आराम की जगह आराम में व्यवधान ही डालती है । थोड़ी दे र को अिभमान को
जरूर सेंक िमल जाती है पर िदखावे की ूवृित्त में वृिद्ध होती है ।
ूश्न-4 जाद ू की जकड़ से बचने का क्या उपाय है ?
उत्तर- जाद ू की जकड़ से बचने के िलए एक ही उपाय है , वह यह है िक बाजार जाओ तो
मन
खाली न हो, मन खाली हो तो बाजार मत जाओ।
13
काले मेघा पानी दे
लेखक-धमर्वीर भारती
पाठ का सारांश -‘काले मेघा पानी दे ’ िनबंध, लोकजीवन के िवश्वास और िवज्ञान के तकर् पर
आधािरत है । जब भीषण गमीर् के कारण व्याकुल लोग वषार् कराने के िलए पूजा-पाठ और कथािवधान कर थकÐहार जाते हैं तब वषार् कराने के िलए अंितम उपाय के रूप में इन्दर सेना
िनकलती है| इन्दर सेना, नंग-धड़ं ग बच्चों की टोली है जो कीचड़ में लथपथ होकर गली-मोहल्ले
में पानी माँगने िनकलती है| लोग अपने घर की छतों-िखड़िकयों से इन्दर सेना पर पानी डालते हैं
| लोगों की मान्यता है िक इन्ि, बादलों के ःवामी और वषार् के दे वता हैं | इन्ि की सेना पर पानी
डालने से इन्ि भगवान ूसन्न होकर पानी बरसाएंगे | लेखक का तकर् है िक जब पानी की
इतनी कमी है तो लोग मुिँकल से जमा िकए पानी को बाल्टी भर-भरकर इन्दर सेना पर
डालकर पानी को क्यों बबार्द करते है ? आयर्समाजी िवचारधारा वाला लेखक इसे अंधिवश्वास
मानता है | इसके िवपरीत लेखक की जीजी उसे समझाती है िक यह पानी की बबार्दी नहीं बिल्क
पानी की बुवाई है | कुछ पाने के िलए कुछ दे ना पड़ता है | त्याग के िबना दान नहीं होता|
ूःतुत िनबंध में लेखक ने ॅष्टाचार की समःया को उठाते हए
ु कहा है िक जीवन में कुछ पाने
के िलए त्याग आवँयक है । जो लोग त्याग और दान की महत्ता को नहीं मानते, वे ही ॅष्टाचार
89 ॅष्टाचार में िलप्त रहकर दे श और समाज को लूटते हैं| जीजी की आःथा, भावनात्मक सच्चाई को
पुष्ट करती है और तकर् केवल वैज्ञािनक तथ्य को सत्य मानता है । जहाँ तकर्, यथाथर् के कठोर
धरातल पर सच्चाई को परखता है तो वहीं आःथा, अनहोनी बात को भी ःवीकार कर मन को
संःकािरत करती है । भारत की ःवतंऽता के ५० साल बाद भी दे श में व्याप्त भष्टाचार और ःवाथर्
की भावना को दे खकर लेखक दखी
है | सरकार द्वारा चलाई जा रही योजनाएँ गरीबों तक क्यों
ु
नहीं पहँु च
पा रहीं
हैं ?
काले मेघा के दल उमड़ रहे हैं पर आज भी गरीब की गगरी फूटी हई
ु
क्यों है ? लेखक ने यह ूश्न पाठकों के िलए छोड़ िदया है |
ूश्न1-इन्दर सेना घर-घर जाकर पानी क्यों माँगती थी?
उत्तर- गाँव के लोग बािरश के िलए भगवान इं ि से
ूाथर्ना िकया करते थे। जब पूजा-पाठ,ोत
आिद उपाय असफ़ल हो जाते थे तो भगवान इं ि को ूसन्न करने के िलए गाँव के िकशोर, बच्चे
कीचड़ में लथपथ होकर गली-गली घूमकर लोगों से पानी माँगते थे।
ूश्न2-इन्दरसेना को लेखक मेढक-मंडली क्यों कहता है , जीजी के बारÐबार कहने पर भी वह
इन्दरसेना पर पानी फेंकने को राजी क्यों नहीं होता ?
उत्तर- इन्दरसेना का कायर् आयर्समाजी िवचारधारा वाले लेखक को अंधिवश्वास लगता है , उसका
मानना है िक यिद इं दरसेना दे वता से पानी िदलवा सकती है तो ःवयं अपने िलए पानी क्यों नहीं
माँग लेती? पानी की कमी होने पर भी लोग घर में एकऽ िकये हए
ु पानी को इं दरसेना पर फ़ेंकते
हैं । लेखक इसे पानी की िनमर्म बरबादी मानता है ।
ूश्न3- रूठे हए
ु लेखक को जीजी ने िकस ूकार समझाया?
उत्तर- जीजी ने लेखक को प्यार से लड्डू -मठरी िखलाते हए
ु िनम्न तकर् िदए1- त्याग का महत्त्व- कुछ पाने के िलए कुछ दे ना पड़ता है ।
2- दान की महत्ता- ॠिष-मुिनयों ने दान को सबसे ऊँचा ःथान िदया है । जो चीज अपने पास
भी कम हो और अपनी आवँयकता को भूलकर वह चीज दसरों
को दान कर दे ना ही
ू
त्याग है |
3- इं िदे व को जल का अध्यर् चढ़ाना- इं दरसेना पर पानी फ़ेंकना पानी की बरबादी नहीं बिल्क
इं िदे व को जल का अध्यर् चढ़ाना है ।
4- पानी की बुवाई करना- िजस ूकार िकसान फ़सल उगाने के िलए जमीन पर बीज डालकर
बुवाई करता है वैसे ही पानी वाले बादलों की फ़सल पाने के िलए इन्दर सेना पर पानी
डाल कर पानी की बुवाई की जाती है ।
ूश्न4-निदयों का भारतीय सामािजक और सांःकृ ितक पिरवेश में क्या महत्व है ?
90 उत्तर- गंगा भारतीय समाज में सबसे पूज्य सदानीरा नदी है । िजसका भारतीय इितहास में
धािमर्क, पौरािणक और सांःकृ ितक महत्व है | वह भारतीयों के िलए केवल एक नदी नहीं अिपतु
माँ है , ःवगर् की सीढ़ी है , मोक्षदाियनी है । उसमें पानी नहीं अिपतु अमृत तुल्य जल बहता है ।
भारतीय संःकृ ित में निदयों के िकनारे मानव सभ्यताएँ फली-फूली हैं|
बड़े -बड़े नगर, तीथर्ःथान
निदयों के िकनारे ही िःथत हैं ऐसे पिरवेश मेंभारतवासी सबसे पहले गंगा मैया की जय ही
बोलेंगे। निदयाँ हमारे जीवन का आधार हैं , हमारा दे श कृ िष ूधान है । निदयों के जल से ही
भारत भूिम हरी-भरी है । निदयों के िबना जीवन की कल्पना नहीं कर सकते, यही कारण है िक
हम भारतीय निदयों की पूजा करते हैं |
ूश्न4-आजादी के पचास वषोर्ं के बाद भी लेखक क्यों दखी
है , उसके मन में कौन से ूश्न उठ रहे
ु
हैं ?
उत्तर- आजादी के पचास वषोर्ं बाद भी भारतीयों की सोच में सकारात्मक बदलाव न दे खकर
लेखक दखी
है । उसके मन में कई ूश्न उठ रहे हैं ु
1. क्या हम सच्चे अथोर्ं में ःवतन्ऽ हैं ?
2. क्या हम अपने दे श की संःकृ ित और सभ्यता को समझ पाए हैं ?
3. राष्टर् िनमार्ण में हम पीछे क्यों हैं , हम दे श के िलए क्या कर रहे हैं ?
4. हम ःवाथर् और ॅष्टाचार में िलप्त रहते हैं , त्याग में िवश्वास क्यों नहीं करते ?
5. सरकार द्वारा चलाई जा रही सुधारवादी योजनाएँ गरीबों तक क्यों नहीं पहँु चती है ?
गद्यांश पर आधािरत अथर्महण-संबंिधत ूश्नोत्तर
सचमुच ऐसे िदन होते जब गली-मुहल्ला, गाँव-शहर हर जगह लोग
गरमी में भुन-भुन कर ऽािहमाम कर रहे होते, जेठ के दसतपा बीतकर आषाढ का पहला पखवाड़ा
बीत चुका होता पर िक्षितज पर कहीं बादलों की रे ख भी नहीं दीखती होती, कुएँ सूखने लगते,
नलों में एक तो बहत
ु कम पानी आता और आता भी तो आधी रात को, वो भी खौलता हआ
ु
पानी हो | शहरों की तुलना में गाँव में और भी हालत खराब थी| जहाँ जुताई होनी चािहए थी
वहाँ खेतों की िमट्टी सूखकर पत्थर हो जाती िफर उसमें पपड़ी पड़कर जमीन फटने लगती, लू
ऐसी िक चलते-चलते आदमी िगर पड़े | ढोर-डं गर प्यास के मारे मरने लगते लेिकन बािरश का
कहीं नाम िनशान नहीं, ऐसे में
पूजा-पाठ कथा-िवधान सब करके
अंितम उपाय के रूप में िनकलती यह इन्दर सेना |
91 लोग जब हार जाते तब
92 ूश्न१- वषार् न होने
पर लोगों की क्या िःथित हो गयी थी ?
उत्तर - वषार् न होने पर गरमी के कारण लोग लू लगने से
बेहोश होने लगे | गाँव-शहर सभी
जगह पानी का अभाव हो गया| कुएँ सूख गए, खेतों की िमट्टी सूखकर पत्थर के समान कठोर
होकर फट गयी| घरों में नलों में पानी बहत
कम आता था | पशु प्यास के मारे मरने लगे थे |
ु
ूश्न२- वषार् के दे वता कौन हैं उनको ूसन्न करने के िलए क्या उपाय िकए जाते थे ?
उत्तर -वषार् के दे वता भगवान इन्ि हैं| उनको ूसन्न करने के िलए पूजाÐपाठ, कथा-िवधान कराए
जाते थे | तािक इन्ि दे व ूसन्न होकर बादलों की सेना भेजकर झमाझम बािरश कराएँ और
लोगों के कष्ट दरू हों |
ूश्न३- वषार् कराने के अंितम उपाय के रूप में क्या िकया जाता था?
उत्तर Ðजब पूजा-पाठ कथा-िवधान सब करके लोग हार जाते थे तब अंितम उपाय के रूप में इन्दर
सेना आती थी | नंग-धडं ग, कीचड़ में लथपथ, Ôकाले मेघा पानी दे
लगाकर प्यास से सूखते गलों और सूखते खेतों के िलए
मेघों को
िनकल पड़ती थी|
पानी दे गुड़धानी देÕ की टे र
पुकारती हई
ु टोली बनाकर
ूश्न४-आशय ःपष्ट करें –
जेठ के दसतपा बीतकर आषाढ़ का पहला पखवाड़ा बीत चुका होता पर
िक्षितज में कहीं बादलों की रे ख भी नजर नहीं आती |
आशय- जेठ का महीना है , भीषण
गरमी है|
तपते हए
ु दस िदन बीत कर आषाढ का महीना
भी आधा बीत गया, पर पानी के िलए तड़पते, वषार् की आशा में आसमान की ओर ताकते लोगों
को कहीं बादल नजर
नहीं आ रहे |
14
पहलवान की ढोलक
फ़णीश्वरनाथ रे णु
पाठ का सारांश –आंचिलक कथाकार फ़णीश्वरनाथ रे णु की कहानी पहलवान की ढोलक में कहानी
के मुख्य पाऽ लुट्टन
के माता-िपता का दे हांत उसके बचपन में ही
लुट्टन को उसकी िवधवा सास
हो गया था |
अनाथ
ने पाल-पोसकर93 बड़ा िकया | उसकी सास को गाँव वाले सताते
को उसकी िवधवा सास
ने पाल-पोसकर बड़ा िकया | उसकी सास को गाँव वाले सताते थे |
लोगों से बदला लेने के िलए कुँती के दाँवपेंच
सीखकर कसरत करके लुट्टन पहलवान बन
गया |
एक बार लुट्टन ँयामनगर मेला दे खने गया जहाँ ढोल की आवाज और कुँती के दाँवपेंच
दे खकर उसने जोश में आकर नामी पहलवान चाँदिसंह को चुनौती दे दी | ढोल की आवाज से
ूेरणा पाकर लुट्टन ने दाँव लगाकर चाँद िसंह को पटककर हरा िदया और राज पहलवान बन
गया | उसकी ख्याित दरू-दरू तक फ़ैल गयी| १५ वषोर्ं तक
पहलवान अजेय बना रहा| उसके दो
पुऽ थे| लुट्टन ने दोनों बेटों को भी पहलवानी के गुर िसखाए| राजा की मृत्यु के बाद नए
राजकुमार
ने गद्दी संभाली। राजकुमार को घोड़ों की रे स का शौक था । मैनेजर ने नये राजा को
भड़काया, पहलवान और उसके दोनों बेटों के भोजनखचर् को भयानक और िफ़जूलखचर् बताया,
फ़लःवरूप नए राजा ने कुँती को बंद करवा िदया और पहलवान लुट्टनिसंह को उसके दोनों बेटों
के साथ महल से िनकाल िदया।
राजदरबार से िनकाल िदए जाने के बाद लुट्टन िसंह अपने
दोनों बेटों के साथ गाँव में
झोपड़ी बनाकर रहने लगा और गाँव के लड़कों को कुँती िसखाने लगा| लुट्टन का ःकूल ज्यादा
िदन
गाँव
नहीं चला और जीिवकोपाजर्न के िलए उसके दोनों बेटों को मजदरी
ू करनी पड़ी| इसी दौरान
में अकाल और महामारी के कारण ूितिदन लाशें उठने लगी| पहलवान महामारी से डरे हए
ु
लोगों को ढोलक बजाकर बीमारी से लड़ने की संजीवनी ताकत दे ता था| एक िदन पहलवान के
दोनों बेटे भी महामारी की चपेट में आकर मर गए पर उस रात भी पहलवान ढोलक बजाकर
लोगों को
िहम्मत बंधा रहा था | इस घटना के चार-पाँच िदन बाद पहलवान की भी मौत हो
जाती है|
पहलवान की ढोलक, व्यवःथा के बदलने के साथ लोक कलाकार के अूासंिगक हो जाने की
कहानी है । इस कहानी में लुट्टन नाम के पहलवान की िहम्मत और िजजीिवषा का वणर्न िकया
गया है । भूख और महामारी, अजेय लुट्टन की पहलवानी को फ़टे ढोल में बदल दे ते हैं । इस
करुण ऽासदी में पहलवान लुट्टन कई सवाल छोड़ जाता है िक कला का कोई ःवतंऽ अिःतत्व है
या कला केवल व्यवःथा की मोहताज है ?
ूश्न1- लुट्टन को पहलवान बनने की ूेरणा कैसे िमली ?
उत्तर- लुट्टन जब नौ साल का
था तो उसके माता-िपता का दे हांत
उसकी शादी हो चुकी थी| अनाथ लुट्टन को उसकी िवधवा सास
हो गया था |
सौभाग्य से
ने पाल-पोसकर बड़ा िकया |
उसकी सास को गाँव वाले परे शान करते थे| लोगों से बदला लेने के िलए उसने पहलवान बनने
की ठानी| धारोंण दध
ू पीकर, कसरत कर उसने अपना बदन गठीला और ताकतवर बना िलया |
कुँती के दाँवपेंच सीखकर लुट्टन पहलवान बन गया |
94 ूश्न2- रात के भयानक सन्नाटे में लुट्टन की ढोलक क्या किरँमा करती थी?
उत्तर- रात के भयानक सन्नाटे मेंलट्
ु टन की ढोलक महामारी से जूझते लोगों को िहम्मत बँधाती
थी | ढोलक की आवाज से रात की िवभीिषका और सन्नाटा कम होता था| महामारी से पीिड़त
लोगों की नसों में िबजली सी दौड़ जाती थी, उनकी आँखों
के सामने दं गल का दृँय
साकार हो
जाता था और वे अपनी पीड़ा भूल खुशी-खुशी मौत को गले लगा लेते थे। इस ूकार ढोल की
आवाज, बीमार-मृतूाय गाँववालों की नसों में संजीवनी शिक्त को भर बीमारी से लड़ने की ूेरणा
दे ती थी।
ूश्न3- लुट्टन ने सवार्िधक िहम्मत कब िदखाई ?
उत्तर- लुट्टन िसंह
ने सवार्िधक
िहम्मत तब िदखाई जब दोनों बेटों की मृत्यु पर वह रोया नहीं
बिल्क िहम्मत से काम लेकर अकेले उनका अंितम संःकार िकया| यही नहीं, िजस िदन पहलवान
के दोनों बेटे महामारी की चपेट में आकर मर गए पर उस रात को भी पहलवान ढोलक बजाकर
लोगों को
िहम्मत बँधा रहा था| ँयामनगर के दं गल में पूरा जनसमुदाय चाँद िसंह के पक्ष में
था चाँद िसंह
को हराते समय लुट्टन ने
िसंह को िचत कर िदया |
िहम्मत िदखाई और िबना हताश हए
ु दं गल में चाँद
ूश्न4- लुट्टन िसंह राज पहलवान कैसे बना?
उत्तर- ँयामनगर के राजा कुँती के शौकीन थे। उन्होंने दं गल का
आयोजन िकया। पहलवान
लुट्टन िसंह भी दं गल दे खने पहँु चा । चांदिसंह नामक पहलवान जो शेर के बच्चे के नाम से
ूिसद्ध था, कोई भी पहलवान उससे िभड़ने की िहम्मत नहीं करता था। चाँदिसंह अखाड़े में
अकेला गरज रहा था। लुट्टन िसंह
ने चाँदिसंह को चुनौती दे दी और चाँदिसंह से िभड़
गया।ढ़ोल की आवाज सुनकर लुट्टन की नस-नस में जोश भर गया।उसने चाँदिसंह को चारों
खाने िचत कर िदया। राजासाहब ने लुट्टन की वीरता से ूभािवत होकर उसे राजपहलवान बना
िदया।
ूश्न5- पहलवान की अंितम इच्छा क्या थी ?
उत्तर- पहलवान की अंितम इच्छा थी िक उसे िचता पर पेट के बल िलटाया जाए क्योंिक वह
िजंदगी में कभी िचत नहीं हआ
था| उसकी दसरी
इच्छा थी िक उसकी िचता को आग दे ते समय
ू
ु
ढोल अवँय बजाया जाए |
ूश्न6- ढोल की आवाज और लुट्टन के में
दाँवपेंच संबंध बताइए-
उत्तर- ढोल की आवाज और लुट्टन के दाँवपेंच में
•
चट धा, िगड़ धा→ आजा िभड़ जा |
•
चटाक चट धा→ उठाकर पटक दे |
95 संबंध -
•
चट िगड़ धा→मत डरना |
•
धाक िधना ितरकट ितना→ दाँव काटो , बाहर हो जाओ |
•
िधना िधना, िधक िधना→ िचत करो
गद्यांश-आधािरत अथर्महण-संबंिधत ूश्नोत्तर
अँधेरी रात चुपचाप आँसू बहा रही थी | िनःतब्धता करुण िससिकयों और आहों को
अपने हृदय में ही बल पूवक
र् दबाने की चेष्टा कर रही थी | आकाश में तारे चमक रहे थे | पृथ्वी
ू कर यिद कोई भावुक तारा पृथ्वी पर आना भी
पर कहीं ूकाश का नाम नहीं| आकाश से टट
चाहता तो उसकी ज्योित और शिक्त राःते में ही शेष हो जाती थी | अन्य तारे उसकी भावुकता
अथवा असफलता पर िखलिखलाकर हँ स पड़ते थे | िसयारों का बंदन और पेचक की डरावनी
आवाज रात
की िनःतब्धता को भंग करती थी | गाँव की झोपिडयों से कराहने और कै करने
की आवाज, हरे राम हे भगवान की टे र सुनाई पड़ती थी| बच्चे भी िनबर्ल कंठों से माँ Ðमाँ
पुकारकर रो पड़ते थे |
(क)अँधेरी रात को आँसू बहाते हए
ु क्यों िदखाया गया है ?
उत्तर– गाँव में है जा और मलेिरया फैला हआ
था | महामारी की चपेट में आकार लोग मर रहे थे
ु
|चारों ओर मौत का सनाटा छाया था इसिलए अँधेरी रात भी चुपचाप आँसू बहाती सी ूतीत हो
रही थी|
(ख)
तारे के माध्यम से लेखक क्या कहना चाहता है ?
उत्तर– तारे के माध्यम से लेखक कहना चाहता है िक अकाल और महामारी से ऽःत गाँव वालों
की पीड़ा को दरू करने वाला कोई नहीं था | ूकृ ित भी गाँव वालों के दःख
से दखी
थी| आकाश
ु
ु
ू कर यिद कोई भावुक तारा पृथ्वी पर आना भी चाहता तो उसकी ज्योित और शिक्त राःते
से टट
में ही शेष हो जाती थी |
(ग) रात की िनःतब्धता को कौन भंग करता था ?
उत्तर– िसयारों की चीख-पुकार, पेचक की डरावनी आवाजें और कुत्तों का सामूिहक रुदन िमलकर
रात के सन्नाटे को भंग करते थे |
(घ) झोपिड़यों से कैसी आवाजें आ रही हैं और क्यों?
उत्तर– झोपिड़यों से रोिगयों के कराहने, कै करने और रोने की आवाजें आ रही हैं क्योंिक गाँव के
लोग मलेिरया और है जे से पीिड़त थे | अकाल के कारण अन्न की कमी हो गयी थी| औषिध
और पथ्य न िमलने के कारण लोगों की हालत इतनी बुरी थी िक कोई
लगाता था तो कोई दबर्
ु ल कंठ से माँÐमाँ पुकारता था |
96 भगवान को पुकार
15
चालीर् चैिप्लन यानी हम सब
लेखक-िवंणु खरे
के बहत
पाठ का सारांश –चालीर् चैिप्लन ने हाःय कलाकार के रूप में पूरी दिनया
ु
ु बड़े दशर्क वगर्
को हँ साया है | उनकी िफल्मों ने िफल्म कला को लोकतांिऽक बनाने के साथ-साथ दशकों की वगर्
और वणर्-व्यवःथा को भी तोड़ा | चालीर् ने कला में बुिद्ध की अपेक्षा भावना को महत्त्व िदया है |
बचपन के संघषोर्ं ने चालीर् के भावी िफल्मों की भूिम तैयार कर दी थी| भारतीय कला और
सौंदयर्शास्तर् में करुणा का हाःय में पिरवतर्न भारतीय परम्परा में नहीं िमलता लेिकन चालीर् एक
ऐसा जादई
ु व्यिक्तत्व है जो हर दे श, संःकृ ित और सभ्यता को अपना सा लगता
हैं| भारतीय
जनता ने भी उन्हें सहज भाव से ःवीकार िकया है| ःवयं पर हँ सना चालीर् ने ही िसखाया|
भारतीय िसनेमा जगत के सुूिसद्ध कलाकार राजकपूर को चालीर् का भारतीयकरण कहा गया है |
चालीर् की अिधकांश िफ़ल्में मूक हैं इसिलए उन्हें अिधक मानवीय होना पड़ा | पाठ में हाःय
िफल्मों के महान अिभनेता ‘चालीर् चैिप्लन’ की जादई
ु
िवशेषताओं का उल्लेख िकया गया है
िजसमें उसने करुणा और हाःय में सामंजःय ःथािपत कर िफ़ल्मों को सावर्भौिमक रूप ूदान
िकया।
ूश्न१ -चालीर् के जीवन पर ूभाव डालने वाली मुख्य घटनाएँ कौन सी थी ?
उत्तर- चालीर् के जीवन में दो ऐसी घटनाएँ घटीं िजन्होंने उनके भावी जीवन पर बहत
ु ूभाव डाला
|
पहली घटना - जब चालीर् बीमार थे उनकी माँ उन्हें ईसा मसीह की जीवनी पढ़कर सुना रही थी
| ईसा के सूली पर चढ़ने के ूसंग तक आते-आते माँ-बेटा दोनों ही रोने लगे| इस घटना ने चालीर्
को ःनेह, करुणा और मानवता जैसे उच्च जीवन मूल्य िदए |
दसरी
घटना है Ð बालक चालीर् कसाईखाने के पास रहता था| वहाँ सैकड़ों जानवरों को रोज मारा
ू
जाता था| एक िदन एक भेड़ वहाँ से भाग िनकली| भेड़ को पकड़ने की कोिशश में कसाई कई
बार िफसला| िजसे दे खकर लोग हं सने लगे, ठहाके लगाने लगे| जब भेड़ को कसाई ने पकड़
िलया तो बालक चालीर् रोने लगा| इस घटना ने उसके भावी िफल्मों में ऽासदी और हाःयोत्पादक
तत्वों की भूिमका तय कर दी |
ूश्न२ Ð आशय ःपष्ट कीिजए–
चैिप्लन ने िसफर् िफल्मकला को ही लोकतांिऽक नही बनाया बिल्क दशर्कों की वगर् तथा वणर्व्यवःथा को भी तोड़ा|
97 उत्तर- लोकतांिऽक बनाने का अथर् है िक िफल्म कला को सभी के िलए लोकिूय बनाना और वगर्
और वणर्-व्यवःथा को तोड़ने का आशय है - समाज में ूचिलत अमीर-गरीब, वणर्, जाितधमर् के
भेदभाव को समाप्त करना |चैिप्लन का चमत्कार यह है िक उन्होंने िफल्मकला को िबना िकसी
भेदभाव के सभी लोगों तक पहँु चाया| उनकी िफल्मों ने समय भूगोल और संःकृ ितयों की सीमाओं
को लाँघ कर सावर्भौिमक लोकिूयता हािसल की | चालीर् ने यह िसद्ध कर िदया िक कला
ःवतन्ऽ होती है , अपने िसद्धांत ःवयं बनाती है |
ूश्न३Ð चालीर् चैिप्लन की िफल्मों में िनिहत ऽासदी/करुणा/हाःय का सामंजःय भारतीय कला और
सौंदयर्शास्तर् की पिरिध में क्यों नहीं आता?
उत्तर- चालीर् चैिप्लन की िफल्मों में िनिहत ऽासदी/करुणा/हाःय का सामंजःय भारतीय कला और
सौंदयर्शास्तर् की पिरिध में नहीं आताक्योंिक भारतीय रस-िसद्धांत में करुणा और हाःय का मेल
नहीं िदखाया जाता क्योंिक भारतीय सौंदयर्शास्तर् में करुणरस और हाःय रस को परःपर िवरोधी
माना गया है अथार्त जहां करुणा है वहाँ हाःय नहीं हो सकता। भारत में ःवयं पर हँ सने की
परं परा नहीं है परं तु चालीर् के पाऽ अपने पर हँ सतेÐहँ साते हैं । चालीर् की िफ़ल्मों के दृँय हँ सातेहँ साते रुला दे ते हैं तो कभी करुण दृँय के बाद अचानक ही हँ सने पर मजबूर कर दे ते हैं ।
ूश्न४Ð चालीर् के िफल्मों की िवशेषताएँ बताइए |
उत्तर- चालीर् की िफ़ल्मों में हाःय और करुणा का अद्भत
ु सामंजःय है । उनकी िफ़ल्मों में भाषा का
ूयोग बहत
ु कम है । चालीर् की िफ़ल्मों में बुिद्ध
की अपेक्षा भावना का महत्त्व अिधक है । उनकी
िफ़ल्मों में सावर्भौिमकता है । चालीर् िकसी भी संःकृ ित को िवदे शी नही लगते। चालीर् सबको अपने
लगते है । चालीर् ने िफ़ल्मों को लोकतांिऽक बनाया और िफ़ल्मों में वगर् तथावणर्-व्यवःथा को
तोड़ा।
अपनी िफल्मों में चालीर् सदै व िचर युवा िदखता है ।
गद्यांश-आधािरत अथर्महण-संबंधी ूश्नोत्तर
गद्यांश संकेत Ðचालीर् चैिप्लन यानी हम सब (पृष्ठ १२० )
यिद यह वषर् चैिप्लन की ..........................................काफी कुछ कहा जाएगा |
ूश्न (क)-िवकासशील दे शों में चैिप्लन क्यों मशहर
ू हो रहे हैं ?
उत्तर - िवकासशील दे शों में जैसे-जैसे टे लीिवजन और वीिडयो का ूसार हो रहा है , लोगों को
उनकी िफल्मों को दे खने का अवसर िमल रहा है |एक बहत
ु बड़ा वगर् नए िसरे से चालीर् को घड़ी
सुधारते और जूते खाने की कोिशश करते दे ख रहा है , इसीिलए चालीर् िवकासशील दे शों में
लोकिूय हो रहे हैं |
(ख)- पिश्चम में चालीर् का पुनजीर्वन कैसे होता रहता है ?
98 उत्तर - पिश्चम में चालीर् की िफल्मों का ूदशर्न होता रहता है| उनकी कला से ूेरणा पाकर हाःय
िफ़ल्में बनती रहती हैं | उनके द्वारा िनभाए िकरदारों की नकल, अन्य कलाकार करते हैं | पिश्चम
में चालीर् का पुनजीर्वन होता रहता है|
(ग)- चालीर् को लोग बुढ़ापे तक क्यों याद रखेंगे ?
उत्तर Ðहाःय कलाकार के रूप में लोग चालीर् को बुढ़ापे तक याद रखेंगे क्योंिक उनकी कला समय,
भूगोल और संःकृ ितयों की सीमाओं को लाँघकर लाखों लोगों को हँ सा रही है|
(घ)- चालीर् की िफल्मों के बारे में काफी कुछ कहा जाना क्यों बाक़ी है ?
उत्तर Ðचैिप्लन की ऐसी कुछ िफ़ल्में या इःतेमाल न की गयी रीलें
िमली हैं िजनके बारे में कोई
नहीं जानता था | चालीर् की भावनाूधान हाःय िफल्मों ने कला के नए ूितमान ःथािपत िकए हैं
अत: चालीर् की िफल्मों के बारे में अभी काफी कुछ कहा जाना बाक़ी है|
16
नमक
लेिखका - रिजया सज्जाद जहीर
सारांश -‘नमक’भारत-पाक िवभाजन पर िलिखत मािमर्क कहानी है | िवःथािपत हए
ु लोगों में
अपनेÐअपने जन्म ःथानों के ूित आज भी लगाव है| धािमर्क आधार पर बनी राष्टर्-राज्यों की
सीमा-रे खाएँ उनके अंतमर्न को अलग नहीं कर पाई हैं | भारत में रहने वाली िसख बीवी लाहौर
को अपना वतन मानती है और भारतीय कःटम अिधकारी, ढाका के नािरयल पानी को यादकर
उसे सवर्ौष्ठ
े बताता है । दोनो दे शों के नागिरकों के बीच मुहब्बत का नमकीन ःवाद आज भी
कायम है इसीिलए सिफ़या भारत में रहने वाली अपनी मुह
ँ बोली माँ, िसख बीवी के िलए लाहौरी
नमक लाने के िलए कःटम और कानून की परवाह नहीं करती।
ूश्न1- ÔनमकÕ पाठ के आधार पर बताइए िक सिफया और उसके भाई के िवचारों में क्या अंतर
था?
उत्तर- १-सिफया भावनाओं को बहतमहत्त्व
दे ती है पर उसका भाई बौिद्धक ूवृित्त का है , उसकी
ु
दृिष्ट में कानून भावनाओं से ऊपर है|
२-सिफया मानवीय संबंधों को बहत
ु महत्त्व दे ती है जबिक उसका भाई अलगाववादी िवचारधारा
का है , िहःसे-बखरे की बात करता है |
99 ३-सिफया का
भाई कहता है िक अदीबों(सािहत्यकार) का िदमाग घूमा हआ
होता है जबिक
ु
सिफया जो ःवयं अदीब है उसका मानना है िक अगर सभी लोगों का िदमाग अदीबों की तरह
घूमा हआ
होता तो दिनया
कुछ बेहतर हो जाती |
ु
ु
ूश्न२- नमक ले जाते समय सिफ़या के मन में क्या दिवधा
थी ?
ु
उत्तर-सिफया सैयद मुसलमान थी जो हर हाल में अपना वायदा िनभाते हैं| पािकःतान से लाहौरी
नमक ले जाकर वह अपना वायदा पूरा करना चाहती थी परन्तु जब उसे पता चला िक कःटम
के िनयमों के अनुसार सीमापार नमक ले जाना विजर्त है तो वह दिवधा
में पड़ गई | सिफ़या का
ु
द्वं द्व यह था िक वह अपनी िसख माँ के िलए नमक, कःटम अिधकािरयों को बताकरले जाए या
िछपाकर|
ूश्न३- पाठ के आधार पर सिफया की चािरिऽक िवशेषताएँ बताइए |
उत्तर संकेत –
१- भावुक
२- ईमानदार
३- दृढ़िनश्चयी
४- िनडर
५- वायदे को िनभाने वाली
६- मानवीय मूल्यों को सवोर्पिर मानने वाली सािहत्यकार
ूश्न ४- ÔनमकÕ पाठ में आए िकरदारों के माध्यम से ःपष्ट कीिजए िक आज भी भारत और
पािकःतान की जनता के बीच मुहब्बत का नमकीन ःवाद घुला हआ
है |
ु
उत्तर Ð भले ही राजनीितक और धािमर्क आधार पर भारत और पािकःतान को भौगोिलक रूप से
िवभािजत कर िदया गया है लेिकन दोनों दे शों के लोगों के हृदय में आज भी पारःपिरक
भाईचारा, सौहािर् , ःनेह और सहानुभिू त िवद्यमान है | राजनीितक तौर पर भले ही संबंध
तनावपूणर् हों पर सामािजक तौर पर आज भी जनता के बीच मुहब्बत का नमकीन ःवाद घुला
हआ
है | अमृतसर में रहने वाली िसख बीबी लाहौर को अपना वतन कहती है और लाहौरी नमक
ु
का ःवाद नहीं भुला पाती | पािकःतान का कःटम अिधकारी नमक की पुिड़या सिफया को वापस
दे ते हए
ु कहता है
जामा मिःजद की सीिढ़यों को मेरा सलाम कहना | भारतीय सीमा पर तैनात
कःटम अिधकारी ढाका की जमीन को और वहाँ के पानी के ःवाद को नहीं
100 भूल पाता |
गद्यांश-आधािरत अथर्महण संबंिधत ूश्नोत्तर
गद्यांश संकेत- पाठÐ नमक (पृष्ठ १३४)
सिफया कःटम के जंगले से ...........................दोनों के हाथों में थी |
(क)सिफया कःटम के जँगले से िनकलकर दसरे
प्लेटफामर् पर आ गयी वे वहीं खड़े रहे Ð इस
ू
वाक्य में ÔवेÕ शब्द का ूयोग िकसके िलए िकया गया है ?
उत्तर- यहाँ ÔवेÕ शब्द का ूयोग पािकःतानी कःटम अिधकारी के िलए िकया गया है जो िवभाजन
से पूवर् िदल्ली में रहते थे और आज भी िदल्ली को ही अपना वतन मानते हैं |
(ख)
प्लेटफामर् पर सिफया को िवदा करने कौन-कौन आए थे, उन्होंने सिफया को कैसे
िवदाई दी ?
उत्तर- प्लेटफामर् पर सिफया को िवदा करने उसके बहत
ु सारे िमऽ, सगे संबंधी और भाई
आए
थे| उन्होंने ठं डी साँस भरते हए
ु , िभंचे हए
ु होंठों के साथ, आँसू बहाते हए
ु सिफया को िवदाई दी |
(ग) अटारी में रे लगाड़ी में क्या पिरवतर्न हए
ु ?
उत्तर- अटारी में रे लगाड़ी से पािकःतानी पुिलस उतरी और िहन्दःतानी
पुिलस सवार हो गई|
ु
(घ) कौन सी बात सिफया की समझ में नहीं आ रही थी ?
उत्तर- दोनों ओर एक सी जमीन, एक जैसा आसमान, एक सी भाषा, एक सा पहनावा और एक
सी सूरत के लोग िफर भी दोनों के हाथों में भरी हई
ू ें हैं |
ु बंदक
17
िशरीष के फूल
आचायर् हजारी ूसाद िद्ववेदी
सारांश –Ôआचायर् हजारी ूसाद िद्ववेदीÕ िशरीष को अद्भत
ु अवधूत मानते हैं , क्योंिक संन्यासी की
भाँित वह सुख-दख
ु की िचंता नहीं करता। गमीर्, लू, वषार् और आँधी में भी अिवचल खड़ा रहता
है । िशरीष के फ़ूल के माध्यम से मनुंय की अजेय िजजीिवषा, धैयश
र् ीलता और कतर्व्यिनष्ठ बने
रहने के मानवीय मूल्यों को ःथािपत िकया गया है ।लेखक ने िशरीष के कोमल फूलों और कठोर
फलों के द्वारा ःपष्ट िकया है िक हृदय की कोमलता बचाने के िलए कभी-कभी व्यवहार की
कठोरता भी आवँयक हो जाती है| महान किव कािलदास और कबीर भी िशरीष की तरह
बेपरवाह, अनासक्त और सरस थे तभी उन्होंने इतनी सुन्दर रचनाएँ संसार को दीं|
गाँधीजी के
व्यिक्तत्व में भी कोमलता और कठोरता का अद्भत
ु संगम था | लेखक सोचता है िक हमारे दे श में
जो मार-काट, अिग्नदाह, लूट-पाट, खून-खच्चर का बवंडर है , क्या वह दे श को िःथर नहीं रहने
101 रहने दे गा? गुलामी, अशांित और िवरोधी वातावरण के बीच अपने िसद्धांतों की रक्षा करते हए
ु
गाँधीजी जी िःथर रह सके थे तो दे श भी रह सकता है । जीने की ूबल अिभलाषा के कारण
िवषम पिरिःथतयों मे भी यिद िशरीष िखल सकता है तो हमारा दे श भी िवषम पिरिःथितयों में
िःथर रह कर िवकास कर सकता है ।
ूश्न1-िसद्ध कीिजए िक िशरीष कालजयी अवधूत की भाँित जीवन की अजेयता के मंऽ का ूचार
करता है ?
उत्तर- िशरीष कालजयी अवधूत की भाँित जीवन की अजेयता के मंऽ का ूचार करता है|
जब
पृथ्वी अिग्न के समान तप रही होती है वह तब भी कोमल फूलों से लदा लहलहाता रहता
है|बाहरी गरमी, धूप, वषार् आँधी, लू उसे ूभािवत नहीं करती। इतना ही नहीं वह लंबे समय तक
िखला रहता है | िशरीष िवपरीत पिरिःथितयों में भी धैयश
र् ील
िजजीिवषा के साथ िनःपृह भाव से
रहने तथा अपनी अजेय
ूचंड गरमी में भी अिवचल खड़ा रहता है ।
ूश्न२-आरग्वध (अमलतास) की तुलना िशरीष से क्यों नहीं की जा सकती ?
उत्तर- िशरीष के फूल भयंकर गरमी में िखलते हैं और आषाढ़ तक िखलते रहते हैं जबिक
अमलतास का फूल केवल पन्िह-बीस िदनों के िलए िखलता है | उसके बाद अमलतास के फूल
झड़ जाते हैं और पेड़ िफर से ठँू ठ का ठँू ठ हो जाता है | अमलतास अल्पजीवी है | िवपरीत
पिरिःथितयों को झेलता हआ
ऊंण वातावरण को हँ सकर झेलता हआ
िशरीष दीघर्जीवी रहता
ु
ु
है
| यही कारण है िक िशरीष की तुलना अमलतास से नहीं की जा सकती |
ूश्न३-िशरीष के फलों को राजनेताओं का रूपक क्यों िदया गया है ?
उत्तर- िशरीष के फल उन बूढ़े, ढीठ और पुराने राजनेताओं के ूतीक हैं जो अपनी कुसीर् नहीं
छोड़ना चाहते | अपनी
अिधकार-िलप्सा के िलए नए युवा नेताओं को आगे नहीं आने दे ते |
िशरीष के नए फलों को जबरदःती पुराने फलों को धिकयाना पड़ता है | राजनीित में भी नई युवा
पीढ़ी, पुरानी पीढ़ी को हराकर ःवयं सत्ता सँभाल लेती है |
ूश्न४- काल दे वता की मार से बचने का क्या उपाय बताया गया है ?
उत्तर- काल दे वता िक मार से बचने का अथर् है Ð मृत्यु से बचना | इसका एकमाऽ उपाय यह है
िक मनुंय िःथर न हो| गितशील, पिरवतर्नशील रहे | लेखक के अनुसार
िजनकी चेतना सदा
ऊध्वर्मख
ु ी (आध्यात्म की ओर) रहती है , वे िटक जाते हैं |
ूश्न५- गाँधीजी और िशरीष की समानता ूकट
कीिजए |
उत्तर- िजस ूकार िशरीष िचलिचलाती धूप, लू, वषार् और आँधी में भी अिवचल खड़ा रहता है ,
अनासक्त रहकर अपने वातावरण से रस खींचकर सरस, कोमल बना रहता है , उसी ूकार गाँधी
102 जी ने भी अपनी आँखों के सामने आजादी के संमाम में अन्याय, भेदभाव और िहं सा को झेला |
उनके कोमल मन में एक ओर िनरीह जनता
के ूित असीम करुणा जागी वहीं वे अन्यायी
शासन के िवरोध में डटकर खड़े हो गए |
गद्यांश-आधािरत अथर्महण संबंिधत ूश्नोत्तर
गद्यांश संकेत- पाठ Ð िशरीष के फूल (पृष्ठ १४७)
कािलदास
सौंदयर्
के
..............................................................................................................वह इशारा है |
(क)कािलदास की सौंदयर्Ðदृिष्ट की क्या िवशेषता थी ?
उत्तर-कािलदास की सौंदयर्Ðदृिष्ट बहत
ू र् थी| वे केवल बाहरी रूप-रं ग और
ु सूआम, अंतभेर्दी और संपण
आकार को ही नहीं दे खते थे बिल्क अंतमर्न की सुद
ं रता के भी पारखी थे| कािलदास की
सौंदयर्
शारीिरक और मानिसक दोनों िवशेषताओं से युक्त था |
(ख)
अनासिक्त का क्या आशय है ?
उत्तर- अनासिक्त का
आशय है - व्यिक्तगत सुख-दःख
और राग-द्वे ष से परे रहकर सौंदयर् के
ु
वाःतिवक ममर् को जानना |
(ग) कािलदास, पंत और रवींिनाथ टै गोर में कौन सा गुण समान था?
महाकिव कािलदास, सुिमऽानंदन पंत और गुरुदे व रवींिनाथ टै गोर तीनों िःथरूज्ञ और अनासक्त
किव थे | वे िशरीष के समान सरस और मःत अवधूत थे |
(घ) रवींिनाथ राजोद्यान के िसंहद्वार के बारे में क्या संदेश दे ते हैं ?
राजोद्यान के बारे में रवींिनाथ कहते हैं राजोद्यान का िसंहद्वार िकतना ही सुद
ं र और गगनचुम्बी
क्यों ना हो, वह अंितम पड़ाव नहीं है| उसका सौंदयर् िकसी और उच्चतम सौंदयर् की ओर िकया
गया संकेत माऽ है िक असली सौंदयर् इसे पार करने के बाद है अत: राजोद्यान का िसंहद्वार हमें
आगे बढ़ने की ूेरणा दे ता है |
18ौम-िवभाजन और जाित-ूथा
डॉ० भीमराव अंबेडकर
सारांश – इस पाठ
में लेखक ने जाितवाद के आधार पर िकए जाने वाले भेदभाव को सभ्य
समाज के िलए हािनकारक बताया है | जाित आधािरत ौम िवभाजन को अःवाभािवक और
मानवता िवरोधी बताया गया है । यह सामािजक भेदभाव को बढ़ाता है । जाितूथा आधािरत ौम
103 िवभाजन में व्यिक्त की रुिच को महत्त्व नहीं िदया जाता फलःवरूप िववशता के साथ अपनाए
गए
पेशे में
कायर्-कुशलता नहीं आ पाती | लापरवाही से िकए गए कायर् में गुणवत्ता नहीं आ
पाती और आिथर्क िवकास बुरी तरह ूभािवत होता है| आदशर् समाज की नींव समता, ःवतंऽता
और बंधत्ु व पर िटकी होती है । समाज के सभी सदःयों से अिधकतम उपयोिगता ूाप्त करने के
िलए सबको अपनी क्षमता को िवकिसत करने
ःवतंऽता होनी चािहए|
तथा
रुिच के अनुरूप व्यवसाय चुनने की
राजनीितज्ञ को अपने व्यवहार में एक व्यवहायर् िसद्धांत की आवँयकता
रहती है और यह व्यवहायर् िसद्धांत यही होता है िक सब मनुंयों के साथ समान व्यवहार िकया
जाए।
ूश्न1-डॉ० भीमराव अंबेडकर जाितूथा को ौम-िवभाजन का ही रूप क्यों नहीं मानते हैं ?
उत्तर –
१- क्योंिक यह िवभाजन अःवाभािवक है |
२- यह मनुंय की रुिच पर आधािरत नहीं है |
३- व्यिक्त की क्षमताओं की उपेक्षा की जाती है |
४- व्यिक्त के जन्म से पहले ही उसका पेशा िनधार्िरत कर िदया जाता है |
५- व्यिक्त को अपना व्यवसाय बदलने की अनुमित नहीं दे ती |
ूश्न२- दासता की व्यापक पिरभाषा दीिजए |
उत्तर – दासता केवल कानूनी पराधीनता नहीं है|
सामािजक दासता की िःथित में कुछ व्यिक्तयों
को दसरे
लोगों के द्वारा तय िकए गए व्यवहार और कतर्व्यों का पालन करने को िववश होना
ू
पड़ता है | अपनी इच्छा के िवरुद्ध पैतक
ृ पेशे अपनाने पड़ते हैं |
ूश्न३- मनुंय की क्षमता िकन बातों पर िनभर्र रहती है ?
उत्तर –मनुंय की क्षमता मुख्यत: तीन बातों पर िनभर्र रहती है १- शारीिरक वंश परं परा
२- सामािजक उत्तरािधकार
३- मनुंय के अपने ूयत्न
लेखक का मत है िक शारीिरक वंश परं परा तथा सामािजक उत्तरािधकार िकसी के वश में नहीं है
परन्तु
मनुंय के अपने ूयत्न उसके अपने वश में है | अत: मनुंय की मुख्य क्षमता- उसके
अपने ूयत्नों को बढ़ावा िमलना चािहए |
104 ूश्न४- समता का आशय ःपष्ट करते हए
ु बताइए िक राजनीितज्ञ पुरूष के संदभर् में समता को
कैसे ःपष्ट िकया गया है ?
जाित, धमर्, संूदाय से ऊपर उठकर मानवता अथार्त ् मानव माऽ के ूित समान व्यवहार ही
समता है । राजनेता के पास असंख्य लोग आते हैं , उसके पास पयार्प्त जानकारी नहीं होती सबकी
सामािजक पृष्ठभूिम क्षमताएँ, आवँयकताएँ जान पाना उसके िलए संभव नहीं होता अतः उसे
समता और मानवता के आधार पर व्यवहार के ूयास करने चािहए ।
गद्यांश-आधािरत अथर्महण संबंिधत ूश्नोत्तर
गद्यांश संकेत-पाठ ौम िवभाजन और जाित ूथा (पृष्ठ १५३)
यह
िवडम्बना.......................................................................................................................ब
ना दे ती है |
ूश्न १-ौम िबभाजन िकसे कहते हैं ?
उत्तर: ौम िवभाजन का अथर् है Ð मानवोपयोगी कायोर्ं का वगीर्करण करना| ूत्येक कायर् को
कुशलता से करने के िलए योग्यता के अनुसार िविभन्न कामों को आपस में बाँट लेना | कमर्
और मानव-क्षमता पर आधािरत यह िवभाजन सभ्य समाज के िलए आवँयक है |
ूश्न २ - ौम िवभाजन और ौिमक-िवभाजन का अंतर ःपष्ट कीिजए |
उत्तर- ौम िवभाजन में क्षमता और कायर्-कुशलता के आधार पर काम का बँटवारा होता है ,
जबिक ौिमक िवभाजन में लोगों को जन्म के आधार पर बाँटकर पैतक
ृ पेशे को अपनाने के िलए
बाध्य िकया जाता है| ौम-िवभाजन में व्यिक्त अपनी रुिच के अनुरूप व्यवसाय का चयन करता
है | ौिमक-िवभाजन में व्यवसाय का चयन और व्यवसाय-पिरवतर्न की भी अनुमित नहीं होती,
िजससे समाज में ऊँच नीच का भेदभाव पैदा करता है , यह अःवाभािवक िवभाजन है |
ूश्न ३ Ðलेखक ने िकस बात को िवडम्बना कहा है ?
उत्तर : लेखक कहते हैं िक आज के वैज्ञािनक युग में भी कुछ लोग ऐसे हैं जो जाितवाद का
समथर्न करते हैं और उसको सभ्य समाज के िलए उिचत मानकर उसका पोषण करते हैं| यह
बात आधुिनक सभ्य और लोकतािन्ऽक समाज के िलए िवडम्बना है |
ूश्न ४ : भारत में ऎसी कौन-सी व्यवःथा है जो पूरे िवश्व में और कहीं नहीं
है ?
उत्तर: लेखक के अनुसार जन्म के आधार पर िकसी का पेशा तय कर दे ना, जीवनभर एक ही
पेशे से बँधे रहना, जाित के आधार पर ऊँच-नीच का भेदभाव करना तथा बेरोजगारी तथा
105 भुखमरी की िःथित में भी पेशा बदलने की अनुमित न होना ऐसी व्यवःथा है जो िवश्व में कहीं
नहीं
है |
अध्ययन साममी
कक्षा बारहवीं - िवतान भाग-2
िवतान भाग -२ पुःतक में से ूश्नपऽ में तीन ूकार के ूश्न पूछे जाएँगे ूश्न १२. अित लघूत्तरात्मक तीन ूश्नों में से दो के उत्तर दे ने हैं | िनधार्िरत अंक - २ * २ = ४
ूश्न १३. दो िनबंधात्मक ूश्नों में से एक ूश्न का उत्तर| िनधार्िरत अंक Ð ५
ूश्न १४. तीन लघूत्तरात्मकूश्नों में से दो ूश्नों के उत्तर| िनधार्िरत अंक Ð ३ * २ = ६
अिधक अंक ूािप्त हे तु ध्यान दे ने योग्य बातें –
१. पूरा पाठ पढ़ें तथा पाठ की िवषयवःतु, ूमुख पाऽ तथा संदेश की ओर अवँय ध्यान दें |
२. ूश्नों के उत्तर दे ते समय अनावँयक न िलखें| ूश्न को ध्यानपूवक
र् पढ़कर, उसका सटीक
उत्तर दें |
३. शब्दसीमा का ध्यान रखें |
४. अंकों के अनुसार उत्तर िलखें |
५. पांच अंक वाले उत्तरों में िविभन्न िबंदओं
को उदाहरण सिहत ूःतुत करें |
ु
पाठ 1िसल्वर वैिडं ग Ð मनोहर ँयाम जोशी
पाठ का सार-िसल्वर वेिडं गÕ कहानी की रचना मनोहर ँयाम जोशी ने की है| इस पाठ के माध्यम
से पीढ़ी के अंतराल का मािमर्क िचऽण िकया गया है| आधुिनकता के दौर में, यशोधर
बाबूपरं परागत मूल्यों को हर हाल में जीिवत रखना चाहते हैं| उनका उसूलपसंद होना दफ्तर एवम
घर के लोगों के िलए सरददर् बन गया था | यशोधर बाबू को िदल्ली में अपने पाँव जमाने में
िकशनदा ने मदद की थी, अतः वे उनके आदशर् बन गए|
106 दफ्तर में िववाह की पच्चीसवीं सालिगरह के िदन ,दफ्तर के कमर्चारी, मेनन और चड्ढा उनसे
जलपान के िलए पैसे माँगते हैं | जो वे बड़े अनमने ढं ग से दे ते हैं क्योंिक उन्हें िफजूलखचीर्
पसंद नहीं |यशोधर
बाबू के तीन बेटे हैं| बड़ा बेटा भूषण, िवज्ञापन कम्पनी में काम करता है| दसरा
बेटा आई. ए.
ू
एस. की तैयारी कर रहा है और तीसरा छाऽवृित के साथ अमेिरका जा चुका है| बेटी भी डाक्टरी
की पढ़ाईं के िलए अमेिरका जाना चाहती है , वह िववाह हे तु िकसी भी वर को पसंद नहीं करती|
यशोधर बाबूबच्चों की तरक्की से खुश हैं िकंतु परं परागत संःकारों के कारण वे दिवधा
में हैं |
ु
उनकी पत्नी ने ःवयं को बच्चों की सोच के साथ ढाल िलया है| आधुिनक न होते हए
ु भी, बच्चों
के ज़ोर दे ने पर वे अिधक माडनर् बन गई है|
बच्चे घर पर िसल्वर वेिडं ग की पाटीर् रखते हैं , जो यशोधर बाबू के उसूलों के िखलाफ था| उनका
बेटा उन्हें से िसंग गाउन भेंट करता है तथा सुबह दध
ू लेने जाते समय उसे ही पहन कर जाने को
कहता है , जो उन्हें अच्छा नहीं लगता| बेटे का ज़रूरत से एयादा तनख्वाह पाना, तनख्वाह की
रकम ःवयं खचर् करना, उनसे िकसी भी बात पर सलाह न माँगना और दध
ू लाने का िजम्मा
ःवयं न लेकर उन्हें
से िसंग गाउन पहनकर दध
ू लेने जाने की बात कहना जैसी बातें, यशोधर
बाबू को बुरी लगती है| जीवन के इस मोड़ पर वे ःवयं को अपने उसूलों के साथ अकेले पाते हैं |
ूश्नोत्तर ूश्न १. अपने घर और िवद्यालय के आस-पास हो रहे उन बदलावों के बारे में िलखें जो
सुिवधाजनक और आधुिनक होते हए
ु भी बुजुगोर्ं को अच्छे नहीं लगते। अच्छा न लगने के क्या
कारण हो सकते हैं ?
उत्तरः
आधुिनक युग पिरवतर्नशील एवं अिधक सुिवधाजनक है । आज के युवा,आधुिनकता और
पिरवतर्नशीलता को महत्त्व दे ते हैं इसीिलए वे नई तकनीक और फैशन की ओर आकिषर्त होते हैं ।
वे तत्काल नयी जानकािरयाँ चाहते हैं , िजसके िलए उनके पास कम्प्यूटर, इन्टरनेट एवं मोबाइल
जैसे
आधुिनक तकनीकी साधन हैं । इनके माध्यम से वे कम समय में ज्यादा
कर लेते हैं । घर से िवद्यालय जाने के िलए अब उनके पास बिढ़या साईिकलें
जानकारी एकऽ
एवं मोटर साईिकलें
हैं । आज युवा लड़के और लड़िकयों के बीच का अन्तराल काफी कम हो गया है । पुराने जमाने में
लड़िकयाँ-लड़कों के साथ पढ़ना और िमलना-जुलना ठीक नहीं माना जाता था जैसे आज के
107 जैसे आज के पिरवेश में है । युवा लड़कों और लड़िकयों द्वारा अंग ूदशर्न आज आम बात हो गई
है । वे एक साथ दे र रात तक पािटर् याँ करते हैं । इन्टरनेट के माध्यम से
असभ्य जानकािरयाँ
ूाप्त करते हैं । ये सारी बातें उन्हें आधुिनक एवं सुिवधाजनक लगती हैं ।
दसरी
ओर इस तरह की आधुिनकताबुज़गोर्ं को रास नहीं आतीक्योंिक जब वे युवा थे,उस समय
ू
संचार के साधनों की कमी थी। पािरवािरक पृष्ठभूिम के कारण वे युवावःथामें अपनी भावनाओं को
काबू में रखते थे और अिधक िजम्मेदार होते थे। अपने से बड़ों का आदर करते थे और परं पराओं
के अनुसार चलते थे। आधुिनक पिरवेश के युवा बड़े -बूढ़ों के साथ बहत
ु कम समय व्यतीत करते
हैं इसिलए सोच एवं दृिष्टकोण में अिधक अन्तर आ गया है । इसी अन्तर को ‘पीढ़ी का अन्तर’
कहते हैं । युवा पीढ़ी की यही नई सोच बुजुगोर्ं को अच्छी नहीं लगती।
२.
यशोधर बाबू की पत्नी समय के साथ ढल सकने में सफल होती है ,लेिकन यशोधर बाबू
असफल रहते हैं ,ऐसा क्यों ?
उत्तरः यशोधर बाबू अपने आदशर् िकशनदा से अिधक ूभािवत हैं और आधुिनक पिरवेश
में बदलते हए
ु जीवन-मूल्यों और संःकारों के िवरूद्ध हैं । जबिक उनकी पत्नीअपने बच्चों के साथ
खड़ी िदखाई दे ती हैं । वह अपने बच्चों के आधुिनक दृिष्टकोण से ूभािवत हैं । इसिलए यशोधर
बाबू की पत्नी समय के साथ पिरवितर्त होती है , लेिकन यशोधर बाबू अभी भी िकशनदा के
संःकारों और परं पराओं से िचपके हए
ु हैं ।
३.’िसल्वर वैिडं ग’कहानी के आधार पर पीढ़ी के अंतराल से होने वाले पािरवािरक अलगाव पर
अपने िवचार ूकट कीिजए।
उत्तरः
सांःकृ ितक संरक्षण के िलए ःवःथ परं पराओं की सुरक्षा आवँयक है , िकंतु बदलते
समय और पिरवेश से सामंजःय की भी उपेक्षा नहीं की जानी चािहए, अन्यथा िसल्वर वैिडं ग के
पाऽों की तरह िबखराव होने लगता है ।
४.’िसल्वर वैंिडग’कहानी को ध्यान में रखते हए
ु परं पराओं और सांःकृ ितक संरक्षकों की वतर्मान
में उपादे यता पर अपने िवचार ूकट कीिजए।
108 उत्तरःपरं पराओं और संःकृ ित से ही िकसी दे श की पहचान कायम रहती है । सादगी से
जीकर गृहःथी को बचाया जा सकता है । यिद ऐसा न होता तो शायद यशोधर बाबू की संतानें
पढ़-िलखकर योग्य न बनतीं ।
५. िसल्वर वैिडं गके कथानायक यशोधर बाबू एक आदशर् व्यिक्त हैं और नई पीढ़ी द्वारा उनके
िवचारों को अपनाना ही उिचत है| इस कथन के पक्ष-िवपक्ष में तकर् दीिजए।
उत्तरः- पक्षः
को लेकर
नयी पीढ़ी के युवा उनकी सादगी और व्यिक्तत्व के कुछे क ूेरक पहलुओं
सफल व संःकारी नागिरक बन सकते हैं अन्यथा वे अपनी पहचान खो बैठेंगे।
िवपक्षःपुरानी रूिढ़यों को छोड़कर ही नए समाज व नई पीढ़ी के साथ सामंजःय िबठाया जा
सकता है अन्यथा िवलगाव सुिनिश्चत है ।
६ .
िसल्वर वैिडं ग कहानी की मूल संवेदना ःपष्ट कीिजए।
उत्तरः - संकेत-िबंद-ु पीढ़ी के अन्तराल से पैदा हई
ु िबखराव की पीड़ा।
७.
िकशनदा की कौन-सी छिव यशोधर बाबू के मन में बसी हई
ु थी?
संकेत िबन्द-ु 1.उनकी दफ्तर में िविभन्न रूपों की छिव,
2.सैर पर िनकलने वाली छिव,
३. एक आदशर्वादी व्यिक्त के रूप में ,
४. परोपकारी व्यिक्त |
८ .Ôिसल्वर वैिडं ग’कहानी का ूमुख पाऽ बार-बार िकशनदा को क्यों याद करता है ?इसे आप क्या
मानते हैं उसकी सामथ्यर् या कमजोरी?औरक्यों ?
उत्तरः
सामथ्यर् मानते हैं , वे उनके ूेरक थे, उनसे अलग अपने को सोचना भी यशोधर
बाबू के िलए मुिँकल था। जीवन का ूेरणा-ॐोत तो सदा शिक्त एवं सामथ्यर्का सजर्क होता है ।
९ . पाठ में ‘जो हआ
होगा’ वाक्य की िकतनी अथर् छिवयाँ आप खोज सकते हैं ?
ु
109 उत्तरः यशोधर बाबू यही िवचार करते हैं िक िजनके बाल-बच्चे ही नहीं होते,वे व्यिक्त अकेलेपन के
कारण ःवःथ िदखने के बाद भी बीमार-से हो जाते हैं और उनकी मृत्यु हो जाती है| िजसूकार
यशोधर बाबू अपने आपको पिरवार से कटा और अकेला पाते हैं उसीूकार अकेलेपन से मःत
होकर उनकी मृत्यु हई
ु होगी। यह भी कारण हो सकता है िक उनकी िबरादरी से घोर उपेक्षा
िमली, इस कारण वे सूख-सूख कर मर गए | िकशनदा की मृत्यु के सही कारणों का पता नहीं
चल सका। बस यशोधर बाबू यही सोचते रह गए िक िकशनदा की मृत्यु कैसे हई
ु ?िजसका उत्तर
िकसी के पास नहीं था।
१० .वतर्मान समय में पिरवार की संरचना,ःवरूप से जुड़े आपके अनुभव इस कहानी से कहाँ तक
सामंजःय िबठा पाते हैं ?
उत्तरः यशोधर बाबू और उनके बच्चों की सोच में पीढ़ीजन्य अंतराल आ गया है ।
यशोधर संःकारों से जुड़ना चाहते हैं और संयक्त
ु पिरवार की संवेदनाओं को अनुभव करते हैं
जबिक उनके बच्चे अपने आप में जीना चाहते हैं । अतः जरूरत इस बात की है िक यशोधर बाबू
को अपने बच्चों की सकारात्मक नई सोच का ःवागत करना चािहए,परन्तु यह भी अिनवायर् है
िक आधुिनक पीढ़ी के
युवा भी वतर्मान बेतुके संःकार और जीवन मूल्यों के ूित आकिषर्त न
हों तथा पुरानी पीढी◌़ की अच्छाइयों को महण करें । यह शुरूआत दोनों तरफ से होनी चािहए
तािक एक नए एवं
११ .
संःकारी समाज की ःथापना की जा सके।
यशोधर बाबू के चिरऽ की िवशेषताएँ िलिखए।
उत्तरः1. कमर्ठ एवं पिरौमी –सेक्शन ऑिफसर होने के बावजूद दफ्तर में दे र तक काम करते
थे | वे अन्य कमर्चािरयों से अिधक कायर् करते थे |
2. संवेदनशील- यशोधर बाबू अत्यिधक संवेदनशील थे | वे यह बात ःवीकार नहीं कर
पाते िक
उनका बेटा उनकी इजाजत िलए िबना ही घर का सोफा सेट आिद खरीद लाता है , उनका
साईिकल से दफ्तर जाने पर ऐतराज करता है , उन्हें दध
ू लेने जाने में असुिवधा न हो इसिलए
से िसंग गाउन भेंट करता है| पत्नी उनकी बात न मानकर बच्चों के कहे अनुसार चलती है | बेटी
िववाह के बंधन में बंधने से इं कार करती है और उसके वस्तर्ों में शालीनता नहीं झलकती |
पिरवारवालों से तालमेल न बैठने के कारण वे अपना अिधकतर समय घर से बाहर मंिदर में
तथा सब्जीमंडी में सब्ज़ी खरीदते िबताते हैं |
110 3. परं परावादीकरने को तैयार
वे परं परावादी थे |आधुिनक समाज में बदलते समीकरणों को ःवीकार
नहीं थे इसिलए पिरवार के अन्य सदःयों से उनका तालमेल नहीं बैठ पा रहा
था |
४. धािमर्क व्यिक्त-यशोधर बाबू एक धािमर्क व्यिक्त थे| वे अपना अिधकतर समय पूजा-पाठ
और मंिदर में िबताते थे |
१२
. यशोधर बाबू जैसे लोग समय के साथ ढ़लने में असफल क्यों होते हैं ?
उत्तरः ऐसे लोग साधारणतया िकसी न िकसी से ूभािवत होते हैं , जैसे यशोधर बाबू िकशन
दा से। ये परं परागत ढरेर् पर चलना पसन्द करते हैं तथा बदलाव
पसन्द नहीं करते। अतः समय
के साथ ढ़लने में असफल होते हैं ।
१३
. भरे -पूरे पिरवार में यशोधर बाबू ःवयं को अधूरा-सा क्यों अनुभव करते हैं ?
उत्तरःसंकेत िबन्द ु - अपने ूाचीन दायरे से बाहर न िनकल सकने के कारण वे ःवयं को
अधूरा अनुभव करते हैं ।
१४.‘‘अभी तुम्हारे अब्बा की इतनी साख है िक सौ रुपए उधार ले सकें।’’ िकन पिरिःथितयों में
यशोधर बाबू को यह कहना पड़ा?
उत्तरःसंकेत िबन्द-ु पुऽ द्वारा उनके िनकट संबंधी की आिथर्क सहायता के िलए मनाही से
आहत होकर ऐसा कहना पड़ा।
१५.
Ôिसल्वर वैिडं ग’ कहानी के तथ्य का िवश्लेषण कीिजए।
उत्तरः
संकेत िबन्द ु - पीढ़ी के अंतराल को उजागर कर, वतर्मान समाज की इस ूकार की
सच्चाई से पदार् उठाया गया है ।
१६.अपने बच्चों की तरक्की से खुश होने के बाद भी यशोधर बाबू क्यामहसूस करते हैं ?
उत्तरः उनके बच्चे गरीब िरँतेदारों के ूित उपेक्षा का भाव रखते हैं । उनकी यह खुशहाली
अपनों के बीच परायापन पैदा कर रही है , जो उन्हें अच्छा नहीं लगता।
१७.आजकल िकशनदा जैसी जीवन-शैली अपनाने वाले बहत
ु कम लोग िमलते हैं , क्यों ?
111 उत्तर: िकशन दा जैसे लोग मःती से जीते हैं , िनःःवाथर् दसरों
की सहायता
ू
करते हैं , जबिक
आजकल सभी सहायता के बदले कुछ न कुछ पाने की आशा रखते हैं , िबना कुछ पाने की आशा
रखे सहायता करने वाले िबरले ही होते हैं ।
१८ . यशोधर बाबू के बच्चों की कौन-सी बातें ूशंसनीय हैं और कौन-सा पक्ष आपित्तजनक है ?
उत्तर- ूशंसनीय बातें- १) महत्वाकांक्षी और ूगितशील होना | २)जीवन में उन्नित करना |
३)समय और सामथ्यर् के अनुसार घर में आधुिनक सुिवधाएँ जुटाना |
आपित्तजनक बातें - १) व्यवहार, २) िपता, िरँतेदारों, धमर् और समाज के ूित नकारात्मक भाव
३) मानवीय सम्बन्धों की गिरमा और संःकारों में रूिच न लेना |
१९.आपकी दृिष्ट में(पाठ से अलग) िसल्वर वैिडं ग मनाने के और कौन से तरीके हो सकते हैं जो
यशोधर बाबू को अच्छे लगते?
उत्तरः
मंिदर जाकर,पिरवार के साथ कहीं घूमने जाकर,पुऽ-पुिऽयों द्वारा
माता-िपता को उपहार दे कर आिद।
२०. िसल्वर वैिडं ग में चटाई का लहँ गा िकसे कहा गया है ?
उत्तरः यह एक ूतीकात्मक ूयोग है िजसे यशोधर बाबू अपनी पत्नी के िलए ूयोग करते
हैं । उनकी पत्नी पुरानी परं पराओं को छोड़ आधुिनकता में ढ़ल गई है , ःवयं को मॉडनर् समझती है ,
इसिलए यशोधर बाबू ने उन्हें यह नाम िदया है । वे उन्हें Ôशानयल बुिढ़या’ तथा Ôबूढ़ी मुह
ँ मुहाँसे,
लोग करें तमासे’ कह कर भी िचढ़ाते हैं ।
२१. यशोधर बाबू पिरवार के बावजूद ःवयं को अधूरा क्यों मानते हैं ?
उत्तर -
यशोधर बाबू पुरानी परं पराओं को माननेवाले हैं , जबिक उनका सारा पिरवार आधुिनक
िवचारधारा
का
है । पीढ़ीगत अंतराल के कारण पिरवार के साथ ताल-मेल न िबठा पाने के
कारण वे ःवयं को अधूरा समझते हैं ।
अन्य महत्त्वपूणर् अभ्यास-ूश्न:
112 1. दफ्तर में यशोधर बाबू से कमर्चारी क्यों परे शान रहते थे?
2. यशोधर बाबू का अपने बच्चों के साथ कैसा व्यवहार था?
3. दफ्तर के बाद पंत जी क्या-क्या काम करते थे?
4. ऑिफस के सािथयों ने यशोधर बाबू को िकस बात की बधारइ◌्
् र दी?
5. यशोधर बाबू तिकया कलाम के रूप में िकस वाक्य का ूयोग करते थे?इस तिकया कलाम
का उनके व्यिक्तत्व तथा कहानी के कथ्य से क्या संबंध है ?
6. पीढ़ी के अंतराल से व्यिक्त अकेला हो जाता है ’- ःपष्ट करें ।
७. अपने बेटे की बड़ी नौकरी पर यशोधर बाबू को क्या आपित्त थी ?
पाठ 2
जूझ: आनंद यादव
पाठ का सार –ÔजूझÕ पाठ आनंद यादव द्वारा रिचत ःवयं के जीवनÐसंघषर् की कहानी है| पढ़ाई पूरी
न कर पाने के कारण, उसका मन उसे कचोटता रहता था |दादा ने अपने ःवाथोर्ं के कारण
ु
उसकी पढ़ाई छड़वा
दी थी |वह जानता था िक दादा उसे पाठशाला नहीं भेजेंगे | आनंद जीवन में
आगे बढ़ना चाहता था | वह जनता था िक खेती से कुछ िमलने वाला नहीं |वह पढ़े गा-िलखेगा
तो बिढ़या-सी नौकरी िमल जाएगी |
आनंद ने एक योजना बनाई िक वह माँ को लेकर गाँव के ूितिष्ठत व्यिक्त दत्ता जी राव
के पास जाएगा| दत्ता जी राव ने उनकी पूरी बात सुनी और दादा को उनके पास भेजने को कहा
| दत्ता जी ने उसे खूब फटकारा, आनंद को भी बुलाया | दादा ने भी कुछ बातें रखीं िक आनंद
को खेती के कायर् में मदद करनी होगी| आनंद ने उनकी सभी बातें सहषर् मान लीं| आनंद की
पढ़ाई शुरू हो गई| शुरु में कुछ शरारती बच्चों ने उसे तंग िकया िकन्तु धीरे -धीरे उसका मन
लगने लगा| उसने कक्षा के मानीटर वसंत पािटल से दोःती कर ली िजससे उसे ठीक ूकार से
पढ़ाई करने की ूेरणा िमली| कई परे शािनयों से जूझते हए
ु आनंद ने िशक्षा का दामन नहीं छोड़ा|
मराठी पढ़ाने के िलए ौी सौंदलगेकर आए| उन्होंने आनंद के हृदय में एक गहरी छाप छोड़ी|
उसने भी किवताओं में रूिच लेनी ूारम्भ की| उसने खेतों में काम करते Ðकरते किवताएँ कंठःथ
की| माःटर ने उसकी किवता बड़े ध्यान से सुनी| बालक का आत्मिवश्वास बढ़ने लगा और उसकी
काव्य-ूितभा में िनखार आने लगा |
113 ूश्नोत्तर१. जूझ कहानी के आधार पर दत्ता जी राव दे साई का चिरऽ-िचऽण कीिजए ।
उत्तर 1.
िफर
समझदार व्यिक्तÐ दत्ता जी राव ने छोटे से बालक आनंद की बातों को सुना|
आनंद के दादा को बुलाया और आनंद को िवद्यालय भेजने पर जोर िदया| उन्होंने दादा से
इस ूकार बातें की िक उन्हें शक ही नहीं हआ
िक ये सब उन्हें आनंद ने बताया है|
ु
2.
मामीणों के मददगार Ð दत्ता जी गाँव के ूितिष्ठत एवम ूभावशाली व्यिक्त
थे िकंतु उन्होंने अपनी ूितष्ठा का दरुपयोग
नहीं िकया| मामीण उनके पास अपनी समःयाएँ
ु
लेकर आते थे| दत्ता जी उनकी समःयाओं को हल करने का ूयास करते थे|
3.ूभावशाली व्यिक्तत्वÐ दत्ता जी गाँव के ूितिष्ठत व्यिक्त थे| सभी उन्हें बहत
ु मान
दे ते थे| उनका व्यिक्तत्व रोबीला था और मामीण उनकी बात मान जाते थे|
२ . जूझ कहानी का उद्दे ँय क्या है ?
अथवा
Ôिवषम पिरिःथितयों में भी िवकास संभव है ।’‘जूझ’ कहानी के आधार पर ःपष्ट कीिजए|
उत्तरः
में
इस कहानी का मुख्य उद्दे ँय है िक मनुंय को संघषर् करते रहना चािहए| कहानी
लेखक के जीवन के संघषर् को, उसके पिरवेश के साथ िदखलाया गया है । लेखक का िशक्षा-
ूािप्त हे तु िपता से संघषर्,कक्षा में संघषर्,खेती में संघषर् और अंत में उसकी सफलता कहानी के
उद्दे ँय को ःपष्ट करते हैं । अतः समःयाओं से भागना नहीं चािहए| पूरे आत्मिवश्वास से उनका
मुकाबला करना चािहए| संघषर् करने वाले को सफलता अवँय िमलती है |
३ . ‘जूझ’ कहानी के शीषर्क की साथर्कता पर िटप्पणी कीिजए।
उत्तरः जूझका अथर् है –संघषर्| यह कथा ,कथानायक के जीवन भर के संघषर् को दशार्ती है|
बचपन से अभावों में पला बालक, िवपरीत पिरिःथितयों पर िवजय हािसल कर सका|अतः यिद
मन में लगन हो, भरपूर आत्मिवश्वास हो तो सफलता कदम चूमती है |
४ . ÔजूझÕ कहानी के आधार पर आंनदा के चिरऽ की िवशेषताएँ बताइए।
अथवा
‘जूझ’शीषर्क के औिचत्य पर िवचार करते हए
ु यह ःपष्ट करें िक क्या यह शीषर्क कथानायक की
िकसी केन्िीय चािरिऽक िवशेषता को उजागर करता है ?
114 उत्तरः 1. पढ़ने की लालसा- आनंदा िपता के साथ खेती का काम सँभालता था| लेिकन पढ़ने की
तीो इच्छा ने उसे जीवन का एक उद्दे ँय दे िदया और वह अपने भिवंय को एक सही िदशा दे ने
में सफल होता है|
2. वचनबद्धता- आनंदा ने िवद्यालय जाने के िलए िपता की जो शतेर्ं मानी थी उनका
पालन हमेशा िकया| वह िवद्यालय जाने से पहले बःता लेकर खेतों में पानी दे ता| वह ढोर चराने
भी जाता| िपता की बातों का अनुपालन करता|
3.
आत्मिवश्वासी एवं कमर्ठ बालकÐआनंदा के जीवन में अभाव ही थे| उसके
िपता ने ही उसका पाठशाला जाना बंद करवा िदया था| िकंतु उसने िहम्मत नहीं हारी, पूरे
आत्मिवश्वास के साथ योजनाबद्ध तरीके से आगे बढ़ा और सफल हआ
ु |
४.
किवता के ूित झुकाव- आनंदा ने माःटर सौंदलगेकर से ूभािवत होकर काव्य में
रुिच लेनी ूारम्भ की| वह खेतों में पानी दे ता, भैंस चराते समय भी किवताओं में खोया रहता
था| धीरे -धीरे वह ःवयं तुकबंदी करने लगा| किवताएँ िलखने से उसमें आत्मिवश्वास बढ़ा|
५ .
आनन्दा के िपता की भाँित आज भी अनेक गरीब व कामगार िपता अपने बच्चों को
ःकूल नहीं भेजना चाहतेथे, क्यों? आपकी दृिष्ट में उन्हें िकस ूकार ूेिरत िकया जा सकता है ?
उत्तरः
अिधकतर लोग अिशिक्षत होने के कारण िशक्षा के महत्त्व को नहीं समझते|
अिधक बच्चे , कम आमदनी, दो अितिरक्त हाथों से कमाई की लालसा आिद इसके ूमुख कारण
हैं । उन्हें ूेिरत करने हे तु उन्हें जागरूक करना, िशक्षा का महत्त्व बताना,िशिक्षत होकर जीवन
ःतर में सुधार के िलए ूोत्सािहत िकया जा सकता है ।
६ . सकारात्मक सृजनात्मकता से आत्मिवश्वास को दृढ़ता िमलती है । जूझ कहानी के आधार पर
समझाइए।
उत्तरः लेखक ने पढ़ाई में ःवयं को िपछड़ता हआ
पाया| िकंतु िशक्षक सौदं लगेकर से ूेिरत
ु
होकर लेखक
कुछ तुकबंदी करने लगा| धीरे Ðधीरे उसमें आत्मिवश्वास बढ़ने लगा| सृजनात्मकता
ने उसके जीवन को नया मोड़ िदया| अतः लेखक के द्वारा किवता रच लेने से उसमें आत्मिवश्वास
का सृजन हआ
ु ।
७. ‘जूझ’कहानी के आधार पर बताइए िक एक ूभावशाली िशक्षक िकस तरह अपने िवद्याथीर् का
भिवंय सँवार सकता है ?
अथवा
115 आनन्दा से किव डॉ. आनन्द यादव बनने की याऽा में माःटर सौंन्दलगेकर की भूिमका ःपष्ट
कीिजए।
उत्तरःमाःटर सौंदलगेकर कुशल अध्यापक, मराठी के ज्ञाता व किव,सुरीले ढं ग से ःवयं कीव
दसरों
की किवताएँ गाते थे। आनन्दा को किवता या तुकबन्दी िलखने के ूारिम्भक काल में
ू
उन्होंने उसका मागर्दशर्न व सुधार िकया,उसका आत्मिवश्वास बढ़ाया िजससे वह धीरे -धीरे किवताएँ
िलखने में कुशल हो कर ूितिष्ठत किव बन गया।
८.
आनन्दा कोपुनः ःकूल जाने पर क्या-क्या झेलना पड़ा ?
उत्तरः
ःवयं से कम आयु के छाऽों के साथ बैठना पड़ा,वह अन्य छाऽों की हँ सी का पाऽ
बना तथा उसे िपता की इच्छा के कारण घर व ःकूल दोनों में िनरं तर काम करना पड़ा।
९ . ःकूल में मानीटर ने आनंदा को सवार्िधक ूभािवत िकया और कैसे?
उत्तरः
मानीटर बसंत पािटल ने उसे सवार्िधक ूभािवत िकया|वह शांत व अध्ययनशील
छाऽ था। आनंदा उसकी नकल कर हर काम करने लगा,धीरे -धीरे एकामिचत व अध्ययनशील
बनकर
आनन्दा भी कक्षा में सम्मान का पाऽ बन गया।
१० . आपके दृिष्ट से पढ़ाई-िलखाई के सम्बन्ध में लेखक और दत्ता जी राव का रवैया सही था या
लेखक के िपता का?तकर् सिहत उतर दीिजए।
उत्तरः लेखक का मत है िक जीवन भर खेतों में काम करके कुछ भी हाथ आने वाला
नहीं है । पीढ़ी दर पीढ़ी खेतों में काम करके कुछ ूाप्त नहीं हो सका। वे मानते हैं िक यह खेती
हमें गढ्ढे में धकेल रही है । अगर मैं पढ़-िलख गया तो कहीं मेरी नौकरी लग जाएगी या कोई
व्यापार करके अपने जीवन को सफल बनाया जा सकता है । माँ-बेटा जब दत्ता जी राव के घर
जाकर पूरी बात बताते हैं तो दत्ता जी राव िपता जी को बुलाकर खूब डाँटते हैं और कहते हैं िक
तू सारा िदन क्या करता है । बेटे और पत्नी को खेतों में जोत कर तू सारा िदन साँड की तरह
घूमता रहता है । कल से बेटे को ःकूल भेज, अगर पैसे नहीं हैं तो फीस मैं दँ ग
ू ा। परन्तु िपता जी
को यह सब कुछ बुरा लगा। दत्ता जी राव के सामने ‘हाँ’ करने के बावजूद भी वे आनन्द को
ःकूल भेजने के पक्ष में नहीं थे। इस ूकार लेखक और उनके िपताजी की सोच में
एक बड़ा
अन्तर है । हमारे खयाल से पढ़ाई-िलखाई के सम्बन्ध में लेखक और दत्ता जी राव का रवैया
116 लेखक के िपता की सोच से ज्यादा ठीक है क्योंिक पढ़ने-िलखने से व्यिक्त का सवार्गींण िवकास
होता है ।
अन्य महत्त्वपूणर् अभ्यास-ूश्न:
१. जूझ पाठ के आधार पर बताइए िक कौन, िकससे, कहाँ जूझ रहा है तथा अपनी जूझ में कौन
सफल होता है ?
२. आनंदा का िपता कोल्हू जल्दी क्यों चलवाता था?
३. पाठशाला में आनंदा का पहला अनुभव कैसा रहा?
४. दत्ता राव के सामने रतनप्पा ने आनंदा को ःकूल न भेजने का क्या कारण बताया ?
५. आनंदा के िपता ने उसे पाठशाला भेजने की सहमित िकस शतर् पर दी?
पाठ 3
अतीत में दबे पाँव: ओम थानवी
पाठ का सार–यहओम थानवी के याऽा-वृत्तांत और िरपोटर् का िमला-जुला रूप है|उन्होंने इस पाठ में
िवश्व के सबसे पुराने और िनयोिजत शहरों-मुअनजो-दड़ो तथा हड़प्पा का वणर्न िकया है |
पािकःतान के िसंध ूांत में मुअनजो-दड़ो ओर पंजाब ूांत में हड़प्पा नाम के दो नगरों को
पुरातत्विवदों ने खुदाई के दौरान खोज िनकाला था|मुअनजो-दड़ो ताॆकाल का सबसे
बड़ा शहर
था |मुअनजो-दड़ो अथार्त मुदोर्ं का टीला| यह नगर मानव िनिमर्त छोटे Ðछोटे टीलों पर बना था
|मुअनजो-दड़ो में ूाचीन और बड़ा बौद्ध ःतूप है | इसकी नगर योजना अिद्वतीय है| लेखक ने
खंडहर
हो चुके टीलों, ःनानागार, मृद-भांडों, कुओंÐतालाबों, मकानों व मागोर्ं का उल्लेख िकया है
िजनसे शहर की सुद
ं र िनयोजन व्यवःथा का पता चलता है| बःती में घरों के दरवाजे मुख्य
सड़क की ओर नहीं खुलते, हर घर में जल िनकासी की व्यवःथा है , सभी नािलयाँ की ढकी हई
ु
हैं , पक्की ईंटों का ूयोग िकया गया है|
नगर में चालीस फुट लम्बा ओर पच्चीस फुट चौड़ा एक महाकुंड भी है |इसकी दीवारें ओर तल
पक्की ईंटों से बने हैं | कुंड के पास आठ ःनानागार हैं | कुंड में बाहर के अशुद्ध पानी को न
आने दे ने का ध्यान रखा गया | कुंड में पानी की व्यवःथा के िलए कुंआ है | एक िवशाल कोठार
भी है िजसमें अनाज रखा जाता था |उन्नत खेती के भी िनशान िदखते हैं -कपास, गेहूं , जौ,
सरसों, बाजरा आिद के ूमाण िमले हैं |
117 िसंधु घाटी सभ्यता में न तो भव्य राजमहल िमलें हैं ओर ही भव्य मंिदर| नरे श के
सर पर रखा मुकुट भी छोटा है| मुअनजो-दड़ो िसंधु घाटी का सबसे बड़ा नगर है िफर भी इसमें
भव्यता व आडम्बर का अभाव रहा है| उस समय के लोगों ने कला ओर सुरुिच को महत्त्व िदया|
नगर-िनयोजन, धातु एवं पत्थर की मूितर्याँ, मृद-भांड ,उन पर िचिऽत मानव ओर अन्य
आकृ ितयाँ ,मुहरें , उन पर बारीकी से की गई िचऽकारी| एक पुरातत्त्ववेत्ता के मुतािबक िसंधु
सभ्यता की खूबी उसका सौंदयर्-बोध है जो
राजपोिषत या
धमर्पोिषत न होकर समाजपोिषत
था|
ूश्नोत्तर–
१. Ôिसन्धु सभ्यता साधन सम्पन्न थी, पर उसमें भव्यता का आडं बर नहीं था |Õ ूःतुत
कथन से आप कहाँ तक सहमत हैं ?
उत्तरः दसरी
सभ्यताएँ राजतंऽ और धमर्तंऽ द्वारा संचािलत थी । वहाँ बड़े -बड़े सुन्दर महल,
ू
पूजा ःथल, भव्य मूितर्याँ, िपरािमड और मिन्दर िमले हैं । राजाओं, धमार्चायोर्ं की
समािधयाँ भी मौजूद हैं । िकंतु िसन्धु सभ्यता, एक साधन-सम्पन्न सभ्यता थी परन्तु
उसमें
राजसत्ता
या
धमर्सत्ताके
िचह्न
नहीं
िमलते।
वहाँ
की
नगर
योजना,वाःतुकला,मुहरों,ठप्पों,जल-व्यवःथा,साफ-सफाई और सामािजक व्यवःथा आिद की
एकरूपताद्वारा उनमें अनुशासन दे खा जा सकता है |सांःकृ ितक धरातलपर यह तथ्य सामने
आता है िक िसन्धु घाटी की सभ्यता, दसरीसभ्यताओं
से अलग एवं ःवाभािवक, िकसी
ू
ूकार की कृ िऽमता एवं आडं बररिहतथी जबिक अन्य सभ्यताओं में राजतंऽ और धमर्तंऽ
की ताकत को िदखाते हए
ु भव्य महल , मंिदर ओर मूितर्याँ बनाई गईं िकंतु िसन्धु घाटी
सभ्यता की खुदाई में छोटी-छोटी मूितर्याँ, िखलौने, मृद-भांड, नावें िमली हैं । इस ूकार
यह ःपष्ट है िक िसन्धु सभ्यता सम्पन्न थी परन्तु उसमें भव्यता का आडं बर नहीं था।
२. Ôिसन्धु सभ्यता की खूबी उसका सौन्दयर् बोध है जो राजपोिषत न होकर समाज-पोिषत
था।Ô ऐसा क्यों कहा गया है ?
उत्तरः
िसन्धु सभ्यता में औजार तो बहत
ु िमलेहैं,परं तु हिथयारों का भी ूयोग होता रहा
होगा , इसका कोई ूमाण नहींहै। वे लोग अनुशासनिूय थे परन्तु यह अनुशासन िकसी ताकत
के बल के द्वारा कायम नहीं िकया गया,बिल्क लोग अपने मन और कमर् से ही अनुशासन िूय
118 थे। मुअनजो-दड़ो
की खुदाई में एक दाढ़ी वाले नरे श की छोटी मूितर् िमली है परन्तु यह मूितर्
िकसी राजतंऽ या धमर्तंऽ का ूमाण नहीं कही जा सकती। िवश्व की अन्य सभ्यताओं के साथ
तुलनात्मक अध्ययन से भी यही अनुमान लगाया जा सकता है िक िसन्धु सभ्यता की खूबी
उसका सौन्दयर्बोध है जो िक समाज पोिषत है , राजपोिषत या धमर्पोिषत नहीं है ।
३.
ू -फूटी सीिढ़याँ अब आप को कहीं नहीं ले
‘यह सच है िक यहाँ िकसी आँगन की टटी
जातीं,वे आकाश की तरफ अधूरी रह जाती हैं । लेिकन उन अधूरे पायदानों पर खड़े होकर
अनुभव िकया जा सकता है िक आप दिनया
की छत पर हैं , वहाँ से आप इितहास को
ु
नहीं उस के पार झाँक रहे हैं ।’ इसके पीछे लेखक का क्या आशय है ?
उत्तरः
होकर आप
ू -फूटे घरों की सीिढ़यों पर खड़े
इस कथन से लेखक का आशय है िक इन टटे
िवश्व की सभ्यता के दशर्नकर सकते हैं क्योंिक िसन्धु सभ्यता िवश्व की महान
सभ्यताओ में से एक है । िसन्धु सभ्यता आडं बररिहत एवं अनुशासनिूय है । खंडहरों से िमले
ू -फूटे घरों से केवल िसन्धु सभ्यता का इितहास ही नहीं दे खा जा सकता है
अवशेषों और इन टटे
बिल्क उससे कहीं आगे मानवता के िचह्न ओर मानवजाित के बिमक िवकास को भी दे खा जा
सकता है । कई ूश्न - ऐसे कौन से कारण रहे होंगे िक ये महानगर आज केवल खंडहर बन कर
रह गए हैं ?, वे बड़े महानगर क्यों उजड़ गए?, उस जमाने के लोगों की वाःतुकला, कला ओर
ज्ञान में रुिच, समाज के िलए आवँयक मूल्य - अनुशासन, सादगी, ःवच्छता, सहभािगता
आिदहमें मानवजाित के बिमक िवकास पर पुनः िचंतन करने पर मजबूर कर दे ते हैं । इस
ूकार हम इन सीिढ़यों पर चढ़कर िकसी इितहास की ही खोज नहीं करना चाहते बिल्क िसन्धु
सभ्यता के सभ्य मानवीय समाज को दे खना चाहते हैं ।
४. हम िसन्धु सभ्यता को जल-संःकृ ित कैसे कह सकते हैं ?
उत्तरःिसन्धु सभ्यता एक जल-संःकृ ित थी | ूत्येक घर में एक ःनानघरथा । घर के
भीतर से पानी या मैला पानी नािलयों के माध्यम से बाहर हौदी में आता है और िफर बड़ी
नािलयों में चला जाता है । कहीं-कहीं नािलयाँ ऊपर से खुली हैं परन्तु अिधकतर नािलयाँ ऊपर
सेबंदहैं । इनकी जलिनकासी व्यवःथा बहत
ु ही ऊँचे दजेर् की था | नगर में कुओं का ूबंध था
। ये कुएँ पक्की ईटों के बने थे। िसन्धु सभ्यता से जुड़े इितहासकारों का मानना है िक यह
सभ्यता िवश्व में पहली ज्ञात संःकृ ित है जो कुएँ खोदकर भू-जल तक पहँु ची। अकेले
मुअनजो-दड़ों नगर में सात सौ कुएँ हैं ।यहाँ का महाकुंड लगभग चालीस फुट लम्बा ओर
119 पच्चीस फुट चौड़ा है|इस ूकार मुअनजो-दड़ों में पानी की व्यवःथा सभ्य समाज की पहचान
है ।
५. मुअनजो-दड़ो की गृह-िनमार्ण योजना पर संक्षेप में ूकाश डािलए|
उत्तर- मुअनजो-दड़ो नगर की मुख्य सड़क के दोनों ओर घर हैं परं तु िकसी भी घर का
दरवाजामुख्य सड़क पर नहीं खुलता। घर जाने के िलए मुख्य सड़क से गिलयों में जाना पड़ता
है ,सभी घरों के िलए उिचत जल िनकासी व्यवःथा है । घर पक्की ईंटों के बने हैं|छोटे घरों में
िखड़िकयाँ नहीं थीं िकंतु बड़े घरों में आंगन के भीतर चारों तरफ बने कमरों में िखड़िकयाँ हैं| घर
छोटे भी हैं ओर बड़े भी िकंतु सभी घर कतार में बने हैं|
६.
‘िसन्धु सभ्यता ताकत से शािसत होने की अपेक्षा समझ से अनुशािसत सभ्यता थी’ःपष्ट कीिजए।
उत्तरःसंकेत िबंद ु -
खुदाई से ूाप्त अवशेषों में औजार तो िमले हैं िकंतु हिथयार नहीं|
कोई खड्ग ,भाला, धनुष-बाण नहीं िमला|
तथा भव्य महलों व समािधयों के न होने से कह सकते हैं िक िसन्धु सभ्यता ताकत से
नहीं समझ से अनुशािसत थी।
७. संसार की मुख्य ूाचीन सभ्यताएँ कौन-कौन सी हैं ?ूाचीनतम सभ्यता कौन-सी है उसकी
ूमािणकता का आधार क्या है ?
उत्तरः
िमॐ की नील घाटी की सभ्यता, मेसोपोटािमया की सभ्यता, बेबीलोन की
सभ्यता, िसन्धु घाटी की सभ्यता। सबसे ूाचीन है िसन्धु घाटी की सभ्यता। ूमाण है 1922में
िमले हड़प्पा व
मुअनजो-दड़ोनगरों के अवशेष। ये नगर ईसा पूवर् के हैं ।
८.िसन्धु घाटी की सभ्यता की िविशष्ट पहचान क्या है ?
उत्तरः1. एक जैसे आकार की पक्की ईटों का ूयोग, 2. जल िनकासी की उत्कृ ष्ट व्यवःथा,
3. तत्कालीन वाःतुकला, 4. नगर का ौेष्ठ िनयोजन।
९ .मुअनजो-दड़ोमें पयर्टक क्या-क्या दे ख सकते हैं ?
120 उत्तरः 1.बौद्ध ःतूप2. महाकुंड 3.अजायबघर आिद
१० . िसन्धु सभ्यता व आजकल की नगर िनमार्ण योजनाओं में साम्य व अन्तर बताइए।
उत्तरः साम्य-1. अच्छी जल िनकास योजना,ढकी हई
ु नािलयाँ, 2. नौकरों के िलए अलग
आवास व्यवःथा,
3. पक्की ईंटों का ूयोग।
अन्तर- नगर योजना, आधुिनक तकनीक का ूयोग, अत्याधुिनक भवनÐिनमार्ण साममी का
ूयोग|
१ १ .कुलधरा कहाँ है ?मुअनजो-दड़ोके खण्डहरों को दे ख कुलधरा की याद क्योंआती है ?
उत्तरः कुलधरा जैसलमेर के मुहाने पर पीले पत्थरों से बने घरों वाला सुन्दर गाँव है । कुलधरा के
िनवासी 150 वषर् पूवर् राजा से तकरार होने पर गाँव खाली करके चले गए। उनके घर अब
खण्डहर बन चुके हैं ,
परं तु ढ़हे नहीं |घरों की दीवारें और िखड़िकयाँ ऐसी हैं मानो सुबह लोग
काम पर गए हैं और साँझ होते ही लौट आएंगें |मुअनजो-दड़ोके खण्डहरों को दे खकर कुछ ऐसा
ही आभास होता है वहाँ घरों के खण्डहरों में घूमते समय िकसी अजनबी घर में अनिधकार चहलकदमी का अपराधबोध होता है| पुराताित्वक खुदाई अिभयान की यह खूबी रही है िक सभी
वःतुओं को बड़े सहे ज कर रखा गया | अतः कुलधरा की बःती और मुअनजो-दड़ोके खण्डहर
अपने काल के इितहास का दशर्न कराते हैं ।
अन्य महत्त्वपूणर् अभ्यास-ूश्न:
१ . िसन्धु घाटी के िनवासी खेती करते थे- इस कथन को िसद्ध कीिजए।
ू -फूटे खण्डहर सभ्यता और संःकृ ित के इितहास के साथ-साथ धड़कती िजन्दिगयों के
२. ’टटे
ु समयों का दःतावेज होते हैं ।’ इस कथन का भाव ःपष्ट कीिजए।
अनछए
३.आज जल संकट एक बड़ी समःया है| ऐसे में िसन्धु सभ्यता के महानगर मुअनजो-दड़ो
की
जल व्यवःथा से क्या ूेरणा लीजा सकती है ? भावी जल संकट से िनपटने के िलए आप क्या
सुझाव दें गे ?
४.मुअनजो-दड़ोके अजायबघर में कौन-कोन सी वःतुएँ ूदिशर्त थीं ?
५.िसंधु सभ्यता अन्य सभ्यताओं से िकस ूकार िभन्न है ?
121 ६.मुअनजो-दड़ोकी ूमुख िवशेषताएँ िलिखए ।
७. िसंधुघाटी की सभ्यता लो-ूोफाइल है -ःपष्ट कीिजए ?
८. लेखक ने ूाचीन लैंडःकेप िकसे कहा है ?उसकी क्या िवशेषता है ?
९. ताॆकाल के दो सबसे बड़े िनयोिजत शहर िकन्हें माना गया है और क्यों ?
--------------------------------------------------------------------------------------------------
पाठ 4 : डायरी के पन्ने: ऐन ृैंक
पाठ का सार Ð Ôडायरी के पन्नेÕ पाठ में Ôद डायरी ऑफ ए यंग गलर्Õ नामक ऐन ृैंक की डायरी
के कुछ अंश िदए गए हैं| Ôद डायरी ऑफ ए यंग गलर्Õ ऐन ृैंक द्वारा दो साल अज्ञातवास के
दरम्यान िलखी गई थी| १९३३ में ृैंकफटर् के नगरिनगम चुनाव में िहटलर की नाजी पाटीर् जीत
गई| तत्पश्चात यहदी
ू -िवरोधी ूदशर्न बढ़ने लगे | ऐन ृैंक का पिरवार असुरिक्षत महसूस करते
हए
ु नीदरलैंड के एम्सटडर् म शहर में जा बसा |िद्वतीय िवश्वयुद्ध की शुरुआत तक(१९३९) तो सब
ठीक था| परं तु १९४० में नीदरलैंड पर जमर्नी का कब्ज़ा हो गया ओर यहिदयों
के उत्पीड़न का
ू
दौर शुरु हो गया| इन पिरिःथितयों के कारण १९४२ के जुलाई मास में ृैंक पिरवारिजसमें मातािपता,तेरह वषर् की ऐन ,उसकी बड़ी बहन मागोर्ट तथा दसरा
पिरवार –वानदान पिरवार ओर उनका
ू
बेटा पीटरतथा इनके साथ एक अन्य व्यिक्त िमःटर डसेल दो साल तक गुप्त आवास में रहे| गुप्त
आवास में इनकी सहायता उन कमर्चािरयों ने की जो कभी िमःटर ृैंक के दफ्तर में काम करते
थे||Ôद डायरी ऑफ ए यंग गलर्Õ ऐन ृैंक द्वारा उस दो साल अज्ञातवास के दरम्यान िलखी गई
थी|अज्ञातवास उनके िपता िमःटर ऑटो ृैंक का दफ्तर ही था| ऐन ृैंक को तेरहवें जन्मिदन
पर एक डायरी उपहार में िमली थी ओर उसमें उसने अपनी एक गुिड़या-िकट्टी को सम्बोिधत
िकया है|
ऐनअज्ञातवास में पूरा िदनÐ पहे िलयाँ बुझाती, अंमेज़ी व ृेंच बोलती, िकताबों की समीक्षा करती,
राजसी पिरवारों की वंशावली दे खती, िसनेमा ओर िथएटर की पिऽका पढ़ती और उनमें से नायकनाियकाओं के िचऽ काटतेिबताती थी| वह िमसेज वानदान की हर कहानी को बार-बार सुनकर
बोर हो जाती थी ओर िम. डसेल भी पुरानी बातेंÐ घोड़ों की दौड़, लीक करती नावें, चार बरस की
उॆ में तैर सकने वाले बच्चे आिद सुनाते रहते|
122 उसने युद्ध संबंधी जानकारी भी दी है - कैिबनेट मंऽी िम. बोल्के ःटीन ने लंदन से डच ूसारण में
यह घोषणा की थी िक युद्ध के बाद युद्ध के दौरान िलखी गईं डायिरयों का संमह िकया जाएगा,
वायुयानों से तेज़ गोलाबारी, हज़ार िगल्डर के नोट अवैध घोिषत िकए गए | िहटलर के घायल
सैिनकों में िहटलर से हाथ िमलाने का जोश , अराजकता का माहौल- कार, साईिकल की चोरी,
घरों की िखड़की तोड़ कर चोरी, गिलयों में लगी िबजली से चलने वाली घिड़याँ, सावर्जिनक
टे लीफोन चोरी कर िलए गए|
ऐन ृैंक ने नारी ःवतंऽता को महत्त्व िदया,उसने नारी को एक िसपाही के बराबर सम्मान दे ने
की बात कही|
एक तेरह वषीर्य िकशोरी के मन की बेचैनी को भी व्यक्त िकया- जैसे िम. डसेल
की ड़ाँट-फटकार ओर उबाऊ भाषण, दसरों
की बातें सुनकर िमसेज ृैंक का उसेडाँटना ओर उस
ू
पर अिवश्वास करना, बड़ों के द्वारा उसके काम ओर केशसज्जा पर टीका-िटप्पणी करना, िसनेमा
की पिऽका खरीदने पर िफज़ूलखचीर् का आरोप लगाना, पीटर द्वारा उसके ूेम को उजागर न
करना आिद|
ऐन ृैंक की डायरी के द्वारा िद्वतीय िवश्वयुद्ध की िवभीिषका, िहटलर एवं नािजयों द्वारा यहिदयों
ू
का उत्पीड़न, डर, भुखमरी, गरीबी, आतंक, मानवीय संवेदनाएँ, ूेम, घृणा, तेरह साल की उॆ के
सपने, कल्पनाएँ, बाहरी दिनया
से अलग-थलग पड़ जाने की पीड़ा, मानिसक ओर शारीिरक
ु
जरूरतें, हँ सी-मज़ाक, अकेलापन आिद का जीवंत रूप दे खने को िमलता है |
ूश्नोत्तर –
१ . ‘‘ऐन की डायरी अगर एक ऐितहािसक दौर का जीवंत दःतावेज है , तो साथ ही उसके िनजी
सुख-दःख
और भावनात्मक उथल-पुथल का भी। इन पृष्ठों में दोनों का फकर् िमट गया है । ’’ इस
ु
कथन पर िवचार करते हए
र् व्यक्त करें ।
ु अपनी सहमित या असहमित तकर्पूवक
उत्तरःऐन की डायरी अगर एक ऐितहािसक दौर का जीवंत दःतावेज है , तो साथ ही उसके
िनजी सुख-दःख
और भावनात्मक उथल-पुथल का भी क्योंिक इसमें ऐन ने िद्वतीय िवश्वयुद्ध के
ु
समय हॉलैंड के यहदी
ू पिरवारों की अकल्पनीय यंऽणाओं का वणर्न करने के साथ-साथ, वहाँ की
राजनैितक िःथित एवं युद्ध की िवभीिषका का जीवंत वणर्न िकया है ।वायुयानों से तेज़ गोलाबारी,
हज़ार िगल्डर के नोट अवैध घोिषत िकए गए , िहटलर के घायल सैिनकों में िहटलर से हाथ
िमलाने का जोश , अराजकता का माहौलआिद| साथ हीयह डायरी, ऐन के पािरवािरक सुख-दःख
ु
और भावनात्मक िःथित को ूकट करती है - गरीबी, भुखमरी,अज्ञातवास में जीवन व्यतीत करना,
123 दिनया
से िबलकुल कट जाना ,पकड़े जाने का डर, आतंक। यह डायरी एक ओर वहाँ के
ु
राजनैितक वातावरण में सैिनकों की िःथित, आचरण व जनता पर होने वाले अत्याचार िदखाती
हैं तो दसरी
ओरएक तेरह वषर् की िकशोरी की मानिसकता, कल्पना का संसार ओर उलझन को
ू
भीिदखाती है जो ऐन की आपबीती है । इस तरह यह डायरी ऐितहािसक दःतावेज होने के साथसाथ ऐन के जीवन के सुख-दख
ु का िचऽण भी है ।
२. ‘‘यह साठ लाख लोगों की तरफ से बोलने वाली एक आवाज है । एक ऐसी आवाज जो िकसी
सन्त या किव की नहीं, बिल्क एक साधारण-सी लड़की की है ।’’ इल्या इहरनबुगर् की इस िटप्पणी
के संदभर् में ऐन ृैंक की डायरी के पिठत अंशों पर िवचार करें ।
उत्तरः
उस समय यूरोप में लगभग साठ लाख यहदीथे
।िद्वतीय िवश्वयुद्ध में नीदरलैंड पर
ू
जमर्नी का कब्ज़ा हो गया और िहटलर की नाजी फौज ने यहिदयों
को िविभन्न ूकार से यंऽणाएं
ू
दे ने लगे | उन्हें तरह-तरह के भेदभाव पूणर् ओर अपमानजनक िनयम-कायदों को मानने
के िलए
बाध्य िकया जाने लगा |गेःटापो (िहटलर की खुिफया पुिलस) छापे मारकर यहिदयों
को
ू
अज्ञातवास से ढँू ढ़ िनकालती ओर यातनागृह में भेज दे ती| अतः चारों तरफ अराजकता फैली हई
ु
थी। यहदी
ू अज्ञातवास में िनरं तर अंधेरे कमरों में जीने को मजबूरथे।उन्हें एक अमानवीय जीवन
जीने को बाध्य होना पड़ा|िहटलर की नाजी फौजका खौफ उन्हें हरवक्त आतंिकत करता रहता था
| ऐन ने डायरी के माध्यम से न केवलिनजी सुख-दःख
और भावनाओं को व्यक्त िकया,बिल्क
ु
लगभग साठ लाख यहदी
ु भरी िजन्दगी को िलिपबद्ध िकया है । इसिलए इल्या
ू समुदाय की दख
इहरनबुगर् की यह िटप्पणी िक ‘‘यह साठ लाख लोगों की तरफ से बोलने वाली एक आवाज है ।
एक ऐसी आवाज जो िकसी संत या किव की नहीं, बिल्क एक साधारण-सी लड़की की
है ।’सवर्मान्य एवं सत्य है ।
३. ऐन ृैंक कौन थी?उसने अपनी डायरी में िकस काल की घटनाओं का िचऽण िकया है ?यह
क्यों महत्वपूणर् है ?
उत्तर:. ऐन ृैंक एक यहदी
ू लड़की थी। उसने अपनी डायरी में िद्वतीय िवश्वयुद्ध (19391945) के दौरान िहटलर की नाजी फौज ने यहिदयों
को िविभन्न ूकार से यंऽणाएं दीं| यह
ू
डायरी युद्ध के दौरान फैली अराजकता और राजनेितक पिरदृँय को दशार्ती है| यह नािजयों द्वारा
यहिदयों
पर िकए गए जुल्मों का एक ूामािणक दःतावेज है और साथ ही साथ एक तेरह वषीर्य
ू
िकशोरी की भावनाएँ और मानिसकता को समझाने में सहायक है ।
124 ४. डायरी के पन्ने पाठ में िम. डसेल एवं पीटर का नाम कई बार आया है । इन दोनों का
िववरणात्मक पिरचय दें ।
उत्तरः
िम.डसेल- ऐन के िपता के साथ काम करते थे। वे ऐन व पिरवार के साथ
अज्ञातवास में रहे थे। डसेल उबाऊ लंबे-लंबे भाषण दे ते थे और अपने जमाने के िकःसे सुनाते
रहते थे। ऐन को अक्सर डाँटते थे । वे चुगलखोर थे और ऐन की मम्मी से ऐन की सच्ची-झूठी
िशकायतें करते थे ।
पीटर- िमःटर और िमसेज वानदान का बेटा था |वह ऐन का हमउॆ था। ऐन का उसके ूित
आकषर्ण बढ़ने लगा था और वह यह मानने लगी थी िक वह उससे ूेम करती है ।ऐन के
जन्मिदन पर पीटर ने उसे फूलों का गुलदःता भेंट िकया था| िकंतु पीटर सबके सामने ूेम
उजागर करने से डरता था| वह साधारणतया शांितिूय, सहज व आत्मीय व्यवहार करने वाला
था।
५. . िकट्टी कौन थी?ऐनृैंक ने िकट्टी को संबोिधत कर डायरी क्यों िलखी?
उत्तरः
Ôिकट्टीÕऐन ृैंक की गुिड़या थी। गुिड़या को िमऽ की भाँित संबोिधत करने से
गोपनीयता भंग होने का डर न था।अन्यथा नािजयों द्वारा अत्याचार बढ़ने का डर व उन्हें
अज्ञातवास का पता लग सकता था।| ऐन ने ःवयं (एक तेरह वषीर्य िकशोरी) के मन की बेचैनी
को भी व्यक्त करने का ज़िरया िकट्टी को बनाया |वह हृदय में उठ रही कई भावनाओं को दसरों
ू
के साथ बाँटना चाहती थी िकंतु अज्ञातवास में उसके िलए िकसी के पास समय नहीं था| िम.
डसेल की ड़ाँट-फटकार ओर उबाऊ भाषण ,दसरों
के द्वारा उसके बारे में सुनकर मम्मी (िमसेज
ू
ृैंक) का उसेड़ाँटना ओर उस पर अिवश्वास करना, बड़ों का उसे लापरवाह और तुनकिमजाज
मानना और उसे छोटी समझकर उसके िवचारों को महत्त्व न दे ना , उसके ॑दय को कचोटता था
|अतः उसने िकट्टी को अपना हमराज़ बनाकर डायरी में उसे ही संबोिधत िकया|
६
‘ऐन ृैंक की डायरी यहिदयों
पर हए
ू
ु जुल्मों का जीवंत दःतावेज है ’पाठ के आधार पर
यहिदयों
पर हए
ू
ु अत्याचारों का िववरण दें ।
उत्तरः िहटलर की नाजी सेना ने यहिदयों
को कैद कर यातना िशिवरों में डालकर
ू
यातनाएँ दी। उन्हें गैस चैंबर में डालकर मौत के घाट उतार िदया जाता था। कई यहदी
ू
भयमःतहोकर अज्ञातवास मेंचले गए जहाँ उन्हें
125 अमानवीय पिरिःथितयों में जीना पड़ा|
अज्ञातवास में उन्हें सेन्धमारों से भी िनबटना पड़ा।। उनकी यहदी
ू संःकृ ित को भी कुचल डाला
गया।
७. ‘डायरी के पन्ने’पाठ िकस पुःतक से िलया गया है ?वह कब ूकािशत हई
ु ?िकसने
ूकािशत कराई?
उत्तरः यह पाठ ऐनृैंक द्वारा डच भाषा में िलखी गई ‘द डायरी ऑफ ए यंग गलर्’ नामक
पुःतक से िलया गया है । यह १९४७ में ऐन ृैंक की मृत्यु के बाद उसके िपता िमःटर ऑटो ृैंक
ने ूकािशत कराई।
अन्य महत्त्वपूणर् अभ्यास-ूश्न:
१ . काश, कोई तो होता जो मेरी भावनाओं को गंभीरता से समझ पाता| अफसोस, ऐसा व्यिक्त
मुझे अब तक नहीं िमला....
| क्या आपको लगता है िकऐन के इस कथन में उसके डायरी
ु
लेखन का कारण छपा
हआ
है ?
ु
२. अज्ञातवास में उबाऊपन दरू करने के िलए ऐन ृैंक व वान पिरवार क्या करते थे ?
३. डच मंऽी िक िकस घोषणा से ऐन रोमांिचत हो उठी?
४. ऐन के अनुसार युद्ध में घायल सैिनक गवर् का अनुभव क्यों कर रहे थे ?
५. Ôहर कोई जानता था िक बुलावे का क्या मतलब होता हैÕ| ऐन की डायरी के आधार पर
िलिखए |
६. ‘ूकृ ित ही तो एक ऐसा वरदान है , िजसका कोई सानी नहीं है ।’ऐसा क्यों कहा गया है ?
126 ूितदशर् ूँन-पऽ
कक्षा : १२
िवषय:िहन्दी (केंििक)
िनधार्िरत समय: ३ घंटे
अिधकतम
अंक: १००
खण्ड क
१. िनम्निलिखत गद्यांश को ध्यान पूवक
र् पिढ़ए तथा पूछे गए ूश्नों के संिक्षप्त उत्तर िलिखए :
लेखक का काम काफी हद तक मधुमिक्खयों के काम से िमलता-जुलता है| मधुमिक्खयाँ
मकरं द- संमह करने के िलए कोसों दरू तक चक्कर लगाती हैं| वे सुद
ं र और अच्छे फूलों
का रसपान करती हैं |तभी तो उनके मधु में संसार का सवर्ौष्ठ
े माधुयर् रहता है | यिद
आप अच्छा लेखक बनना चाहते हैं तो आपको भी यही वृित्त
अपनानी होगी | अच्छे
मंथों का खूब अध्ययन करना होगा और उनके िवचारों का मनन करना होगा | िफर
आपकी रचनाओं में मधु का-सा माधुयर् आने लगेगा | कोई अच्छी उिक्त, कोई अच्छा
िवचार भले ही दसरों
से महण िकया गया हो, लेिकन उस पर यथेष्ट मनन कर आप उसे
ू
अपनी रचना में ःथान दें गे, तो वह आपका ही हो जाएगा | मननपूवक
र् िलखी हई
ु
के संबंध
वःतु
में िकसी को यह कहने का साहस नहीं होगा िक वह अमुक ःथान से ली गई
है या उिच्छष्ट है | जो बात आप अच्छी तरह आत्मसात कर लेंगे, वह मौिलक हो जाएगी
|
(क) अच्छा लेखक बनने के िलए क्या करना चािहए ?
३
(ख) मधुमक्खी एवं अच्छे
३
लेखक में क्या समानताएँ होती हैं ?
(ग) लेखक अपनी रचनाओं में माधुयर् कैसे ला सकता है ?
३
(घ) कोई भी बात मौिलक कैसे बनती है ?
(ङ) मधुमिक्खयों के मधु में संसार की सवर् ौेष्ठ
२
मधुरता कैसे आती है ?
(च) यथेष्ट तथा उिच्छष्ट शब्दों के अथर् िलिखए |
(छ) उपयुक्त
र् गद्यांश का उपयुक्त शीषर्क िलिखए |
२
१
१
२. िनम्निलिखत काव्यांश को ध्यान पूवक
र् पढ़कर पूछे गए ूश्नों के उत्तर िलिखए: १*५=५
साक्षी है इितहास हमीं पहले जागे हैं ,
जामत सब हो रहे हमारे ही आगे हैं ,
127 शऽु हमारे कहाँ नहीं भय से भागे?
कायरता से कहाँ ूाण हमने त्यागे हैं ?
हैं हमीं ूकिम्पत कर चुके, सुरपित तक का भी हृदय
िफर एक बार हे िवश्व गाओ तुम भारत की िवजय !
कहाँ ूकािशत नहीं रहा है तेज हमारा,
दिलत कर चुके शऽु सदा हम पैरों द्वारा,
बतलाओ तुम, कौन नहीं जो हम से हारा,
पर शरणागत हआ
कहाँ, कब हमें न प्यारा
ु
बस युद्ध
माऽ को छोड़कर, कहाँ नहीं हैं हम सदय !
िफर एक बार हे िवश्व! तुम गाओ भारत की िवजय !
(क) Ôहमीं पहले जागे हैंÕ से क्या अिभूाय है ?
(ख) Ôहैं हमीं ूकिम्पत कर चुके, सुरपित तक का भी भी हृदयÕ से भारतवािसयों की िकस
िवशेषता का पता लगता है ?
(ग) हमारी दयालुता का वणर्न किवता की िकन पंिक्तयों में िकया गया है ?
(घ) िकसके जयघोष करने के िलए कहा गया है ?
(ङ) सुरपित तथा शरणागत शब्दों के अथर् िलिखए |
खण्ड ख
३. िनम्निलिखत िवषयों में से िकसी एक पर िनबंध िलिखए :
५
(क) संचार-बांित और भारत
(ख) पिरौम : सफलता की कुंजी
(ग) महँ गाई की समःया
(घ) पराधीन सपनेहुँ सुख नाहीं
४. ःववृत्त (बायोडाटा) ूःतुत करते हए
ु केन्िीय िवद्यालय िवकासपुरी नई िदल्ली के ूाचायर् को
पुःतकालय सहायक के पद हे तु आवेदन-पऽ िलिखए |
अथवा
५
दरदशर्
न के केन्ि िनदे शक को िकसी िवशेष कायर्बम की सराहना करते हए
ू
ु पऽ िलिखए ।
५. (क) िनम्न ूश्नों के संिक्षप्त उत्तर िलिखए-
५
(i) फ़ीडबैक क्या है ?
(ii) जनसंचार के ूमुख माध्यम कौन Ðकौन से हैं ?
(iii) पऽकारीय लेखन से आप क्या समझते है ?
(iv) समाचार के ूमुख तत्त्व िलिखए |
(v) पीत पऽकािरता से क्या अिभूाय है ?
128 (ख) एक िदवसीय हिरद्वार याऽाÕ अथवा बेकार पदाथोर्ं से उपयोगी वःतुएÕँ िवषय पर ूितवेदन
िलिखए ।
६. Õबःते का बढ़ता बोझÕ अथवा Õजातीयता का िवषÕ िवषय पर फ़ीचर िलिखए । ५
खण्ड ग
७. िनम्निलिखत में से िकसी काव्यांश को पढ़कर पूछे गए ूश्नों के उत्तर िलिखए-
२*४
=८
मैं िनज उर के उद्गार िलये िफ़रता हँू ,
मैं िनज उर के उपहार िलये िफ़रता हँू ,
यह अपूणर् संसार न मुझको भाता ,
मैं ःवप्नों का संसार िलये िफ़रता हँू ।
मैं जला हृदय में अिग्न, दहा करता हँू ,
सुख-दख
ु दोनों में मग्न रहा करता हँू ,
जग भव सागर तरने को नाव बनाए,
मैं भव मौजों पर मःत बहा करता हँू ।
(क)
Õिनज उर के उद्गार व
उपहारÕ- से किव का क्या तात्पयर् है ?
(ख)
किव ने संसार को अपूणर् क्यों कहा?
(ग)
किव को संसार अच्छा क्यों
नहीं लगता?
(घ)
खुशी का माहौल कैसे बनाया जा सकता है ?
अथवा
िकसबी िकसान-कुल, बिनक, िभखािर, भाट
चाकर, चपल नट,चोर, चार, चेटकी ।
पेट को पढत,गुन गढत ,चढत िगिर
अटत गहन वन , अहन अखेटकी ।
ऊँचे-नीचे करम,धरम अधरम किर
पेट ही को पचत, बेचत बेटा-बेटकी ।
ÔतुलसीÕ बुझाइ एक राम घनँयाम ही तें ,
आिग बडवािगतें बड़ी हैं आिग पेटकी ||
129 संसार में कष्टों को सह कर भी
(क)
पेट भरने के िलए क्या Ð क्या
करते हैं ?
(ख) किव ने िकन Ð िकन लोगों का वणर्न िकया हैं ?
(ग)
किव के अनुसार पेट की आग
कौन बुझा सकता है ?
(घ)
पेट की आग को बडवािग्न से
भी बड़ा क्यों बताया गया है ?
८.
िनम्निलिखत काव्यांश पर पूछे
गए ूश्नों के उत्तर िलिखए Ð २*३=६
आिखरकार वही हआ
िजसका मुझे डर था
ु
जोर जबरदःती से
बात की चूड़ी मर गई
और वह भाषा में बेकार घूमने लगी !
हार कर मैंने उसे कील की तरह
उसी जगह ठोंक िदया |
(क)बात की चूड़ी मर जाने व बेकार घूमने से किव का क्या आशय
(ख)
है ?
काव्यांश में ूयुक्त दोनों उपमाओं के ूयोग सौंदयर् पर िटप्पणी कीिजए |
(ग) रचनाकार के सामने कथ्य और माध्यम की क्या समःया थी?
अथवा
ु
आंगन में ठनक
रहा है िजदयाया है
बालक तो हई चाँद पे ललचाया है ,
दपर्ण उसे दे के कह रही है माँ
दे ख आइने में चाँद उतर आया है ।
(क)काव्यांश की भाषा की दो िवशेषताओं का उल्लेख कीिजए।
(ख)
यह काव्यांश िकस छं द
में िलखा गया है तथा उसकी क्या िवशेषता है ?
(ग) Õदे ख आइने में चाँद उतर आया हैÕ-कथन के सौंदयर् को ःपष्ट कीिजए ।
९. िकन्हीं दो ूश्नों के उत्तर दीिजए Ð३*२=६
(क)बािन्त की गरजना का शोषक-वगर् पर क्या ूभाव पड़ता है ? उनका मुख ढाँपना िकस
मानिसकता का द्योतक है ?
(ख) िफ़राक की रुबाइयों में उभरे घरे लू जीवन के िबंबों का सौंदयर् ःपष्ट कीिजए ।
(ग) कैमरे में बंद किवता में िनिहत व्यंग्य को उजागर कीिजए ।
१०. गद्यांश को पढ़कर पूछे गए ूश्नों के उत्तर दीिजए Ð २*४=८
130 पर उस जाद ू की जकड़ से बचने का सीधा Ð सा उपाय है | वह यह िक बाजार जाओ तो
खाली मन न
हो | मन खाली हो, तब बाजार न जाओ | कहते हैं लू में जाना हो तो पानी
पीकर जाना चािहए | पानी भीतर हो, लू का लू-पन व्यथर्
हो जाता है | मन लआय में भरा
हो तो बाजार भी फैला Ðका- फैला ही रह जाएगा | तब वह घाव िबल्कुल नहीं
दे सकेगा ,
बिल्क कुछ आनंद ही दे गा | तब बाजार तुमसे कृ ताथर् होगा , क्योंिक तुम कुछ-न-कुछ सच्चा
लाभ उसे दोगे | बाजार की असली कृ ताथर्ता है आवँयकता के साथ काम आना |
(क)बाजार के जाद ू की पकड़ से बचने का सीधा Ð सा उपाय क्या है ?
(ख)
बाजार कब नहीं जाना चािहए और क्यों ?
(ग) बाजार की साथर्कता िकसमें है ?
(घ) बाजार से कब आनंद िमलता है ?
अथवा
एक बार मुझे मालूम होता है िक यह िशरीष एक अदभुत अवधूत है | दःख
हो या
ु
सुख,वह हार नहीं मानता | न ऊधो का लेना, न माधो का दे ना | जब धरती और
आसमान जलते रहते हैं , तब भी यह हजरत न जाने कहाँ से अपना रस खींचते रहते
हैं | मौज में आठों याम मःत रहते हैं | एक वनःपितशास्तर्ी ने मुझे बताया है िक यह
उस ौेणी का पेड़ है जो वायुमड
ं ल से अपना रस खींचता है| जरुर खींचता होगा | नहीं
तो भयंकर लू के समय इतने कोमल तंतुजाल और ऐसे सुकुमार केसर को कैसे उगा
सकता था? अवधूतों के
मुह
ँ से ही संसार की सबसे सरस रचनाएँ िनकली हैं | कबीर
बहत
ु कुछ इस िशरीष के समान थे, मःत और बेपरवा, पर सरस और मादक |
कािलदास भी अनासक्त योगी रहे होंगे | िशरीष के फूल फक्कड़ाना मःती से ही उपज
सकते हैं और Ôमेघदत
ू Ô का काव्य उसी ूकार के अनासक्त, अनािवल, उन्मुक्त हृदय में
उमड़ सकता है | जो किव अनासक्त नहीं रह सका, जोफक्कड़ नहीं बन सका, जो
िकए Ð िकराए का लेखा- जोखा िमलाने में उलझ गया, वह क्या किव है ?
(क)लेखक ने िशरीष को क्या संज्ञा दी है तथा क्यों ?
(ख)
वनःपितशास्तर्ी के अनुसार िशरीष कैसे जीिवत रहता है ?
(ग) Ôअवधूतों के मुहँ से ही संसार की
सबसे सरस रचनाएँ िनकली हैंÕ- आशय ःपष्ट
करें |
(घ) लेखक ने सच्चे किव के बारे में क्या बताया है ?
१ १. िकन्हीं चार ूश्नों के उत्तर दीिजए Ð३*४=१२
(क)चालीर् चैिप्लन कौन था ? उसके भारतीयकरण से लेखक का क्या आशय है ?
(ख) भिक्तन द्वारा शास्तर् के ूश्न को सुिवधा से सुलझा लेने का क्या उदाहरण लेिखका ने िदया
है ?
131 (ग) Ôगगरी फूटी बैल िपयासाÕ इन्दर सेना के इस खेलगीत में बैलों को प्यासा रहने की बात
क्यों मुखिरत हई
ु है ?
(घ) लुट्टन के राजपहलवान लुट्टन िसंह बन जाने के बाद की िदनचयार् पर ूकश डािलए ।
(ङ) आदशर् समाज के तीन तत्त्वों में
समाज ॆें िस्तर्यों
से एक ॅातृता को रखकर लेखक ने अपने आदशर्
को भी सिम्मिलत िकया है अथवा नहीं ? आप इस ॅातृता शब्द से
कहाँ तक सहमत हैं ?
१२. िकन्हीं दो ूश्नों के उत्तर दीिजए Ð ३*२=६
(क) डायरी के पन्नेÕ के आधार पर मिहलाओं के बारे में ऐन के िवचारों पर िटप्पणी
कीिजए।
(ख) अपने घर में अपनी Õिसल्वर वैिडं गÕ के आयोजन में भी यशोधर बाबू की अनेक बातें
Õसम हाउ इं ूोपरÕ लग रही थीं , ऐसा क्यों ?
(ग) ÔजूझÕ शीषर्क को उपयुक्त ठहराने
के िलए तीन तकर् दीिजए ।
१३. िकन्हीं दो ूश्नों के उत्तर संक्षेप में दीिजए-२*२=४
(क)िसन्धु-सभ्यता के सबसे बड़े शहर मुअन-जोदड़ो की नगर-योजना दशर्कों को अिभभूत क्यों
करती है ? ःपष्ट कीिजए ।
(ख) ÕजूझÕ कहानी में दे साई सरकार की भूिमका पर ूकश डािलए ।
(ग) ऐसी दो िवशेषताओं का उल्लेख कीिजए जो सेक्सन ऑिफसर वाई.डी. पंत को अपने रोल
मॉडल िकसन दा से उत्तरािधकार में िमली थी ।
१४. Ôमुअनजोदड़ोÕ के खनन से ूाप्त जानकािरयों के आधार पर िसन्धु-सभ्यता की िवशेषताओं पर
एक लेख िलिखए ।
५
अथवा
Õिसल्वर वैिडं गÕ कहानी के आधार पर यशोधर बाबू के व्यिक्तत्व की चार िवशेषताओं पर
सोदाहरण ूकश डािलए ।
132 ूितदशर् ूश्न-पऽ-२
कक्षा: बारहवीं
िवषय- िहन्दी(केिन्िक)
िनधार्िरत समय:3घण्टे
अिधकतम अंक-100
िनदेर् श: िवद्याथीर् जाँच कर लें िक–
¾ ूश्नपऽ में कुल १४ ूश्न हैं |
¾ सभी ूश्न ठीक से छपे हैं ।
¾ ूश्न का उत्तर िलखने से पूवर् ूश्न का बमांक अवँय िलखें।
¾
इस ूश्न पऽ को पढ़ने के िलए 15 िमनट का समय िदया गया है ।
खण्ड-क
प्रश्न-1. िनम्निलिखत काव्यांश को पढ़कर पूछे गए ूश्नों के उत्तर दीिजए:- (1*5=5)
तूफानों की ओर घुमा दो
नािवक ! िनज पतवार।
आज िसंधु ने िवष उगला है
लहरों का यौवन मचला है
आज हृदय में और िसंधु में
साथ उठा है ज्वार ।
तूफानों की ओर घुमा दो
नािवक ! िनज पतवार ।
लहरों के ःवर में कुछ बोलो
इस अंधड में साहस तोलो
कभी-कभी िमलता जीवन में
133 तूफानों को प्यार ।
तूफानों की ओर घुमा दो
नािवक ! िनज पतवार
यह असीम,िनज सीमा जाने
सागर भी तो यह पहचाने
िमट्टी के पुतले मानव ने
कभी न मानी हार ।
• (क) उपयुक्त
र् काव्यांशकाउपयुक्तशीषर्कदीिजए।
(1)
• (ख) किवकेहृदयऔरिसंधम
ु ेंक्यासमानताहै ?
(1)
• (ग) जबजीवनमेंतूफानोंकाप्यारिमलेतोक्याकरनाचािहएऔरक्यों?
(1)
• (घ) किवसागरकोक्यासमझानाचाहताहै ?
• (ड) ूःतुतकाव्यांशकामूलभावक्याहै ?
प्रश्न-2. िनम्निलिखत गद्यांश को पढकर पूछे गए ूश्नों के उत्तर दीिजए :
(1)
(1)
(15)
सुचािरत्र्य के दो सशक्त ःतंभ हैं -ूथम सुसः
ं कार और िद्वतीयसत्संगित। सुसः
ं कार भी पूवर्
जीवन की सत्संगित व सत्कमोर्ं की अिजर्त संपित्त है और सत्संगित वतर्मान जीवन की दलर्
ु भ
िवभूित है ।िजस ूकार कुधातु की कठोरता और कािलख, पारस के ःपशर् माऽ से कोमलता और
कमनीयता में बदल जाती है ,ठीक उसी ूकार कुमागीर् का कालुःय सत्संगित से ःविणर्म आभा में
पिरवितर्त हो जाता है ।सतत सत्संगित से िवचारों को नई िदशा िमलती है और अच्छे िवचार
मनुंय को अच्छे कायोर्ं में ूेिरत करते हैं ।पिरणामत: सुचिरऽ का िनमार्ण होता है । आचायर् हजारी
ूसाद िद्ववेदी ने िलखा है -
महाकिव टै गोर के पास बैठने माऽ से ऐसा ूतीत होता था मानो
भीतर का दे वता जाग गया हो।
वःतुत: चिरऽ से ही जीवन की साथर्कता है ।चिरऽवान व्यिक्तसमाज की शोभा है , शिक्त
है ।सुचािरत्र्य से व्यिक्त ही नहीं,समाज भी सुवािसत होता है और इस सुवास से राष्टर् यशःवी बनता
है । िवदरजी
की उिक्त अक्षरश: सत्य है िक सुचिरऽ के बीज हमें भले ही वंश-परं परा से ूाप्त हो
ु
134 ूाप्त हो सकते हैं पर चिरऽ-िनमार्ण व्यिक्त के अपने बलबूते पर िनभर्र है । आनुवंिशक
परं परा,पिरवेश और पिरिःथित उसे केवल ूेरणा दे सकते हैं पर उसका अजर्न नहीं कर सकते;
वह व्यिक्त को उत्तरािधकार में ूाप्त नहीं होता।
व्यिक्त-िवशेष के िशिथल चिरऽ होने से पूरे राष्टर् पर चिरऽ-संकट उपिःथत हो जाता है क्योंिक
व्यिक्त पूरे राष्टर् का एक घटक है ।अनेक व्यिक्तयों से िमलकर एक पिरवार,अनेक पिरवारों से एक
कुल, अनेक कुलों से एक जाित या समाज और अनेकानेक जाितयों और समाज-समुदायों से
िमलकर ही एक राष्टर् बनता है । आज जब लोग राष्टर्ीय चिरऽ-िनमार्ण की बात करते हैं , तब वे
ःवयं उस राष्टर् के एक आचरक घटक हैं - इस बात को िवःमृत कर दे ते हैं ।
1. सत्संगितकुमागीर्कोकैसेसध
ु ारतीहै ?सोदाहरणःपष्टकीिजए।
2. चिरऽकेबारे मेंिवदरक
ु े क्यािवचारहैं ?
(2)
(2)
3. व्यिक्त-िवशेषकाचिरऽसमूचेराष्टर्कोकैसेूभािवतकरताहै ?
4. व्यिक्तकेचािरऽ-िनमार्णमेंिकस-िकसकायोगदानहोताहै ?
(2)
(2)
5. संगितकेसंदभर्मेंपारसकेउल्लेखसेलेखकक्याूितपािदतकरनाचाहताहै ?(2)
6. व्यिक्तसुसः
ं कृ तकैसेबनताहै ?ःपष्टकीिजए।
(1)
7. आचरणउच्चबनानेकेिलएव्यिक्तकोक्याूयासकरनाचािहए?
(1)
8. गद्यांशकाउपयुक्तशीषर्कदीिजए।
(1)
9. ’सु’और’कु’उपसगोर्ंसेएक-एकशब्दबनाइए।
10. ’चिरऽवान’और‘पिरवेश’मेंूयुक्तउपसगर्औरूत्ययअलगकीिजए।
(1)
(1)
खण्ड- ख
प्रश्न-3. िनम्निलिखत में से िकसी एक िवषय पर िनबंध िलिखए(क) ॅष्टाचार: कारण और िनवारण
(ख) इक्कीसवीं सदी का भारत
(ग) बदलते समाज में मिहलाओं की िःथित
(घ) बढता ूदषण
और जन-ःवाःथ्य
ू
135 (5)
प्रश्न-4. िकसी दै िनक समाचार-पऽ के सम्पादक को जान्ह्वी की ओर से एक पऽ
िलिखए,िजसमें िनरं तर मँहगी होती िशक्षा को लेकर िचंता ूकट की गई हो ।
(5)
अथवा
रे ल याऽा के दौरान साधारण ौेणी के यािऽयों को ःटे शनों एवं चलती गािडयों में
िमलने वाली खान-पान की साममी संतोषजनक नहीं होती। इस समःया की ओर
अिधकािरयों का ध्यान आकृ ष्ट करने के िलए अधीक्षक,खान-पान िवभाग,रे ल
भवन,नई िदल्ली के नाम पऽ िलिखए।
प्रश्न-5. (अ)िनम्निलिखत ूश्नों के संिक्षप्त उत्तर दीिजए• (क) ’उल्टािपरािमड’शैलीसेआपक्यासमझतेहैं?
(1)
• (ख) ‘वॉचडॉग’पऽकािरतासेक्याआशयहै ?
(1)
• (ग) लाइविकसेकहतेहैं?
(1)
• (घ) िहं दीकेिकन्हींदोराष्टर्ीयसमाचारपऽोंकेनामिलिखए।
(1)
• (ड.) जनसंचारकेिकन्हींतीनकायोर्ंकाउल्लेखकीिजए।
(1)
(आ) िनम्निलिखत में से िकसी एक िवषय पर आलेख िलिखए।
(5)
(क) खेतीकी जमीन पर फैक्टरी लगाने को लेकर चलने वाली बहस में भाग लेते हए
ु आलेख
िलिखए ।
(ख) आजादी के साठ सालों में गणतांिऽक मूल्यों का ॑ास हआ
है । ॅष्टाचार ने हर क्षेऽ को
ु
मःत कर रखा है ।Ðइस िवषय पर एक आलेख तैयार कीिजए।
प्रश्न-6. Ôओलिम्पक खेलÕअथवा Ôमहानगरों में बढ़ते अपराध की समःयाÕ पर लगभग 150
शब्दों में एक फीचर तैयार कीिजए।
136 (5)
खण्ड-ग
प्रश्न-7. िनम्निलिखत में से िकसी एक काव्यांश को पढ़कर पूछे गए ूश्नों के उत्तर दीिजए(२*4=8)
मैं यौवन का उन्माद िलए िफरता हँू ,
उन्मादों में अवसाद िलए िफरता हँू ,
जो मुझको बाहर हँ सा, रुलाती भीतर,
मैं, हाय, िकसी की याद िलए िफरता हँू ,
कर यत्न िमटे सब,सत्य िकसी ने जाना ?
नादान वहीं है , हाय, जहाँ पर दाना !
िफर मूढ न क्या जग, जो इस पर भी सीखे
मैं सीख रहा हँू , सीखा ज्ञान भुलाना !
(क)किव जीवन में क्या िलए घूमता है ?
(2)
(ख)किव को बाहर-भीतर क्या हँ साता-रुलाता है ?
(2)
(ग)‘नादान वही हैं , हाय,जहाँ पर दाना!’Ðकिव ने ऐसा क्यों कहा होगा ?
(2)
(घ) किव ने यह क्यों कहा िक सत्य िकसी ने नहीं जाना ?
(2)
अथवा
खेती न िकसान को,िभखारी न भीख,बिल,
बिनक को बिनज,न चाकर को चाकरी ।
जीिवका िबहीन लोग सीद्यमान सोच बस,
कहें एक एकन सों ‘कहाँ जाई, का करी ?’
बेदहँू पुरान कही, लोकहँू
िबलोिकअत,
137 साँकरें सबैं पै, राम! रावरें कृ पा
करी।
दािरद-दसानन दबाई दनी
ु , दीनबंधु !
दिरत
ु -दहन दे िख तुलसी हहा करी ॥
(क) येपिं क्तयाँिकसकिवतासेलीगईहैं औरइसकेकिवकौनहैं ?
(ख)
(२)
तुलसीकेसमयकीआिथर्कदशाकैसीथी?
(ग) वेदोंमेंक्याकहागयाहै ?
(2)
(2)
(घ)तुलसीनेरावणकीतुलनािकससेकीहै औरक्यों?
प्रश्न-8. िनम्निलिखत काव्यांश पर पूछे गए ूश्नों के उत्तर दीिजए:
ूात नभ था बहत
ु नीला शंख जैसे
भोर का नभ
राख से लीपा हआ
चौका
ु
(अभी गीला पड़ा है)
बहत
ु काली िसल जरा से लाल केसर से
िक जैसे धुल गई हो
ःलेट पर या लाल खिड़या चाक
मल दी हो िकसी ने ।
(क)ूःतुत काव्यांश का भाव-सौंदयर् ःपष्ट कीिजए ।
(ख)कव्यांश में ूयुक्त उपमा अलंकार का उदाहरण चुनकर िलिखए ।
(ग)उपयुक्त
र् काव्यांश की भाषा की दो िवशेषताएं बताइए।
अथवा
138 (2)
(२*3=6)
नहला के छलके-छलके िनमर्ल जल से
उलझे हए
ु
में कंघी करके
ु गेसओं
िकस प्यार से दे खता है बच्चा मुह
ँ को
जब घुटिनयों में ले के है िपन्हाती कपड़े ।
(क) ूःतुत काव्यांश का भाव-सौंदयर् ःपष्ट कीिजए ।
(ख) ूःतुत काव्यांश की भाषा संबंधी िवशेषताएं बताइए।
(ग)पुनरुिक्त ूकाश व अनुूास अलंकार छाँिटए।
प्रश्न-9. िनम्निलिखत में से िकन्हीं दो ूश्नों के उत्तर दीिजए-
(3+3)
(क)
‘भाषा को सहिलयत
’ से बरतने से क्या अिभूाय है ?
ू
(ख)
किव के पास जो कुछ अच्छा-बुरा है , वह िविशष्ट और मौिलक कैसे है ?‘सहषर्
ःवीकारा है ’ किवता के आधार पर उत्तर दीिजए ।
(ग)
‘कैमरे में बंद अपािहज’ करुणा के मुखौटे में िछपी बूरता की किवता है Ð िवचार
कीिजए।
प्रश्न-10.
िनम्निलिखत में से िकसी एक गद्यांश को पढकर पूछे गए ूश्नों के उत्तर
दीिजए -(२*4=8)
बाजार में एक जाद ू है ।वह जाद ू आँख की राह काम करता है वह रुप का जाद ू है
पर जैसे चुंबक का जाद ू लोहे पर ही चलता है ,वैसे ही इस जाद ू की भी मयार्दा है ।जेब भरी
हो, और मन खाली हो, ऐसी हालात में जाद ू का असर खूब होता है ।जेब खाली पर मन
भरा न हो तो भी जाद ू चल जायेगा। मन खाली है तो बाजार की अनेकानेक चीजों का
िनमंऽण उस तक पहँु च जाएगा।कहीं उस वक्त जेब भरी हो तब तो िफर वह मन िकसकी
मानने वाला है ! मालूम होता है यह भी लू,ँ वह भी लूँ।सभी सामान जरुरी और आराम को
बढाने वाला मालूम होता है । पर यह सब जाद ू का असर है ।जाद ू की सवारी उतरी िक
फैंसी चीजों की बहतायत
आराम में मदद नहीं दे ती, बिल्क खलल ही डालती है ।
ु
139 (क)बाजार के जाद ू को ‘रुप का जाद’ू क्यों कहा गया है ?
(ख)बाजार के जाद ू की मयार्दा ःपष्ट कीिजए।
(ग)बाजार का जाद ू िकस ूकार के लोगों को लुभाता है ?
(घ)इस जाद ू के बंधन से बचने का क्या उपाय हो सकता है ?
अथवा
चालीर् की अिधकांश िफल्में भाषा का इःतेमाल नहीं करती इसिलए उन्हें ज्यादा से ज्यादा
मानवीय होना पडा।सवाक् िचऽपट पर कई बडे -बडे कॉमेिडयन हए
ु हैं , लेिकन वे चैिप्लन की
सावर्भौिमकता तक क्यों नहीं पहँु च पाए इसकी पड़ताल अभी होने को है । चालीर् का िचर-युवा
होना या बच्चों जैसा िदखना एक िवशेषता तो है ही,सबसे बड़ी िवशेषता शायद यह है िक वे
िकसी भी संःकृ ित को िवदे शी नहीं लगते। यानी उनके आसपास जो भी चीजें,अड़ं गें,
खलनायक, दष्ट
ु औरतें आिद रहते हैं वे एक सतत ‘िवदे श’ या ‘परदे श’ बन जाते हैं और
चैिप्लन ‘हम’ बन जाते हैं । चालीर् के सारे संकटों में हमें यह भी लगता है िक यह ‘मै’ भी हो
सकता हँू , लेिकन ‘मै’ से ज्यादा चालीर् हमें ‘हम’ लगतेहैं। यह संभव है िक कुछ अथोर्ं में
‘बःटर कीटन’ चालीर् चैिप्लन से बड़ी हाःय-ूितभा हो लेिकन कीटन हाःय का काफ्का है
जबिक चैिप्लन ूेमचंद के ज्यादा नजदीक हैं ।
(क)चालीर् की िफल्मों को मानवीय क्यों होना पड़ा?
(ख)चालीर् चैिप्लन की सावर्भौिमकता का क्या कारण है ?
(ग) चालीर् की िफल्मों की िवशेषता क्या है ?
(घ) चालीर् के कारनामें हमें ‘मैं’ न लगकर ‘हम’ क्यों लगते हैं ?
प्रश्न-11.
िनम्निलिखत में से िकन्हीं चार ूश्नों के उत्तर दीिजए-
(४*3=12)
(क)भिक्तन और लेिखका के बीच कैसा संबंध था ?‘भिक्तन’ पाठ के आधार पर बताइए ।
140 (ख)िदनों-िदन गहराते पानी के संकट से िनपटने के िलए क्या आज का युवावगर् ‘काले
मेघापानी दे ’ की इं दर सेना की तजर् पर कोई सामूिहक आंदोलन ूारम्भ कर
सकता है ? अपने
िवचार िलिखए ।
(ग)लुट्टन पहलवान ने ऐसा क्यों कहा होगा िक मेरा गुरु कोई पहलवान नहीं, यही ढ़ोल
है ?
(घ) ‘मानिचऽ पर एक लकीर खींच दे ने भर से जमीन और जनता बँट नहीं जातीहै ’उिचत तकोर्ं व उदाहरणों के जिरए इसकी पुिष्ट कीिजए ।
(ड.)िद्ववेदी जी ने िशरीष को कालजयी अवधूत(संन्यासी) की तरह क्यों कहा है ?
प्रश्न-12.
िनम्निलिखत में से िकन्हीं दो ूश्नों के उत्तर दीिजए-
(3+3=6)
(क)‘िसल्वरवैिडं ग’
में यशोधर बाबू एक ओर जहाँ बच्चोंकी तरक्की से खुश होते हैं ,वहीं कुछ
‘समहाउइं ूॉपर’ भी अनुभवकरते हैं , ऐसा क्यों ?
(ख)
मोहं जोदडो की गृह-िनमार्ण योजना पर संक्षेप में ूकाश डािलए ।
(ग) ऐन ृेंक कौन थी, उसकी डायरी क्यों ूिसद्ध है ?
ूश्न-13 िनम्निलिखत में से िकन्हीं दो ूश्नों के उत्तर दीिजए-
(2+2=4)
(क)सौंदलगेकर कौन थे तथा उनकी क्या िवशेषता थी ?
(ख)यशोधर बाबू िकससे ूभािवत थे ?
(ग़)ऐन ने 13जून, 1944 के िदन िलखी अपनी डायरी में क्या बताया है ?
ूश्न-14 िनम्निलिखत में से िकसी एक ूश्न का उत्तर दीिजए-
(5)
‘िसंधु सभ्यता की खूबी उसका सौंदयर्-बोध है जो राज-पोिषत या धमर्-पोिषत न होकर समाजपोिषत था।‘ ऐसा क्यों कहा गया ?
अथवा
‘िसल्वर वैिडं ग’ कहानी के आधार पर यशोधर बाबू के चिरऽ की िवशेषताओं पर ूकाश डािलए
।
141 ूितदशर् ूश्न-पऽ-३
िहन्दी (केिन्िक)
कक्षा-बारह
िनधार्िरत समय- 3 घण्टे
अिधकतम अंक-100
िनदेर् शः-इस ूश्न पऽ में तीन खंड
हैं - क ख और ग। सभी खंडों के उत्तर िलखना अिनवायर् है ।
कृ पया ूश्न का उत्तर िलखना शुरू करने से पहले, ूश्न का बमांक अवँय िलखें।
खण्ड-क
ू0-1 िनम्निलिखत काव्यांश को पढ़कर पूछे गए ूश्नों के उत्तर दीिजएपर िजन्होंने ःवाथर्वश जीवन िवषाक्त बना िदया है
कोिट-कोिट बुभिु क्षतों का कौर तलक छीन िलया है
लाभ-शुभ िलखकर जमाने का हृदय चूसा िजन्होंने
और कल बंगालवाली लाश पर थूका िजन्होंने।
िबलखते िशशु की व्यथा पर दृिष्ट तक िजनने न फेरी
यिद क्षमा कर दँ ू इन्हें िधक्कार माँ की कोख मेरी,
चाहता हँू ध्वंस कर दे ना िवषमता की कहानी
हो सुलभ सबको जगत में वस्तर्,भोजन,अन्न, पानी
नव-भवन िनमार्ण िहत में जजर्िरत ूाचीनता का
गढ़ ढहाता जा रहा हँू
पर तुम्हें भूला नहीं हँू ।
िवश्वास बढ़ता ही गया
142 क)
ूथम चार पंिक्तयों में भारत की िकन राजनीितक और सामािजक िःथित का वणर्न है 1
ख)
किव िकन लोगों को क्षमा नहीं करना चाहता?
ग)
किव जगत के िहत के िलए क्या करना चाहता है ? 1
घ)
पर तुम्हें भूला नहीं हँू िकसके िलए संबोधन
ङ)
काव्यांश का उपयुक्त शीषर्क दीिजए। 1
है ?
1
1
ू0-2 िनम्निलिखत गद्यांश को पढ़कर पूछे गए ूश्नों के उत्तर दीिजए।
आज गांधी िचंतन
की शाश्वतता महत्ता एवं ूासंिगकता उनके अिहं सा दशर्न के
कारण ही है । महात्मा गांधी के दशर्न की नींव अिहं सा की व्यापकता को वैयिक्तक आचरण तक
सीिमत न करके जीवन के ूत्येक क्षेऽ-धािमर्क, नैितक, सामािजक, आिथर्क और राजनैितक क्षेऽों
में सफलतापूवक
र् ूयुक्त िकया।
भारत में ही नहीं वरन ् संसार के अन्य धमोर्ं में भी अिहं सा के िसद्धान्त को महत्त्व ूदान िकया
गया है । इःलाम धमर् के ूवतर्क हजरत मुहम्मद साहब के उपदे शों में अिहं सा के पालन का
आमह है । यहिदयों
के धमर्मन्थों का महत्त्वपूणर् ःथान रहा है । ईसा मसीह का संपूणर् जीवन
ू
अिहं सा का अिभव्यक्तीकरण है । सुकरात के अिहं सा पालन का उदाहरण अिवःमरणीय है । अतः
ःपष्ट है िक अिहं सा का िसद्धांत
अिपतु इितहास में उसके संदभर् की कहानी बहत
ु ूाचीन और
िवःतृत है ।
यद्यिप अिहं सा भारतीय दशर्न व धमर् के िलए अित ूाचीन है परन्तु गांधी की िवशेषता
इस तथ्य में है िक उन्होंने अिहं सा के परम्परागत िसद्धांत को आत्मसात ् कर अपने अनुभव से
उसकी ःमृितयों को नया आयाम िदया। उन्होंने अिहं सा को व्यिक्तगत जीवन का ही नहीं वरन ्
सामािजक जीवन का िनयम बनाकर उसका व्यावहािरक ूयोग िकया।
क)
‘आज’से यहाँ क्या तात्पयर् है ?गांधी िचंतन ’से आप क्या समझते हैं ?
ख)
अिहं सा का िसद्धांत िवश्व के अन्य िकन धमर्शास्तर्ों में भी विणर्त है ? 2
घ)
गद्यांश के दसरे
अनुच्छे द को पढ़कर अिहं सा के िसद्धांत के बारे में आपकी क्या धारणा
ू
बनती है ?
ङ)
2
गांधी जी के अिहं सा िसद्धान्त की क्या िवशेषता है ?जो परं परागत तरीकों से हटकर है । 2
143 2
च)
अिहं सा के व्यावहािरक ूयोग से आप क्या समझते हैं ?2
छ)
गद्यांश के िलए उपयुक्त शीषर्क दीिजए।
ज)
उपसगर् और ूत्यय अलग कीिजए।
1
अिहं सा,सामािजक
झ)
1
रचना के अनुसार वाक्य भेद बताइए।
ईसा मसीह का सम्पूणर् जीवन अिहं सा का अिभव्यक्तीकरण है ।
ङ)
1
िवशेषण बनाइएशाश्वतता, ूासंिगकता 1
खण्ड-ख
ू0-3 िवद्यालय में खेल सामिमयों की कमी की िशकायत करते हए
ु ूाचायर् को पऽ िलिखए।
अथवा
िवद्यालय के चौराहे पर अराजक तत्वों की भीड़ की िशकायत करते हए
ु पुिलस अधीक्षक को पऽ
िलिखए।
5
ू0-4 िकसी एक िवषय पर िनबंध िलिखएक)
भारत की वैज्ञािनक उपलिब्धयाँ
ख)
सवर्िशक्षा अिभयान
ग)
साम्ूादाियकता: एक अिभशाप
घ)
कम्प्यूटर का महत्व
ू0-5(अ)
5
गंगा ूदषण
के ूित जागरूकता पैदा करने के िलए िकसी दै िनक समाचार पऽ के
ू
िलए संपादकीय तैयार कीिजए।
अथवा
144 Ôमिहला आरक्षण: समाज की आवँयकताÕ शीषर्क पर आलेख ूःतुत कीिजए।
ब)
िनम्निलिखत ूश्नों के संिक्षप्त उत्तर िलिखए- 1
क)
एडवोकेसी पऽकािरता िकसे कहते हैं ?
ख)
छापाखाना का आिवंकार कब हआ
ु ?
(ग)
इं टरनेट पर ूकािशत होने वाला पहला समाचार-पऽ कौन था?
(घ)
पऽकािरता का छः ककार क्या है ?
(ङ)
संपादक िकसे कहते हैं ?
5
ू0-6 विरष्ठ नागिरकों के ूित िजम्मेदारी िवषय पर फीचर आलेख तैयार कीिजए।
अथवा
पोिलयो बूथ के दृँय की िरपोटर् तैयार कीिजए।
5
खण्ड-ग
ू0-7 िनम्निलिखत काव्यांश के आधार पर पूछे गए ूश्नों के उत्तर िलिखए।
बात सीधी थी पर एक बार
भाषा के चक्कर में
जरा टे ढ़ी फँस गई।
उसे पाने की कोिशश में
भाषा को उलटा पलटा
तोड़ा मरोड़ा,
घुमाया िफराया
िक बात या तो बने,
या िफर भाषा से बाहर आए145 लेिकन इससे भाषा के साथ-साथ
बात और भी पेचीदा होती चली गई।।
(क)
बात िकस ूकार भाषा के चक्कर में फँस जाती है ?
(ख)
भाषा को तोड़ने मरोड़ने के पीछे किव का क्या ूयोजन था?
(ग)
उपयुक्त
र् पंिक्त में िनिहत व्यंग्य को ःपष्ट करें ।
(घ)
किव ने बात को बन जाने के िलए क्या सुझाव िदया है ?
अथवा
जन्म से ही वे अपने साथ लाते हैं कपास
पृथ्वी घूमती हई
ै पैरों के पास
ु आती है उनके बेचन
जब वे दौड़ते हैं बेसध
ु
छतों को भी नरम बनाते हए
ु
िदशाओं को मृदंग की तरह बजाते हए
ु
जब वे पेंग भरते हए
ु चले आते हैं
डाल की तरह लचीले वेग से अकसर।।
ू0-8 हो जाए न पथ में रात कहीं
मंिजल भी है तो दरू नहीं
यह सोच थका िदन का पंथी भी जल्दी-जल्दी क्यों चलता है ?
2
(क)
काव्यांश की भाषा की िवशेषताओं का उल्लेख करें ।
(ख)
पथ शब्द एवं मंिजल शब्द का ूयोग िकस रूप में िकया गया है ?
(ग)
िदन का पंथी जल्दी-जल्दी क्यों चलता है ?
अथवा
146 ु
आँगन में ढनक
रहा है िज़दयाया है
बालक तो हई चाँद पै ललचाया है
दपर्ण उसे दे दे के कह रही है माँ
दे ख आइने में चाँद उतर आया है
(क)
ये पिक्तयाँ िकस छं द में िलखी गई हैं ? पद की िवशेषता को ःपष्ट करें ।
(ख)
भाषागत सौदयर् की िकन्हीं दो िवशेषताओं का उल्लेख करें ।
(ग)
उपयुक्त
र् पंिक्तयों का भाव सौंदयर् ःपष्ट करें ।
ू0-9 िनम्निलिखत में से िकन्हीं दो ूश्नों के उत्तर दीिजए-
3
(क)
भोर के नभ को राख से लीपा हआ
चौका क्यों कहा गया है ?
ु
(ख)
किव को बहलाती, सहलाती आत्मीयता बरदाःत क्यों नहीं होती है ?
(ग)
किव नें िकसे सहषर् ःवीकारा है ?
ू0-10 नीचे िदए गए गद्यांश के आधार पर पूछे गए ूश्नों के उत्तर दें -
2
भोजन के समय जब मैंने अपनी िनिश्चत सीमा के भीतर िनिदर् ष्ट ःथान महण कर िलया,
तब भिक्तन ने ूसन्नता से लबालब दृिष्ट और आत्मतुिष्ट से आप्लािवत मुःकराहट के साथ मेरी
फूल की थाली में एक अंगल
ु मोटी और गहरी काली िचत्तीदार चार रोिटयाँ रखकर उसे टे ढ़ी कर
गाढ़ी दाल परोस दी। पर जब उसके उत्साह पर तुषारापात करते हए
ु मैंने रुआँसे भाव से कहायह क्या बनाया है ?तब हो वह हतबुिद्ध हो रही।
(क)
लेिखका की थाली में कैसी रोिटयाँ थीं? रोिटयों के िचत्तीदार होने के क्या कारण हो सकते
हैं ?
(ख)
भिक्तन की ूसन्नता से लबालब दृिष्ट एवं आत्मतुिष्ट के क्या कारण हो सकते हैं ?
(ग)
खाने के आसन पर लेिखका क्यों रुआँसी हो गई?
(घ)
भिक्तन क्यों हतबुिद्ध हो गई?
147 अथवा
अब तक सिफया का गुःसा उतर चुका था। भावना के ःथान पर बुिद्ध धीरे -धीरे उस ःथान पर
हावी हो रही थीं नमक की पुिड़या तो ले जानी है , पर कैसे? अच्छा, अगर इसे हाथ में ले लें
और कःटम वालों के सामने सबसे पहले इसी को रख दें ?लेिकन अगर कःटमवालों ने न जाने
िदया! तो मजबूरी है , छोड़ दें गें। लेिकन िफर उस वायदे का क्या होगा जो हमने अपनी माँ से
िकया था?हम अपने को सैयद कहते हैं । िफर वायदा करके झुठलाने के क्या मायने? जान दे कर
भी वायदा पूरा करना होगा। मगर कैसे?अच्छा! अगर इसे कीनुओं की टोकरी में सबसे नीचे रख
िलया जाए तो इतने कीनुओं के ढे र में भला कौन इसे दे खेगा? और अगर दे ख िलया? नहीं
जी,फलों की टोकिरयाँ तो आते वक्त भी िकसी की नहीं दे खी जा रही थीं। इधर से केले, इधर से
कीनू सब ही ला रहे थे, ले जा रहे थे। यही ठीक है ,िफर दे खा जाएगा।
(क)
सिफया का गुःसा क्यों उतर गया था?
(ख)
सिफया की क्या भावना थी और वह उसकी बुिद्ध के सामने िकस ूकार पराःत हो गई?
(ग)
सिफया की उधेड़बुन का क्या कारण है ?
(घ)
सिफया ने िकस वायदे को पूरा करने की बात की है ?उसे उसने िकस ूकार पूरा िकया।
ू0-11 िनम्निलिखत ूश्नों में से िकन्हीं चार ूश्नों के उत्तर दीिजए -3
(क)
लेखक ने िकस उद्दे ँय से भगत जी का उल्लेख िकया है ?उनका कौन-सा व्यिक्तत्व
उभरकर हमारे सामने आया है ?
(ख)
इन्दर सेना सबसे पहले गंगा भैया की जय क्यों बोलती है ?निदयों का भारतीय
सामािजक,सांःकृ ितक,पिरवेश में क्या महत्व है ?
(ग)
लुट्टन पहलवान ढोलक को ही अपना गुरू क्यों मानता है ?
(घ)
लेखक ने िशरीष के पेड़ को कालजयी अवधूत की तरह क्यों माना है ?
(ङ)
लेखक िकन तकोर्ं के आधार पर जाित ूथा को अमानवीय एवं अलोकतांिऽक बताया है ?
ू0-12 िनम्निलिखत में से िकन्हीं दो ूश्नों के उत्तर िलिखए।
148 3
(क)
Ôडायरी के पन्ने’पाठ के आधार पर मिहलाओं के ूित ऐन ृैंक के दृिष्टकोण को ःपष्ट
कीिजए।
(ख)
जूझ’कहानी का ूितपाद्य संक्षेप में
ःपष्ट कीिजए।
(ग)
Ôिसल्वर वैिडं गÕ कहानी के आधार पर िसद्ध कीिजए िक िकशन दा की परं परा िनभाते हए
ु
यशोधर पंत वतर्मान के साथ नहीं चल पाए।
ू0-13 ौी सौंदलगेकर के व्यिक्तत्व की उन िवशेषताओं पर ूकाश डािलए, िजनके कारण जूझ’के
लेखक के मन में
किवता के ूित लगाव उत्पन्न हआ।
ु
अथवा
‘िसल्वर वैिडं ग’में
एक ओर िःथित को ज्यों-का-त्यों ःवीकार लेने का भाव है तो दसरी
ओर
ू
अिनणर्य की िःथित भी। कहानी के इस द्वन्द्व को ःपष्ट कीिजए।
5
ू0-14 िनम्निलिखत में से िकन्हीं दो ूश्नों के उत्तर संक्षेप में दीिजए।
(क)
Ôिसल्वर वेिडं ग’के आधार पर यशोधर बाबू के सामने
2
आई िकन्हीं
दो Ôसमहाव
इं ूॉपर’िःथितयों का उल्लेख कीिजए।
(ख)
Ôऐन की डायरी’उसकी िनजी भावनात्मक उथल-पुथल का दःतावेज भी है । इस कथन की
िववेचना कीिजए।
ग) क्या िसंधु घाटी की सभ्यता को जल संःकृ ित कह सकते हैं ? ःपष्ट कीिजए।
149 उपयोगी सािहित्यक बेबसाईट
1. www.download.com
2. www.google.com
3. www.foldermarker.com
4. www.webdunia.com
5. www.holy-statues.com
6. www.hindinest.com
7. www.anubhuti-hindi.org
8. www.abhivyakti-hindi.org
9. http://wwwsamvedan.blogspot.com
10. http://www.indianrail.gov.in/index.html
11. www.rachanakar.blogspot.com
12. www.computechpub.com
13. www.desktopvidep.about.com
14. www.technorati.com
15. www.sahityakunj.net
16. www.teachersdomain.org
17. www.thisismyindia.com
18. www.mathforum.org
19. www.nirantar.org
20. www.blogger.com
21. www.hindiwriter.org
22. www.bbc.co.uk/hindi
23. www.sakshat.ac.in
24. www.sakshat.ignou.ac.in
25. www.incometaxindia.gov.in
26. www.pitra.com
27. www.education-world.com
28. www.guru.net.edu
29. www.kidsites.org
30. www.bartleyby.com(Eng lit.2 lac passages +22thousand quotations)
150 31. www.newdelhi.usembassy.gov
32. [email protected]
33. www.dilkedarmiyan.blogspot.com
34. www.south-asia.com/embassy-India
35. www.careerlauncher.com
36. www.rajbhasha.nic.in
37. www.hunkinsexperiments.com
38. www.presidentofindia.nic.in
39. www.freetranslation.com
40. http://www.kavitakosh.org/
41. http://www.udanti.com/
42. www.lekhni.net
43. http://hindihaiku.wordpress.com/
44. http://trivenni.blogspot.com/
45. www.kathakram.in
46. http://www.garbhanal.com/
47. http://yugchetna.blogspot.com/
48. http://lamhon-ka-safar.blogspot.com/
49. http://aviramsahitya.blogspot.com/
50. http://www.hindisamay.com/
151 
Fly UP